Вы находитесь на странице: 1из 99

22/03/2016

DrBhatiaMedicalCoachingInstitute:OnlineTestPlatform

Dashboard

TakeTest

Results

MyProgress

StudyMaterial

WelcomeM
GT128(AIIMSMock)

Welcome,M

DearColleagueTodaystestisAIIMSPattern.Itcontains200MCQ,tobedonein3Hrs.Ifyouhaveanysuggestionyoucancontactme
at:contactdrbhatia@gmail.comDr.Bhatia
GrandTest128(AIIMSMOCKTEST)
UserID: 30729714

Marking:+1MarksforcorrectanswerandnonegativeMarkingforincorrectanswer.

RollNo.:

TestDuration(mins):

180

Group:

TimeLeft(mins):

180

PG2015

Instructions
QuickLink
Syllabus

Oncethetesthasstarted,donotpresstherefreshbutton(orF5onyourkeyboard)
Itisadvisabletosavethetestregularlytoavoidlosingyourinformation,savetestreferstostoringtheattemptedpartofthetest.
Incaseofaccidentalfailureofinternetconnectivitythesystemwillsavetheattemptedportionofthetestautomatically.
Oncethetesttimeisover,youwillbeawardedagracedurationtowrapupthetestand"Submit"it.
Submittestreferstothefinalcompletionoftest;onceyousubmitthetestyouwillnotbeabletoedit/previewyouranswers.

TestPackage

Pleaseensurethatyouareconnectedtotheinternet,whilesubmittingthetest.

GrandTestSeries

GRANDTEST(AIIMSMOCKTEST)

2014
SubjectTestSeries
2014
AIIMSTestSeries
DNBTestSeries
SpecialVisualBased
Test
APPGFREEMOCK
PART1
GrandTestSeries
2015

Part
(Q.1) Whichofthefollowingisthetypeofjointbetweenearossicles?
(a)

Primarycartilaginous

(b)

Secondarycartilaginous

(c)

Synovial

(d)

Fibrous

YourResponse:

CorrectAnswer:

Exp:

Synovial.
TYPESOFJOINTSINBODY:HEADTOTOE:
JOINT

TYPE

SubjectTestSeries

Temporomandibularjoints

Condylar(Bicondylar)synovialjoint

2015

Cricothyroidandcricoareytenoidjoint

Synovialjoint

DNBSimplified

Atlantooccipitaljoint,Wristjointand
Metacarpophalangealjoint

Ellipsoidjoint

Sternoclavicularjoint

Saddle(sellar)joint

Earossicles

SaddletypeSynovialjoint

MelleoIncaljoint

BallandsockettypeSynovialjoint

Incudostapedialjoint

Syndesmosis

Stapesfootplate
JointbetweenalaofvomerandRostrumof
sphenoid

Schindylesis(Wedgeandgroovesuture)

Costovertebraljoint

Planesynovialjoint

Costotransversejoint

Synovialjoint

1stchondrosternaljoint,

Primarycartilaginousjoint/Synchondrosis/
Hyalinecartilaginousjoint.

AllCostochondraljoints,
Sphenooccipitaljoint
Jointbetweenepiphysisanddiaphysisof
growingbone
2nd7thchondrosternaljoint

Synovialjoint

5th9thcostalcartilagearticulation

Synovialjoint

10thcostalcartilageisunitedto9thcostal
cartilageby

Fibroustissue.

SuperiorandinferiorRadioulnarjoints

Pivot(Trochoid)joint

MedianAtlantoaxialjoint

http://dbmci.examonair.com/Result/ShowAllQuestionInHtml.aspx?testid=3572

1/99

22/03/2016

DrBhatiaMedicalCoachingInstitute:OnlineTestPlatform
Middleradioulnarjoint

Syndesmosis

Sternoclavicularjoint

Saddlejoint

ElbowandAnklejoint

Hingesynovialjoint

Hipjoint

Ballandsocketsynovialjoint

Kneejoint

Compoundsynovialjoint
2condylarjointsbetweenmedialandlateral
condylesoffemurandtibia.
1saddlejointbetweenfemurandpatella.

SuperiorTibiofibularjoint

Planesynovialjoint

MiddleTibiofibularjoint

Fibrousjoint

InferiorTibiofibularjoint

Syndesmosis`

Talocalcaneal/Subtalarjoint

Planesynovialjoint

Calcaneocuboidjoint

Saddlejoint

Talocalcaneonavicularjoint

Ballandsocketsynovialjoint

Otherintertarsaljoints

Planesynovialjoint

Symphysispubis

Secondarycartilaginousjoint/Symphyses/
Fibrocartilagenousjoint

Sacrococcygealjoint
Manubriosternaljoint
Intervertebraljoints
(Q.2) Ipsilateralhorizontalgazeisimpairedinlesioninvolving
(a)

Oculomotornucleus

(b)

Abducensnucleus

(c)

Trochlearnucleus

(d)

Vestibularnucleus

YourResponse:

CorrectAnswer:

Exp:

Abducensnucleus(RefHarrisonsprinciplesofinternalmedicine16thed.Chapter25)
HorizontalGaze
Descendingcorticalinputsmediatinghorizontalgazeultimatelyconvergeatthelevelofthepons.
Neuronsintheparamedianpontinereticularformationareresponsibleforcontrollingconjugategazetoward
thesameside.
Theyprojectdirectlytotheipsilateralabducensnucleus.
Alesionofeithertheparamedianpontinereticularformationortheabducensnucleuscausesanipsilateral
conjugategazepalsy.
Lesionsateitherlocusproducenearlyidenticalclinicalsyndromes,withthefollowingexception:vestibular
stimulation(oculocephalicmaneuverorcaloricirrigation)willsucceedindrivingtheeyesconjugatelytothe
sideinapatientwithalesionoftheparamedianpontinereticularformation,butnotinapatientwitha
lesionoftheabducensnucleus.

(Q.3) Trojanigradingsystemisforwhichofthefollowingtumors?
(a)

Bonetumors

(b)

Softtissuesarcomas

(c)

Vasculartumor

(d)

Braintumor

YourResponse:

CorrectAnswer:

Exp:

Softtissuetumors
Baily&Love25thedition,page556
Trojanigradingisusedforgradingsofttissuesarcomas.
thisincludesfourparameters1.Tumordifferentiation,2.Mitoticcount,3.Tumornecrosis,4.Histologic
grade
BonetumorsEnnekingstagingsystem
Softtissuesarcoma:
Twocelltypescanbeseenmicroscopicallyinsynovialsarcoma.Onefibroustype,knownasaspindleor
sarcomatouscell,isrelativelysmallanduniform,andfoundinsheets.Theotherisepithelialinappearance.
Classicalsynovialsarcomahasabiphasicappearancewithbothtypespresent.Synovialsarcomacanalso
appeartobepoorlydifferentiatedortobemonophasicfibrous,consistingonlyofsheetsofspindlecells.
Someauthoritiesstatethat,extremelyrarely,therecanbeamonophasicepithelialformwhichcauses
difficultyindifferentialdiagnosis.Likeothersofttissuesarcomas,thereisnouniversalgradingsystemfor
reportinghistopathologyresults.InEurope,theTrojaniorFrenchsystemisgaininginpopularitywhile
theNCIgradingsystemismorecommonintheUnitedStates.TheTrojanisystemscoresthesample,
dependingontumourdifferentiation,mitoticindex,andtumournecrosis,between0and6andthenconverts
thisintoagradeofbetween1and3,with1representingalessaggressivetumour.TheNCIsystemisalsoa
threegradeone,buttakesanumberofotherfactorsintoaccount.

http://dbmci.examonair.com/Result/ShowAllQuestionInHtml.aspx?testid=3572

2/99

22/03/2016

DrBhatiaMedicalCoachingInstitute:OnlineTestPlatform
(Q.4) Whichofthefollowingisthenucleusoftrigeminalnervewheretheafferentarch(impulses)formasseterreflexarecarried?
(a)

Spinalnucleus

(b)

Mesencephalicnucleus

(c)

Sensorynucleus

(d)

Motornucleus

YourResponse:

CorrectAnswer:

Exp:

Mesencephalicnucleus
TRIGEMINALNERVENUCLEI
MotorNucleus
ThemotornucleusofCNVislocatedintheponsjustmedialtothemainsensorynucleusofthetrigeminal
andadjacenttothepointofexitorentryofthetrigeminalnervefibers.Thesemotorfiberssupplythe
musclesofmastication(masseter,temporalis,andmedialandlateralpterygoid;
SensoryNucleus
Themainsensorynucleusislocatedjustlateraltothemotornucleus.
Themainsensorynucleusreceivestactileandpressuresensationsfromtheface,scalp,oralcavity,nasal
cavity,anddura.
SpinalTrigeminalNucleus
Thespinaltrigeminalnucleusisacaudalcontinuationofthemainsensorynucleus,extendingfromthemid
ponsthroughthemedullatothecervicalcord.Centralprocessesfromcellsinthetrigeminalganglion
conveyingpainandtemperaturesensationsfromthefacedescendinthespinaltractofVandsynapseon
cellsinthespinalnucleus.
MesencephalicNucleus
ThemesencephalicnucleusofCNVislocatedatthepointofentryofthefifthnerveandextendsintothe
midbrain.Itreceivesproprioceptiveinputfromjoints,musclesofmastication,extraocularmuscles,teeth,
andtheperiodontium.Someofthesefiberssynapsemonosynapticallyonthemotoneurons,formingthe
sensorylimbofthejawjerkreflex(Massetermuscle).
Thereisonlyoneareaofthenervoussystemwherethenucleiofprimarysensoryneuronsarelocatedwithin
theCNSratherthaninoutsideganglia.ThisisthemesencephalicnucleusofCN5whichcontainsthenucleiof
CN5proprioceptivefibers.
Locusceruleusisanorepinephrinecontainingbrainstemnucleusthatliesnearthemesencephalicnucleusof
CN5.Itprojectstowidespreadareasofthebrainandmayhaveageneraleffectonmodulatingbrainfunction

(Q.5) WagnerandGrossmanshypothesispertainsto?
(a)

Jointinnervation

(b)

Spinalintersegmantalmovements

(c)

Bonedensitychanges

(d)

Recurrentlaryngealnervepalsy

YourResponse:

CorrectAnswer:

Exp:

Recurrentlaryngealnervepalsy
Laws/Formulations/ConceptsinAnatomy:
Hiltonslaw:Nervesupplytoamusclewhichliesacrossajointnotonlysuppliesthatmusclebutalsosupllies
thejointunderneathandtheskinoverlyingthemuscle.
Lastsformulation:4contiguousspinalsegmentsregulatethemovementofajoint.Upper2segments
controlonemovementsandlower2segmentsregulatetheoppositemovement.
Wolffslaw:Alawstatingthatbonedensitychangesinresponsetochangesinthefunctionalforcesonthe
bone.Wolff(18361902)proposedthatchangesintheformandfunctionofbones,orchangesinfunction
alone,arefollowedbychangesintheinternalstructureandshapeoftheboneinaccordancewith
mathematicallaws.Thus,inmaturebonewherethegeneralformisestablished,theboneelementsplaceor
displacethemselves,anddecreaseorincreasetheirmass,inresponsetothemechanicaldemandsimposed
onthem.Thetheoryissupportedbytheobservationthatbonesatrophywhentheyarenotmechanically
stressedandhypertrophywhentheyarestressed.AlthoughWolff'sproposalrelatesspecificallytobone,the
lawhasalsobeenappliedtootherconnectivetissuessuchasligamentsandtendons.
Gardnersconceptofjointinnervation:Eachnerveinnervatesaspecificregionofthejointcapsuleandthat
thepartofthecapsulewhichismadetautbyagivenmuscleisinnervatedbythenervesupplyingits
antagonist.
DuBoisformula:forcalculatingsurfaceareaofthebody.
A=WxHx
71.84
whereW=weightinkg,H=heightincm.
NeurontheoryofWaldeyer:Contactbetweenneuronsisbycontiguityandnotbycontinuity.
Semonslaw:Inprogressivelesionsoftherecurrentlaryngealnerveabductormusclesoflarynx(Postcrico
arytenoid)are1sttobeparalysedlasttorecoverascomparedtoadductors.Infunctionalparalysisof
recurrentlaryngealnerveadductorsare1stparalysedandrecoverthelast.
Wallenslawofdegeneration:Ifaspinalnerveiscompletelydividedthedistalportionundergoesfatty
degeneration.
WagnerandGrossmanshypothesis:Inrecurrentlaryngealnerveparalysisthecricothyroidmuscleisspared

http://dbmci.examonair.com/Result/ShowAllQuestionInHtml.aspx?testid=3572

3/99

22/03/2016

DrBhatiaMedicalCoachingInstitute:OnlineTestPlatform
whichkeepsthecordsinparamedianpositionduetoitsadductoraction.
(Q.6) Kegelexercisesareadvisedinwhichcondition?
(a)

JRA

(b)

Overactivebladder

(c)

Duchennemyopathy

(d)

Talardomefracture

YourResponse:

CorrectAnswer:

Exp:

Overactivebladder
RefNelson,18thedition,page2252
Kegelexercisesareusedtostrengthenpelvicfloormusclesinvolvedinurination.theseexercisestreatboth
stressincontinenceandurgeincontinence(overactivebladder).Theseexercisesarealsoadvicedto
pregnantfemalestopreventincontinence
IndicationsforFEMALES:
Factorssuchaspregnancy,childbirth,aging,beingoverweight,andabdominalsurgerysuchascesarean
section,oftenresultintheweakeningofthepelvicmuscles.Thiscanbeassessedbyeitherdigital
examinationofvaginalpressureorusingaKegelperineometer.Kegelexercisesareusefulinregainingpelvic
floormusclestrengthinsuchcases.
Urinaryincontinence
Pelvicfloorexercisesarerecommendedforwomenwithurinaryincontinenceofthestress,urge,ormixed
types.Thereistentativeevidencethatbiofeedbackmaygiveaddedbenefitwhenusedwithpelvicfloormuscle
training.
Pelvicprolapse
Thereissomeevidenceshowingapositiveeffectofpelvicfloorexercisesonthesymptomsofprolapseand
itsseverity
Sexualfunction
In1952,Dr.Kegelpublishedareportinwhichheclaimedthatthewomendoinghisexerciseswereorgasming
moreeasily,morefrequentlyandmoreintensely:"ithasbeenfoundthatdysfunctionofthepubococcygeus
existsinmanywomencomplainingoflackofvaginalfeelingduringcoitusandthatinthesecasessexual
appreciationcanbeincreasedbyrestoringfunctionofthepubococcygeus."Thereiscontroversyoverthe
preciseeffectsofthePCmuscleonorgasticresponsebutcertainbenefitsofastrongpelvicfloorarewell
accepted.
IndicationforMEN:
Thoughmostcommonlyusedbywomen,mencanalsouseKegelexercises.Kegelexercisesareemployedto
strengthenthepubococcygealmuscleandothermusclesofthepelvicdiaphragm.Kegelscanhelpmenachieve
strongererections,maintainhealthyhips,andgaingreatercontroloverejaculation.Theobjectiveofthismay
besimilartothatoftheexerciseinwomenwithweakenedpelvicfloor:toincreasebladderandbowel
controlandsexualfunction.
Urinaryincontinence
Afteraprostatectomythereisnoclearevidencethatteachingpelvicfloorexercisesalterstheriskofurinary
incontinence(leakageofurine).
Sexualfunction
Kegelworkoutscanprovidemenwithstrongererections.Researchpublishedin2005issueofBJU
Internationalhasshownthatpelvicfloorexercisescouldhelprestoreerectilefunctioninmenwitherectile
dysfunction.Therearesaidtobesignificantbenefitsfortheproblemofprematureejaculationfromhaving
moremuscularcontrolofthepelvis.

(Q.7) Facialcolliculusislocatedin?
(a)

Pons

(b)

Medulla

(c)

Midbrain

(d)

Interpeduncularfossa

YourResponse:

CorrectAnswer:

Exp:

Pons
Facialcolliculusisanelevatedarealocatedonthedorsalponsintheflooroffourthventricle,oneoneither
sideofmediansulcus.Itisformedbyfibersfromthemotornucleusoffacialnerveastheyloopoverthe
abducensnucleus
Facialcolliculussyndromeinvolveslesionofthesethree
facialnervefiberatgenucausingipsilateralfacialpalsy
abducensnervenucleus
mediallongitudinalfasciculus

http://dbmci.examonair.com/Result/ShowAllQuestionInHtml.aspx?testid=3572

4/99

22/03/2016

DrBhatiaMedicalCoachingInstitute:OnlineTestPlatform
Nerve

Components

Function

OpeninginSkull

I.

Olfactory

Sensory

Smell

Openingsin
cribriformplateof
ethmoid

II.

Optic

Sensory

Vision

Opticcanal

III.

Oculomotor

Motor

Liftsuppereyelid,turnseyeball
upward,downward,andmedially;
constrictspupil;accommodateseye

Superiororbital
fissure

IV.

Trochlear

Motor

Assistsinturningeyeballdownward
andlaterally

Superiororbital
fissure

V.

Trigeminal
Sensory

Cornea,skinofforehead,scalp,eyelids, Superiororbital
andnose;alsomucousmembraneof fissure
paranasalsinusesandnasalcavity

Ophthalmic
division

Maxillarydivision Sensory

Skinoffaceovermaxillaandtheupper Foramenrotundum
lip;teethofupperjaw;mucous
membraneofnose,themaxillaryair
sinus,andpalate

Mandibular
division

Motor

Musclesofmastication,mylohyoid,
anteriorbellyofdigastric,tensorveli
palatini,andtensortympani

Sensory

Skinofcheek,skinovermandible,
lowerlip,andsideofhead;teethof
lowerjawandtemporomandibular
joint;mucousmembraneofmouthand
anteriortwothirdsoftongue

Foramenovale

VI.

Abducent

Motor

Lateralrectusmuscle:turnseyeball
laterally

Superiororbital
fissure

VII.

Facial

Motor

Musclesofface,cheek,andscalp;
stapediusmuscleofmiddleear;
stylohyoid;andposteriorbellyof
digastric

Internalacoustic
meatus,facialcanal,
stylomastoid
foramen

Sensory

Tastefromanteriortwothirdsof
tongue,floorofmouth,andpalate

Secretomotor

Submandibularandsublingualsalivary
parasympatheticglands,lacrimalgland,
andglandsofnoseandpalate

Vestibular

Sensory

Positionandmovementofhead

Cochlear

Sensory

Hearing

VIII.

IX.

X.

XI.

XII.

Vestibulocochlear

Glossopharyngeal Motor

Internalacoustic
meatus

Stylopharyngeusmuscle:assists
swallowing

Secretomotor
parasympathetic

Parotidsalivarygland

Sensory

Generalsensationandtastefrom
posteriorthirdoftongueandpharynx;
carotidsinusandcarotidbody

Motor

Constrictormusclesofpharynxand
Jugularforamen
intrinsicmusclesoflarynx;involuntary
muscleoftracheaandbronchi,heart,
alimentarytractfrompharynxto
splenicflexureofcolon;liverand
pancreas

Sensory

Tastefromepiglottisandvalleculaand
afferentfibersfromstructuresnamed
above

Cranialroot

Motor

Musclesofsoftpalate,pharynx,and
larynx

Spinalroot

Motor

Sternocleidomastoidandtrapezius
muscles

Hypoglossal

Motor

Musclesoftonguecontrollingitsshape Hypoglossalcanal
andmovement(exceptpalatoglossus)

Vagus

Jugularforamen

Accessory
Jugularforamen

(Q.8) Whichofthefollowingisnotafeatureofradialnerveinjuryatupperarm?
(a)

WeaknessofBrachioradialis

(b)

Inabilitytoextendfingers

(c)

Paralysisofextensorcarpiradialisbravis

(d)

Lossofsensationsoverdorsumofhand

http://dbmci.examonair.com/Result/ShowAllQuestionInHtml.aspx?testid=3572

5/99

22/03/2016

DrBhatiaMedicalCoachingInstitute:OnlineTestPlatform
YourResponse:

CorrectAnswer:

Exp:

WeaknessofBrachioradialis
Radialnerveisknownasthegreatextensornerve.
Causesofinjury
Theradialnervemaybedamagedbytraumaorentrapped,especiallybetweentheheadsofmuscles.
Intheaxilla:
Withfeaturesofweaktriceps,wristdropandpossiblyalsomedianandulnarnerveinvolvement.Themost
commoncauseiscompression.
Theradialnervemaybedamagedintheaxillabyfractureordislocationoftheheadofthehumerus.
Saturdaynightsyndrome(sonamedbecauseitcanbeacquiredbysleepingwiththearmoverthebackofa
chairwhilstinadrunkenstupor,socompressingtheplexus):
Isduetocompressionofthelowerpartofthebrachialplexus.Asthisisreallyabrachialplexusinjury,the
medianandulnarnervesmayalsobeinvolved.
Itmayalsobecompressedbytheuseofshouldercrutches.
Nervefunctionusuallyfullyrecoverswithinafewweeks.
Intheupperarm(tricepsandbrachioradialisareoftenspared):
Maybeduetoacompressionlesionbutfractureistheusualcause.Injectionsgiveninthearmofsmall
babiescandamagetheradialnerve.
Asthenerveoftenpassesdowninthespiralgrooveofthehumerus,itmaybeinjuredwithafractureofthe
shaftofthehumerus.
Attheelbow:
Theradialnervemaybeentrappedattheelbowatanumberofsitesbutthemostcommonistheproximal
borderofthetendonofsupinatorcalledthearcadeofFrohse.
Checkfortendernessovertheradialtunnel.Theremaybepainwhenthefingersareextendedagainst
resistance.
Supinationfromapronatedpositionalongwithflexionofthewristmayreproducethesymptoms.
Lesionsatthewrist:
Causefingerdropwithanormalwristandintactsensation.
Causesincludefractureoftheradius,elbowdeformity,softtissuemassesandcompressionbytheextensor
carpiradialisbrevis.
Lesionsofthesuperficialnerves(causepainandsensorylossbutnomotorloss):
Attheelbow,rupturedsynovialeffusionisthemostcommoncause.Intheforearmtheremaybeanaberrant
coursethroughthemuscles.
Atthewrist,causesincludecompressionfromplastercasts,wristbandsorhandcuffs,especiallythetypethan
gettighterwithstruggling.Othercausesaresurgery,injectionsandnervetumours.
Injurytoradialnerveatdifferentlevelscausesdifferentsyndromeswithvaryingmotorandsensorydeficits.
Attheaxilla
Commonmechanismsofinjury:Saturdaynightpalsy,crutchpalsy
Motordeficit:
Lossofextensionofforearm,weaknessofsupination,andlossofextensionofhandandfingers.
Presenceofwristdrop,duetoinabilitytoextendthehandandfingers.
Sensorydeficit:
Lossofsensationinlateralarm,posteriorforearm,theradialhalfofdorsumofhand,anddorsalaspectof
radial3digits,excludingtheirnailbeds.
Atmidarm
Commonmechanismofinjury:Midshafthumeralfracture
Motordeficit:
Weaknessofsupination,andlossofextensionofhandandfingers.
Presenceofwristdrop,duetoinabilitytoextendthehandandfingers.
Sensorydeficit:Lossofsensationinposteriorforearm,theradialhalfofdorsumofhand,anddorsalaspectof
radial3digits,excludingtheirnailbeds.
Justbelowtheelbow
Commonmechanismofinjury:Neckofradiusfracture,elbowdislocationorfracture,tightcast,rheumatoid
nodules,injectionsduetotenniselbow,injuringthedeepbranchoftheradialnervethatpiercestheradial
head,causingposteriorinterosseousnervesyndrome
Motordeficit:
Weaknessinextensionofhandandlossofextensionoffingers.Presenceoffingerdrop,and
partialwristdrop,sincetheextensorcarpiradialislongusandbrachioradialismusclesareworking.
Sensorydeficit:None,assensationissuppliedbythesuperficialradialnerve
Soinlowlevelradialnerveinjurybrachioradialisisstillworkingsothereisonlypartialwristdrop.
RadialnerveprovidesinnervationoftheBrachioradialis,Extensorsofthewristandfingers,Supinator,and
Triceps.

SR.NERVEMUSCLEGROUPFUNCTIONALDEFICITORSIGN

1.LongthoracicSerratusanteriorWingingscapula

http://dbmci.examonair.com/Result/ShowAllQuestionInHtml.aspx?testid=3572

6/99

22/03/2016

DrBhatiaMedicalCoachingInstitute:OnlineTestPlatform
2.SuprascapularSupraspinatusandDifficultyininitiatingarm
Infraspinatusabduction
3.AxillaryDeltoidandTeresminorInabilitytofullyabductarm
4.RadialExtensorsofforearms,Lossofarmextension,lossof
wristproximalphalanforeamsextension,supination,
gesandthumbabduction,lossofwristextension
(wristdrop),lossofproximal
phalangealextensionandthumbextension.
5.MusculoFlexorsofarmandWeakarmandforearmflexion,
cutaneousforearmweakforearmsupination.
6.MedianWristandhandflexorsParalysisofflexor,pronator,and
thenarmuscles,inabilitytofully
flextheindexandmiddlefingers
(signofbenediction),Pointing
indexfinger,Labourersnerve.
7.UlnarWristandhandflexorsInabilitytoextendthedistaland
middlephalanges(clawhand);
lossofthumbabduction,
Fromentsignpositive.
(Q.9) ThePhysiologicallockingoftheKNEEjointoccursasaresultof?
(a)

Medialrotationoftibiaoverthefixedfemur

(b)

Lateralrotationoffemuroverthetibia

(c)

Lateralrotationoftibiaoverthefemur

(d)

Medialrotationoffemuroverthefixedtibia

YourResponse:

CorrectAnswer:

Exp:

Medialrotationoffemuroverthefixedtibia(RefBDChaurasiaVol.II,3rded.130)
LOCKINGANDUNLOCKINGOFKNEEJOINT
Lockingofthekneejointoccursasaresultofmedialrotationofthefemurduringthelaststageofextension.
TheAPdiameterofthelateralfemoralcondyleislessthanthatofthemedialcondyle.Asaresult,whenthe
lateralcondylararticularsurfaceisfullyusedupbyextension,partofmedialcondylarsurfaceremains
unused.
Atthisstagethelateralcondyleactsasanaxisaroundwhichmedialcondylerotatesbackwards(i.e.medial
rotationofthefemuroccurs),sothattheremainingpartofthemedialcondylarsurfaceisalsotaken
up.Thismovementlocksthekneejoint.
*LockingisproducedbyactionofQuadricepsfemoris(italsoproducesextension).
*Unclockingisbroughtaboutbytheactionofthepopliteusmuscle(popliteusalsoprotectslateralmeniscus
bypullingitbackwards).

(Q.10) Stochasticeffectisseeninwhichcondition?
(a)

Cancerchemotherapy

(b)

Radiotherapy

(c)

Posttransplantimmunosuppressivetherapy

(d)

Aftercoronaryarterybypasssurgery

YourResponse:

CorrectAnswer:

Exp:

RadiotherapyRefNelson,18thedition,page2899
BIOLOGICEFFECTSOFRADIATION
Stochastic(random)effectsareofgreaterconcernbecausetheycanoccuratanydose;i.e.,thereisno
threshold,withtheprobabilityofaneffectincreasingwithincreasingdose.Theseeffectscanbecausedby
anyradiationstrikingvulnerabletissue(mostimportantlyDNA,butcytoplasmalsomaybeatrisk)and
causingirreversibledamage.Theseeffectsleadtothelinearnodosethreshold(LNT)concept,whichstates
thatradiationdamageincreaseswithincreasingdoseinalinearfashion.Thisconceptstressesthatnolevel
ofradiationexposurecanbeconsideredtobeabsolutelysafe

(Q.11) ThetermZeitgebersisusedinrelationto?
(a)

Intrinsiccircadianrhythm

(b)

Arrhythmias

(c)

GImotility

(d)

Nutrition

YourResponse:

CorrectAnswer:

Exp:

Intrinsiccircadianrhythm(RefNelson,18thedition,page9192)

http://dbmci.examonair.com/Result/ShowAllQuestionInHtml.aspx?testid=3572

7/99

22/03/2016

DrBhatiaMedicalCoachingInstitute:OnlineTestPlatform
Theterm"zeitgeber"(Germanfor"timegiver"or"synchronizer")wasfirstusedbyJrgenAschoff,oneof
thefoundersofthefieldofchronobiology.Hisworkdemonstratedtheexistenceofendogenous(internal)
biologicalclocks,whichsynchronizebiologicalrhythms.Inaddition,hefoundthatcertainexogenous
(external)cues,whichhecalledzeitgebers,influencethetimingoftheseinternalclocks.
Intrinsiccircadianrhythms,whichgovernmanyphysiologicprocessesinadditiontosleepwakerhythms,areinturn
affectedtoadegreebyexternaltimecues,orzeitgebers(timingofmeals,alarmclocks),buttheyaremostsensitiveto
lightdarknesssignals,whichswitchtheproductionofthehormonemelatoninbythepinealglandoff(light)oron(dark).
(Q.12)

Falseregardingthe1stPharyngealarchis?
(a) MaxillaryandMandibulardivisionsoftrigeminalnervearetherelatednerves
(b) Musclesofmasticationdevelopfromit
(c) Mandibledevelopsfromit
(d) Stapedialarteryisthearchartery
YourResponse:

CorrectAnswer:

Exp:

Stapedialarteryisthearchartery
HEADANDNECKEMBRYOLOGY:SUMMARIZED:
Pharyngeal ArchArtery

CranialNerve

Skeletalelements

Muscles

Arch
1

TerminalBranch Maxillaryand
ofmaxillary
mandibular
artery
divisionof
trigemenial(V)

Derivedfromarch
cartilages(originating
fromneuralcrest):

Musclesof
mastication
(temporalis,masseter,
Frommaxillarycartilages: andpterygoids),
mylohyoid,anterior
Alispenoid,incus
bellyofdigastric,
Frommandibular:
tensortympani,tensor
velipalatini(originate
Meckels
fromcranial
cartilage,malleus
somitomere4)
Upperportionofexternal
ear(auricle)isderived
fromdorsalaspectof1st
pharyngealarch.
Derivedbydirect
ossificationfromarch
dermalmesenchyme:
Maxilla,zygomatic,
squamousportionof
temporalbone,mandible

Stapediusartery Facialnerve(VII)
(embryologic)
and
cortiotympanic
artery(adult)

Common
Glossopharyngeal Lowerrimandgreater
carotidartery, (IX)
hornofhyoid(derived
mostofinternal
fromthethirdarch
carotid
cartilage;originatefrom
neuralcrestcells)

Left:Archof
aorta;
Right:Right
subclavian
artery;

Stapes,styloidprocess,
Musclesoffacial
stylohyoidligament,lesser expression(orbicularis
hornsandupperrimof
oculi,orbicularisoris,
hyoid(derivedfromthe auricularis,platysma,
secondarchcartilage;
frontoooccipitalis,
originatefromneural
buccinator),posterior
crest).
bellyofdigastric,
Lowerportionofexternal stylohyoid,stapedius
(originatefromcranial
ear(auricle)isderived
somitomere6)
from2ndpharyngealarch.
Sytlopharyngeus
(originatefromcranial
somitomere7)

Superiorlaryngeal Laryngealcartilages
Constrictorsof
branchofvagus (Derivedfromthe4tharch pharynx,cricothyroid,
(X)
cartilage,originatefrom levatorvelipalatine
lateralplatemesoderm) (originatefrom
occipitalsomites24)

Originalsprouts
ofpulmonary
arteries
6

Ductus
arteriosus;
rootsof
definitive
pulmonary
arteries

Recurrent
laryngealbranch
ofvagus(X)

Laryngealcartilages
Intrinsicmusclesof
(derivedfromthe6tharch larynx(originatefrom
cartilage;originatefrom occipitalsomites1and
lateralplatemesoderm) 2)

Originsofcraniofacialmuscles:
MesodermalOrigin

Muscles

Innervation

Somitomeres1,2

Superior,medialandventralrecti

Oculomotor(III)

Somitomere3

Superioroblique

Trochlear(IV)

Somitomere4

Jawclosingmuscles

Trigeminal(V)

http://dbmci.examonair.com/Result/ShowAllQuestionInHtml.aspx?testid=3572

8/99

22/03/2016

DrBhatiaMedicalCoachingInstitute:OnlineTestPlatform
Somitomere5

Lateralrectus

Abducens(VI)

Somitomere6

Jawopeningandother2ndarchmuscles

Facial(VII)

Somitomere7

Stylopharyngeus

Glossopharyngeal(IX)

Somites1,2

Intrinsiclaryngeals

Vagus(X)

Somites25

Tonguemuscles

Hypoglossal(XII)

Derivatesofpharyngealpouches:
Auditorytube,whichcomesincontactwithepitheliallineoffirst
pharyngealcleft,wherefutureexternalacousticmeatuswillform.

Firstpouch

Distalportionwillformtympaniccavity(liningwillbecomeeardrum)
Proximalportionwillbecomeauditorytube
Secondpouch

Formsbudsthatpenetratesurroundingmesenchyme,whichtogetherform
thepalatinetonsils

Thirdpouch

Formsthymusandinferiorparathyroidglands

Fourthpouch

Formssuperiorparathyroidglands

Fifthpouch

Formsutlimobranchialbody

Derivativesofpharyngealclefts/grooves:
Initially,fourcleftsexist;however,onlyonegivesrisetoadefinitestructureinadults.
st
1 pharyngealcleft

PenetratesunderlyingmesenchymeandformsEAM.Thebottomof
EAMformslateralaspectoftympaniccavity.

2ndpharyngealcleft

Undergoesactiveproliferationandoverlapsremainingclefts.It
mergeswithectodermoflowernecksuchthattheremainingclefts
losecontactwithoutside.Temporarily,thecleftsforman
ectodermallylinedcavity,thecervicalsinus,butthisdisappears
duringdevelopment.

Viscera/organdevelopment:
Tongue

Appearsat4thweek.
Musculaturederivedfrommesodermofoccipitalsomites.Precursormusclescells
migratetoregionoftongueandareinnervatedbygeneralsensoryefferentfibers
ofCNXII.
Mucosaderivedfromanteriorendodermliningarches14;accordingly,innervation
dependsonarchderivation:
Mucosaofanterior2/3oftonguecomesfromthefirstarchCNV
Mucosaofposterior1/3oftonguecomesfromthirdandfortharchCNIX,
X
Specialtasteofanterior2/3oftonguecomesfromCNVII.
Specialtasteofposterior1/3oftonguecomesfromCNX.
Tonguefreedfromfloorofmouthbyextensivedegenerationofunderlying
tissue.Midlinefrenulumcontinuestoanchortonguetofloorofmouth.

ThyroidGland Developsasingrowthofmucosalepitheliumlocatedinthemidlineofthetongue
(atforamencecum).Itdescendsalongfrontofpharyngealgut,butremains
connectedtotonguebythyrooglossalduct,whichisobliteratedlaterin
development.Thyroidglanddescendstoapointjustcaudaltolaryngealcartilages.
Facial
structures
(general)

a)medialnasalprominenceformsmidlineofnose,philtrumandprimarypalate
b)lateralnasalprominenceformsalaeofnose
c)maxillaryprominenceformscheekregionandlaterallip
d)cleftscanformatinterprominencefusionlines

Nose

Atthetimeofanteriorneuraltubeclosure,mesenchymearoundforebrain,
frontonasalprominence(FNP),hassmoothroundedextendedcontour.Nasal
placodes(thickeningofsurfaceectodermtobecomeperipheralneuraltissue)
developonfrontolateralaspectsofFNP.Mesenchymeswellsaroundnasalplacode
producingamedialandlateralnasalprominence(nasomedialandnasolateral
processes).Thesenasalprominencesformthenose.

Mouth

Stomadeum(primitiveoralcavity)formsbetweenfrontonasalprominenceandfirst
pharyngealarch.Thefirstpharyngealarchformsthedorsalmaxillaryprominence
andventralmandibularprominence.Themaxillaryprominencewillmergewith
medialnasalprominences,pushingthemclosertocausefusion.Fusedmedial
nasalprominenceswillformmidlineofnoseandmidlineofupperlip(philtrum)and
primarypalate(first4teeth).

Nasolacrimal Maxillaryandlateralnasalprominencesareseparatedbydeepfurrow,the
structures
nasolacrimalgroove.Ectoderminfloorofgrooveformsepithelialcord,which
detachesfromoverlyingectoderm.Theepithelialcordcanalizestoformthe
nasolacrimalduct.Theupperendoftheductwidenstoformthelacrimal
sac.Afterdetachmentofthecord,themaxillaryandlateralnasalprominences
mergewitheachother,resultingintheformationofanasolacrimalductthatruns
fromthemedialcorneroftheeyetotheinferiormeatusofthenasalcavity.
Themaxillaryprominencesenlargetoformthecheeksandmaxillae.
Thelateralnasalprominencesformthealaeofthenose.
Secondary
Mainpartofdefinitivepalateformedbytwopalatineshelvesderivedfrom
(hard)palate intraoralbilateralextensionsofthemaxillaryprominences.Theseappearatthe
6thweek.Theyaredirectedobliquelydownwardoneachsideofthetongue;they
movedownwhenmandiblegetsbigger.
Attheseventhweek,theyascendtoattainahorizontalposition,thenfusetoform

http://dbmci.examonair.com/Result/ShowAllQuestionInHtml.aspx?testid=3572

9/99

22/03/2016

DrBhatiaMedicalCoachingInstitute:OnlineTestPlatform
thesecondarypalate.Atthetimethepalatineshelvesfuse,thenasalseptum(an
outgrowthofmediantissueofthefrontonasalprominence)growsdownandjoins
thecephalicaspectofthenewlyformedpalate
Anteriorly,shelvesfusewithtriangularprimarypalate.Theincisiveforamenmarks
themidlinebetweentheprimaryandsecondarypalate.
ExternalEar

Theauricleisderivedfrom6auricularhillocks(mesenchymalproliferations)along
thedorsalaspectofarches1(topofear)and2(bottomofear).Thesefusetoform
thedefinitiveauricle.Atthemandiblegrows,theearispushedupwardand
backwardfromitsinitialhorizontalpositionontheneck.
TheEAMisderivedfromthe1stpharyngealarch.
Theeardrum(tympanicmembrane)iscomposedof3layersofcells:1)ectodermal
epithelialliningofbottomofEAM;2)endodermalepitheliumliningoftympanic
cavity;3)intermediatelayerofconnectivetissue.
Theeardrumiscomposedofmultiplecelllayersbecauseitrepresentsthefirst
pharyngealmembrane,andthusliesatthejunctionofthefirstpharyngealpouch
andcleft.

MiddleEar

Themiddleearconsistsofanauditorytube(fromthe1stpharyngealpouch,along
withtympaniccavity)andtheossicles(frompharyngealarches1and2cartilage).
Thefirstarchcartilageformsthemalleusandincus.Thetensortympani(muscleof
themalleus)isderivedfromthefourthsomitomere(associatedwiththefirstarch)
andisthereforeinnervatedbyCNV.
Thesecondarchcartilageformsthestapes.Thestapedius(musclesofthestapes)
isderivedfromthesixthsomitomere(associatedwiththesecondarch)andis
thereforeinnervatedbyCNVII.
Theossiclesareinitiallyembeddedinmesenchyme,butinthe8thmonth,the
mesenchymedegeneratesandanendodermalepithelialliningofthetympanic
cavityenvelopstheossiclesandconnectsthemtothewallofthecavityina
mesenterylikefashion.

InnerEar

Theinnerearisderivedthickeningofsurfaceectodermonbothsidesofthe
hindbrain(oticplacodes).Theplacodesinvaginatetoformoticvesicles
(otocytes).Thevesiclesthendivideintoventralanddorsalcomponents.
Theventralcomponentformsthesacculeandcochlearduct.
Thedorsalcomponentformstheutricleandsemicircularcanalsandendolymphatic
duct.

CochlearDuct Derivedfromanoutgrowthofthesacculeduringthe6thweek.Theoutgrowth
penetratesthesurroundingmesenchymeinaspiralfashion.Thesurrounding
mesenchymeformsthecartilageandundergoesvacuolization.
Thescalavestibuleandscaletympaniformandsurroundthecochlearduct.They
arefilledwithperiplymptoreceivemechanicalvibrationsofossicles.The
mechanicalstimuliactivatessensory(ciliary)cellsinthecochlearduct.
Semicircular
canals

Theutricleisinitiallythreeflattenedoutpocketings,whichlosethecentral
core.Fromthisthreesemicircularcanalsareforms,eachat90degreeanglesfrom
oneanother.Sensorycellsariseintheampullaatoneendofeachcanal,inthe
utricleandsaccule.

Eye

Atweek4,twodepressionsareevidentoneachoftheforebrainhemispheres.As
theanteriorneuralfoldcloses,theopticpitselongatetoformtheoptic
vesicles.Theopticvesiclesremainconnectedtotheforebrainbyopticstalks.
Theinvaginationoftheopticvesiclesformsabilayeredopticcup.Thebilayered
cupbecomestheduallayeredretina(neuralandpigmentedlayer)
Surfaceectodermformsthelensplacode,whichinvaginateswiththeopticcup.
Theopticstalkisdeficientventrallytocontainchoroidsfissuretoallowblood
vesselsintotheeye(hyaloidartery).Thearteryfeedsthegrowinglens,butwillits
distalportionwilleventuallydegeneratesuchthattheadultlensreceivesno
hyaloidvasculature.
Atthe7thweek,thechoroidsfissureclosesandwallsfuseastheretinalnerveget
bigger.
Theanteriorrimoftheopticvesiclesformstheretinaandiris.Theirisisan
outgrowthofthedistaledgeoftheretina.
Opticvesiclesinduces/maintainsthedevelopmentofthelensvesicle,whichforms
thedefinitivelens.Followingseparationofthelensvesiclefromthesurface
ectoderm,thecorneadevelopsintheanterior1/5thoftheeye.
Thelensandretinaaresurroundedbymesenchymewhichformsatough
connectivetissue,thesclera,thatiscontinuouswiththeduramateraroundthe
opticnerve.
Iridopupillarymembraneformstoseparatetheanteriorandposteriorchambersof
theeye.Themembranebreaksdowntoallowforthepupil
Mesenchymesurroundingtheformingeyeformsmusculature(ciliarymusclesand
pupillarymusclesfromsomitomeres1and2;innervatedbyCNIII),supportive
connectivetissueelementsandvasculature.

Eyelids

Formedbyanoutgrowthofectodermthatisfusedatitsmidlineinthe2nd
trimester,butlaterreopen.

(Q.13) Pickthewrongmatchofcoronaryarteryinvolvementandmyocardialinfarctlocationbasedonthenormalpatternofbloodsupplyto

theheart:

http://dbmci.examonair.com/Result/ShowAllQuestionInHtml.aspx?testid=3572

10/99

22/03/2016

DrBhatiaMedicalCoachingInstitute:OnlineTestPlatform
(a)

ProximalLADAnteriorwall

(b)

CircumflexarteryPosterolateralwall

(c)

DistalRightCoronaryarteryInferiorwall

(d)

Earlyobtuse,marginalarteryRightventricularmedialwall

YourResponse:

CorrectAnswer:

Exp:

Earlyobtuse,marginalartery

D.

Rightventricular
medialwall

ClinicalAnatomybyRegions;RichardSnell8thed,Chap3
CoronaryArteryLesions,InfarctLocation,andECGSignature
CoronaryArtery

InfarctLocation

ECGSignature

ProximalLAD

Largeanteriorwall

STelevation:I,L,V1,V6

MoredistalLAD

Anteroapical

STelevation:V2,V4

Inferiorwallifwraparound
LAD

STelevation:II,III,F

DistalLAD

Anteroseptal

STelevation:V1,V3

Earlyobtuse,marginal

Highlateralwall

STelevation:I,L,V4,V6

Moredistalmarginalbranch, Smalllateralwall
circumflex

STelevation:I,L,orV4,V6,ornoabnormality

Circumflex

Posterolateral

STelevation:V4,STdepression:V1,V2

DistalRCA

Smallinferiorwall

STelevation:II,III,F;STdepression:I,L

ProximalRCA

Largeinferiorwalland
posteriorwallSomelateral
wall

STelevation:II,III,F;STdepression:I,L,
V1,V3STelevation:VV6

RCA

Rightventricular

STelevation:V2,V4;someSTelevation:V1;or
STdepression:V2,V3

Usuallyinferior

STelevation:II,III,F

ECG,electrocardiographic;LAD,leftanteriordescending(interventricular);RCA,rightcoronaryartery.

(Q.14) Whichofthefollowingisnotdirectlyconnectedtothecavernoussinus?
(a)

Superficialmiddlecerebralvein

(b)

Superiorpetrosalsinus

(c)

Inferiorpetrosalsinus

(d)

Deepcerebralvein

YourResponse:

CorrectAnswer:

Exp:

Deepcerebralvein
*Eachcavernoussinusreceivesbloodfromsomeofthecerebralveins,ophthalmicveins,andthe
sphenoparietalsinus.
TRIBUTARIES:
Superiorandinferiorophthalmicveins
Sphenoparietalsinus
Superficialmiddlecerebralveins
DRAINAGE:
SuperiorPetrosalsinus
Iinferiorpetrosalsinuses
Emissaryveinsthroughtheforaminaoftheskull(mostlythroughforamenovale).
Therearealsoconnectionswiththepterygoidplexusofveinsviainferiorophthalmicvein,deepfacial
veinandemissaryveins.

(Q.15) Allofthefollowinghormonesareattheirpeaklevelduringthethirdtrimesterofpregnancyexcept?
(a)

hCG

(b)

Prolactin

(c)

Estradiol

(d)

Progesterone

YourResponse:

CorrectAnswer:

Exp:

GanongReviewofMedicalPhysiology23rdEdChap25
Inallmammals,thecorpusluteumintheovaryatthetimeoffertilizationfailstoregressandinstead
enlargesinresponsetostimulationbygonadotropichormonessecretedbytheplacenta.Theplacental
gonadotropininhumansiscalledhumanchorionicgonadotropin(hCG).Theenlargedcorpusluteumof

http://dbmci.examonair.com/Result/ShowAllQuestionInHtml.aspx?testid=3572

11/99

22/03/2016

DrBhatiaMedicalCoachingInstitute:OnlineTestPlatform
pregnancysecretesestrogens,progesterone,andrelaxin.Therelaxinhelpsmaintainpregnancybyinhibiting
myometrialcontractions.Inmostspecies,removaloftheovariesatanytimeduringpregnancyprecipitates
abortion.Inhumans,however,theplacentaproducessufficientestrogenandprogesteronefrommaternal
andfetalprecursorstotakeoverthefunctionofthecorpusluteumafterthesixthweekofpregnancy.
Ovariectomybeforethesixthweekleadstoabortion,butovariectomythereafterhasnoeffectonthe
pregnancy.Thefunctionofthecorpusluteumbeginstodeclineafter8wkofpregnancy,butitpersists
throughoutpregnancy.hCGsecretiondecreasesafteraninitialmarkedrise,butestrogenandprogesterone
secretionincreaseuntiljustbeforeparturition.
HormoneLevelsinHumanMaternalBloodduringNormalPregnancy.
Hormone

ApproximatePeakValue

TimeofPeakSecretion

hCG

5mg/mL

Firsttrimester

Relaxin

1ng/mL

Firsttrimester

hCS

15mg/mL

Term

Estradiol

16ng/mL

Term

Estriol

14ng/mL

Term

Progesterone

190ng/mL

Term

Prolactin

200ng/mL

Term

HumanChorionicGonadotropin
hCGisaglycoproteinthatcontainsgalactoseandhexosamine.Itisproducedbythesyncytiotrophoblast.
Likethepituitaryglycoproteinhormones,itismadeupofandsubunits.hCGisidenticaltothe
subunitofLH,FSH,andTSH.ThemolecularweightofhCGis18,000,andthatofhCGis28,000.hCGis
primarilyluteinizingandluteotropicandhaslittleFSHactivity.Itcanbemeasuredbyradioimmunoassayand
detectedinthebloodasearlyas6dafterconception.Itspresenceintheurineinearlypregnancyisthe
basisofthevariouslaboratorytestsforpregnancy,anditcansometimesbedetectedintheurineasearlyas
14dafterconception.ItappearstoactonthesamereceptorasLH.hCGisnotabsolutelyspecificfor
pregnancy.Smallamountsaresecretedbyavarietyofgastrointestinalandothertumorsinbothsexes,and
hCGhasbeenmeasuredinindividualswithsuspectedtumorsasa"tumormarker."Italsoappearsthatthe
fetalliverandkidneynormallyproducesmallamountsofhCG.
(Q.16) MostofthefollowingGIsecretionshaveabasaloutputduringtheinterdigestiveperiod(betweenmeals).However,thesightand

smellofatastymealstimulatesGIsecretions.OfthevariousGIsecretions,whichisthemoststimulated?
(a)

Gastricsecretion

(b)

Intestinalsecretion

(c)

Pancreaticsecretion

(d)

Salivarysecretion

YourResponse:

CorrectAnswer:

Exp:

Salivarysecretion(RefGanongReviewofMedicalPhysiology23rdEdPage573)
Salivarysecretionisexclusivelyunderneuralcontrol.
Theothersneedbothneuralandhormonalstimulationandare,therefore,onlypartiallystimulatedbythe
sight,smell,andchewingoffood(cephalicphase).
Thesight,smell,andchewingoffoodstimulatetheparasympatheticnervoussystem,whichstimulates
salivarysecretion.

(Q.17) Bileaciduptakebyhepatocytesisdependenton
(a)

Calcium

(b)

Iron

(c)

Sodium

(d)

Potassium

YourResponse:

CorrectAnswer:

Exp:

SodiumRefGanongReviewofMedicalPhysiology23rdEdPage643
TheBileAcid:NaSymporter(BASS)Family
FunctionallycharacterizedmembersoftheBASSfamilycatalyzeNa:bileacidsymport.Thesesystemshave
beenidentifiedinintestinal,liverandkidneytissuesofanimals,andatleastthreeisoformsarepresentina
singlespeciessuchashumans.TheBASSfamilyisalsocalledtheSoluteCarrierFamily10.
ABASSintheapicalmembraneofthehumanilealintestinecatalyzestheelectrogenicuptakeofbileacids
withastoichiometryofbileacid:Naof1:2.Thisproteinisassociatedwiththe16kDasubunitcofthe
vacuolarprotonpump(Sunetal.,2004).Thismayaccountforitsapicallocation.Thus,thevacuolarproton
pumpassociatedapicalsortingmachinerymaybeusedtosorttheapicalNa:bilesymporter.Proteinsofthe
BASSfamilyvaryinsizefromabout340to480aminoacylresiduesandpossess7to10putative
transmembranespanners(TMSs).ThebileacidbindingsiteappearstobelocalizedtothelastTMS(last60
residues)(Krameretal.,2001).TheBASSfamilybelongstotheBARTsuperfamily(Mansouretal.,2007)
Thesesymportersexhibitbroadspecificity,takingupavarietyofnonbileorganiccompoundsaswellas
taurocholateandotherbilesalts.Homologuesarefoundinplants,yeast,archaeaandbacteria.Forexample,

http://dbmci.examonair.com/Result/ShowAllQuestionInHtml.aspx?testid=3572

12/99

22/03/2016

DrBhatiaMedicalCoachingInstitute:OnlineTestPlatform
functionallyuncharacterizedhomologuesarepresentinSynechocystis(292aas;gbD90911)andBacillus
subtilis(283aas;spP55190;Z99104).Thebacterialhomologuesexhibit610putativeTMSs.Becausethe
familyisrepresentedinwidelydivergentorganisms,itisprobablyubiquitous.
(Q.18) Jcurvephenomenonisrelatedtowhichofthefollowing?
(a)

Hypertension

(b)

Diabetes

(c)

HIV

(d)

Alloftheabove

YourResponse:

CorrectAnswer:

Exp:

Hypertension
Jcurvephenomenon
PeoplewithhighBPand/orhighbloodcholesterollevelshaveagreaterriskofdevelopingcardiovascular
diseases(CVD).
Thehigherthebloodpressureand/orcholesterollevel,thegreatertherisk.Wealsoknowthatlowering
bloodpressureandcholesterollevelslowerstheriskforCVD.
Whenthebloodpressureorbloodcholesterollevelsoflargegroupsofpeopleareplottedonagraph
againstCVDmortality,itoftenresultsinaJshapedcurve.
Thiscurveshowsthatthosewithhigherbloodpressureand/orcholesterollevels,closertothetopofthe
curve,aremorelikelytodiefromCVD.
Thecurvealsoshowsthatthoseatthelowestendofthecurve(withverylowbloodpressureand/orlow
cholesterollevels)alsohavehigherCVDmortality.ThisaccountsfortheJshapeandisknownastheJcurve
phenomenon.(RefH18thPg2058)
ThetermJcurveisusedinseveraldifferentfieldstorefertoavarietyofunrelatedJshapeddiagramswhere
acurveinitiallyfalls,butthenrisestohigherthanthestartingpoint.
Inmedicine,the'Jcurve'referstoagraphinwhichthexaxismeasureseitheroftwotreatablesymptoms
(bloodpressureorbloodcholesterollevel)whiletheyaxismeasuresthechancethatapatientwilldevelop
cardiovasculardisease(CVD)
Itiswellknownthathighbloodpressureorhighcholesterollevelsincreaseapatient'srisk.
WhatislesswellknownisthatplotsoflargepopulationsagainstCVDmortalityoftentakestheshapeofaJ
curvewhichindicatesthatpatientswithverylowbloodpressureand/orlowcholesterollevelsarealsoat
increasedrisk

(Q.19) Carbonicanhydraseisanenzymethatoccursinplants,bacteria,andanimalsandisinvolvedintheformationofwhichchemical?
(a)

Carbondioxidefromcarbonandoxygen

(b)

Carbonicacidfromcarbondioxideandwater

(c)

Bicarbonateionfromcarbonicacid

(d)

Hydrochloricacid

YourResponse:

CorrectAnswer:

Exp:

CarbonicacidfromcarbondioxideandwaterRefGanongReviewofMedicalPhysiology23rdEdPage
816
Carbonicanhydrasecatalyzestheformationofcarbonicacidfromcarbondioxideandwater.Itisnot
involvedintheformationofcarbondioxidefromcarbonandoxygen,bicarbonateionfromcarbonicacid,
hydrochloricacid,orhypochlorousacid.

(Q.20) Parasympatheticstimulationinducessalivaryacinarcellstoreleasewhichofthefollowingprotease
(a)

Bradykinin

(b)

Kallikrein

(c)

Kininogen

(d)

Kinin

YourResponse:

CorrectAnswer:

http://dbmci.examonair.com/Result/ShowAllQuestionInHtml.aspx?testid=3572

13/99

22/03/2016

DrBhatiaMedicalCoachingInstitute:OnlineTestPlatform
Exp:

KallikreinRefGanongReviewofMedicalPhysiology23rdEdPage757
Theproductionofsalivaisstimulatedbothbythesympatheticnervoussystemandtheparasympathetic.[10]
Thesalivastimulatedbysympatheticinnervationisthicker,andsalivastimulatedparasympatheticallyis
morewatery.
Sympatheticstimulationofsalivaistofacilitaterespiration,whereasparasympatheticstimulationisto
facilitatedigestion.
Parasympatheticstimulationleadstoacetylcholine(ACh)releaseontothesalivaryacinarcells.AChbinds
tomuscarinicreceptors,specificallyM3,andcausesanincreasedintracellularcalciumionconcentration
(throughtheIP3/DAGsecondmessengersystem).Increasedcalciumcausesvesicleswithinthecellstofuse
withtheapicalcellmembraneleadingtosecretion.AChalsocausesthesalivaryglandtoreleasekallikrein,
anenzymethatconvertskininogentolysylbradykinin.Lysylbradykininactsuponsbloodvesselsand
capillariesofthesalivaryglandtogeneratevasodilationandincreasedcapillarypermeabilityrespectively.
Theresultingincreasedbloodflowtotheaciniallowsproductionofmoresaliva.Inaddition,SubstanceP
canbindtoTachykininNK1receptorsleadingtoincreasedintracellularcalciumconcentrationsand
subsequentlyincreasedsalivasecretion.Lastly,bothparasympatheticandsympatheticnervousstimulation
canleadtomyoepitheiliumcontractionwhichcausestheexpulsionofsecretionsfromthesecretoryacinus
intotheductsandeventuallytotheoralcavity.
Sympatheticstimulationresultsinthereleaseofnorepinephrine.Norepinephrinebindingtoadrenergic
receptorswillcauseanincreaseinintracellularcalciumlevelsleadingtomorefluidvs.proteinsecretion.If
norepinephrinebindsadrenergicreceptors,itwillresultinmoreproteinorenzymesecretionvs.fluid
secretion.Stimulationbynorepinephrineinitiallydecreasesbloodflowtothesalivaryglandsdueto
constrictionofbloodvesselsbutthiseffectisovertakenbyvasodilationcausedbyvariouslocalvasodilators.
Salivaproductionmayalsobepharmacologicallystimulatedbysocalledsialagogues.Itcanalsobe
suppressedbysocalledantisialagogues.

(Q.21) Thefollowinggraphrepresentseffectofsubstrateconcentrationontheinitialvelocityofanenzyme

catalyzedreaction.Wrongstatementaboutthisgraphis?

(a) Thecurveishyperbolicinshape.
(b) ?hererepresentsKmoftheenzyme.
(c) AtpointConlyasmallamountoftheenzymeispresentastheEnzymeSubstratecomplex.
(d)

AtpointC,vidependssolelyontherapiditywithwhichproductdissociatesfromtheenzymesothatitmaycombinewithmore
substrate

YourResponse:

CorrectAnswer:

Exp:

AtpointConlyasmallamountoftheenzymeispresentastheEnzymeSubstratecomplex.
REF:Harper'sIllustratedBiochemistry,28e>Chapter8.Enzymes:Kinetics
Enzymereactionsaretreatedasiftheyhadonlyasinglesubstrateandasingleproduct.Forenzymeswith
multiplesubstrates,theprinciplesdiscussedbelowapplywithequalvalidity.Moreover,byemploying
pseudofirstorderconditions(seeabove),scientistscanstudythedependenceofreactionrateuponan
individualreactantthroughtheappropriatechoiceoffixedandvariablesubstrates.Inotherwords,under
pseudofirstorderconditionsthebehaviorofamultisubstrateenzymewillimitateonehavingasingle
substrate,Inthisinstance,however,theobservedrateconstantwillbeafunctionoftherateconstantk1for
thereactionaswellastheconcentrationofthefixedsubstrate(s).
Foratypicalenzyme,assubstrateconcentrationisincreased,viincreasesuntilitreachesamaximumvalue
Vmax.Whenfurtherincreasesinsubstrateconcentrationdonotfurtherincreasevi,theenzymeissaidto
be"saturated"withsubstrate.Notethattheshapeofthecurvethatrelatesactivitytosubstrate
concentrationishyperbolic.Atanygiveninstant,onlysubstratemoleculesthatarecombinedwiththe
enzymeasanEScomplexcanbetransformedintoproduct.Second,theequilibriumconstantforthe
formationoftheenzymesubstratecomplexisnotinfinitelylarge.Therefore,evenwhenthesubstrateis
presentinexcess(pointsAandB,onlyafractionoftheenzymemaybepresentasanEScomplex.Atpoints
AorB,increasingordecreasing[S]thereforewillincreaseordecreasethenumberofEScomplexeswitha
correspondingchangeinvi.AtpointC,essentiallyalltheenzymeispresentastheEScomplex.Sincenofree
enzymeremainsavailableforformingES,furtherincreasesin[S]cannotincreasetherateofthereaction.
Underthesesaturatingconditions,vidependssolelyonandthusislimitedbytherapiditywithwhich
productdissociatesfromtheenzymesothatitmaycombinewithmoresubstrate.

(Q.22) Agatstonscoreisusedfor?
(a)

Regionalwallmotionabnormality

(b)

Myocardialscar

(c)

Quantificationofcalcifiedplaque

(d)

Toknowejectionfraction.

YourResponse:

CorrectAnswer:

http://dbmci.examonair.com/Result/ShowAllQuestionInHtml.aspx?testid=3572

14/99

22/03/2016

DrBhatiaMedicalCoachingInstitute:OnlineTestPlatform
Exp:

Quantificationofcalcifiedplaque
RecentAdvances:PathologyofPlaque.
I.Atheroscleroticplaquesbecomeprogressivelycalcifiedovertime,increaseswithage.
II.Forthisreason,methodsfordetectingcoronarycalciumhavebeendevelopedasameasureofthe
presenceofcoronaryatherosclerosis.
III.DetectionofcalciuminatheroscleroticplaquebyelectronbeanCTscan(EBCT).
IV.CoronarycalciumdetectedbytheseimagingtechniquesisquantifiedusingtheAgatstonscoremost
commonly,whichisbasedontheareaanddensityofcalcification.
TwomodesofcardiacCTareusedforCornonaryarteryCalcification(CAC)quantification.Formerly,electron
beamcomputedtomography(EBCT)andmorerecentlymultidetectorcomputedtomography(MDCT)have
beenusedforthisevaluation.Electronbeamcomputedtomographyallowedfasterimagingwithhigher
temporalresolution.However,MDCThavetheadvantageofhigherspatialresolutionandimagequality
especiallywithrecentscannergenerations,butoptimallyshouldbedonewithheartratecontroltolimit
motionartifactsfromhighheartrates.
CACisdefinedasahyperattenuatinglesion>130Hounsfieldunitswithanareaof3pixels.BaselineCAC
hasbeenquantifiedbyseveralmethods.TheAgatstonscoreiscalculatedbymultiplyingthelesionarea
(mm2)byadensityfactor(between1and4).Becauseofthestepwisenatureofthedensityfactor,changes
intheAgatstonscoremightnotaccuratelycapturechangesincoronarycalcium.IncontrasttotheAgatston
score,thecalciumvolumescore(CVS)representsanactualvolumeofCACandreducesvariabilitybetween
scansopposedtotheincreaseinAgatstonscorewhichmightjustrepresentanincreaseinplaque
attenuationratherthansizeovertime.Themassscorehasalsobeenadvocated,withlessinterscanner
variability,howeverlimitedoutcomedataisavailablewiththismeasure,soitisrarelyused.CACistypically
scannedinaprospectivelyECGtriggeredmodewith2.53.0mmthickaxialimages.Theradiationdoseis
low,withatypicaleffectivedoseof<1.5mSv.
CACscorehadbeenvalidatedinanoriginalhistopathologicalstudybyRumbergeretal.whichshowedahigh
correlationbetweenCACareabyEBCTandplaqueareain13autopsyhearts.EBCTCACScorewasalso
directlycomparedtocoronaryangiographyin213consecutivepatientsandacutpointAgatstonscoreof
371wasfoundtobeassociatedwithcoronaryarterialluminalstenosis>70%inatleastonecoronaryartery
withhighsensitivityandspecificity.Specificpatternsofcalcifications,namely;shelllikeanddiffusewere
showntobemoreassociatedwithseverestenosisratherthananodularpattern

(Q.23) Stabilityofanklejointmaintainedbyallexcept
(a)

Collateralligaments

(b)

Cruciateligaments

(c)

Tendonsofmusclescrossingthejoint

(d)

Closeappositionofarticularsurfacesofbones

YourResponse: b
CorrectAnswer: B
Exp:

Cruciateligaments
Ref1.Pg.150;BDChaurasia(4thEd.);Vol2,2.Pg.1443;Gray's
Anatomy(40thEd.)
Cruciateligamentsarepresentinthekneeandnottheanklejoint.
Anklejointisstabilizedbythefollowingfactors:
Closeappositionofarticularsurfacesofbones
Tendonsofmusclescrossingthejoint
Ligamentslikecollateralligamentsanddistaltibiofibularligaments
Gravityandgroundreactionforcesprovidedynamicstability.

(Q.24) Regardinggenitaldevelopment,trueis
(a)

Ychromosomeisassociatedwithovarydevelopment

(b)

Genitalridgestartsdevelopingat5thweek

(c)

Malegenitalsdevelopearlierthanfemalegenitals

(d)

ExternalGenitaldevelopmentiscompleteby10thweek

YourResponse:

CorrectAnswer:

Exp:

Genitalridgestartsdevelopingat5thweek
References:1.Pg.263,271;Thedevelopinghuman(KeithLMoore);8thEd.
ThekeytosexualdimorphismistheYchromosome,whichcontaintheTDF(TestesDeterminingFactor)
gene,calledastheSRYgeneonitsshortarm.
UndertheinfluenceofTDF,maledevelopmentoccurs;initsabsence,femaledevelopmentisestablished.
Hence,Ychromosomeregulatesmalesexualdevelopment.
Genitalridgesstartdevelopingatthe5thweek
Thegonadsdonotacquiremaleorfemalemorphologicalcharacteristicsuntilthe7thweekofdevelopment.
So,attheendof6thweekitisimpossibletodistinguishbetweenthetwosexesbylookingattheexternal
genitalia.
Distinguishingsexualcharacteristicsbegintoappearduringthe7thweekbuttheexternalgenitaliaarenot
fullydifferentiateduntilthe12thweek

http://dbmci.examonair.com/Result/ShowAllQuestionInHtml.aspx?testid=3572

15/99

22/03/2016

DrBhatiaMedicalCoachingInstitute:OnlineTestPlatform
(Q.25) Pelvicsplanchnicnervessupplyallexcept
(a)

Vermiformappendix

(b)

Urinarybladder

(c)

Uterus

(d)

Rectum

YourResponse:

CorrectAnswer:

Exp:

Vermiformappendix
References:1.Pg.1045,1143;Gray'sAnatomy(40thEd.),2.Pg.258;BDChaurasia(4thEd.);Vol2
Pelvicsplanchnicnervesaretheparasympatheticnervestosupplythepelvicvisceralikeurinarybladder,
uterusandrectum.
AppendixisapartofMidgutandissuppliedbytheVagusnerve.
Parasympatheticnervoussystem
Functions:
Evacuationofbodytubesby
Increaseinperistalsis
Relaxationofsphincters
Erectionofgenitalia
Parasympatheticnervesupply:
Tillmidgut:Vagus
Hindgut:Pelvicsplanchnicnerves
Pelvicsplanchnicnerves(alsocalledasNervierigentes)arebranchesofsacralplexuscarrying
parasympatheticfibres.
Theyalsoprovidevasodilatorfibreserectiletissueofpenis&clitoriscausingerection.
Conusmedullarysyndrome:Pelvicsplanchnicnervesarelesionedleadingtourinaryretention,constipation
andimpotence(erectiledysfunction).
Sympatheticnervoussystem
Functions:
Retentionofthebodytubescontentsby
Decreaseinperistalsis
Constrictionofsphincters
Ejaculation
Sympatheticsupply:
Foregut:Greatersplanchnicnerve(T:59)
Midgut:Lessersplanchnicnerves(T10,11)
Kidney:Leastsplanchnicnerves(T12)
Hindgut:Lumbarsplanchnicnerves(L1,2)
Visceralpainisusuallycarriedbythesympatheticfibersandpainofacuteappendicitis(Midgut;T10)is
carriedbythelessersplanchnicnervesandreferredtothecorrespondingdermatome(Periumbilicalregion:
T10).

(Q.26) Urogenitaldiaphragmformedbyallexcept
(a)

Collesfascia

(b)

Deeptransverseperinei

(c)

Perinealmembrane

(d)

Sphincterurethrae

YourResponse:

CorrectAnswer:

Exp:

Collesfascia
Urogenitaldiaphragmcontainsthedeepperinealpouchandislinedinferiorlybytheperinealmembrane
(d)andnotCollesfascia(a).
Collesfasciaactuallyistheinferiorliningofsuperficialperinealpouch.
Urogenitaldiaphragmisatriangularmusculofascialdiaphragmintheanteriorpartofperineum.Itis
contributedbymainlytwomuscles:sphincterurethrae(d)anddeeptransverseperinei(b).
Thesemusclesareenclosedwithinasuperiorandinferiorfascia.Theinferiorfasciaisalsocalledasperineal
membrane.
Perinealbodyisafibromuscularbodyattachedattheposteriorborderofperinealmembraneinthe
midline.Boththefasciaofurogenitaldiaphragmareattachedtotheperinealbody.
Perinealbodyisagoodsupportofpelvicvisceraandisattachedbymanymusclesoftheperineumincluding
themusclesofurogenitaldiaphragmdeeptransverseperinei&sphincterurethrae.
References:
1.Pg.361;BDChaurasia(4thEd.);Vol2
2.Pg.10946;Gray'sAnatomy(40thEd.)
3.Pg.429;SnellsClinicalAnatomy(7thEd.)

http://dbmci.examonair.com/Result/ShowAllQuestionInHtml.aspx?testid=3572

16/99

22/03/2016

DrBhatiaMedicalCoachingInstitute:OnlineTestPlatform
(Q.27) Hemivertebraisadefectof
(a)

Body

(b)

Transverseprocess

(c)

Spine

(d)

Posteriorvertebralarches

YourResponse:

CorrectAnswer:

Exp:

Body
Vertebralbodiesmainlydevelopfromventralsclerotome(VS)andthushemivertebraisadefectofVS.
Hemivertebracanleadtodefectsinthevertebralcolumnscoliosis.
Posteriorvertebralarches(PVA)contributemainlytothelamina&spines.PVAarecontributedbythedorsal
sclerotome.
Ahemivertebraistypeofvertebralanomalyandresultsfromalackofformationofonehalfofavertebral
body.Itcanbeacommoncauseofacongenitalscoliosis/kyphosisoralordosis.Theestimatedincidenceisat
~0.3per1000livebirths.
Itfallsunderthespectrumofsegmentationalanomaliesandcaninvolveoneormultiplelevels.A
hemivertebraactsasawedgewithinthevertebralcolumn,resultingincurvatureawayfromthesideon
whichitispresent.
Recognisedassociationsaremanyandinclude
Aicardisyndrome
cleidocranialdysostosis
gastroschisis3
Gorlinsyndrome
fetalpyelectasis3
JarchoLevinsyndrome
OEIScomplex
VACTERLassociation
Classification
Ahemivertebracanbeclassifiedaccordingtomanytypeswhichinclude
Attachment
fullysegmental(free):
notattachedtoeithervertebraaboveorbelow
mostconcerning
semisegmental:halfsegmentisfusedwithvertebraaboveorbelowwithnointerveningintervertebraldisc
nonsegemental:
notseperatedfrom(i.econnectedto)eitherlevelaboveorbelow
causeslessofaconcern
incarcerated:
joinedbypediclestolevelsaboveandbelow
causeslessofaconcern
Orientation
dorsalhemivertebra:classicallyresultsinakyphosis
lateralhemivertebra:classicallyresultsinascoliosis
ventralhemivertebra(extremelyrare):resultsinthelordosis
Radiographicfeatures
Antenatalultrasound
Ahemivertebramaybeseenasanasymmetricalvertebralbodyonsagittalorcoronalscanning,whileon
axialscanning,afocaldefectmaybeseenateithersideofthevertebralcolumn5.
Plainfilm/CT
Usuallydirectlyoutlinesthebonyanomalyandoftenseenasawedgeshapedvertebralbody.
Theprognosiscanbevariabledependantonthetypefrombeingaprogressivetononprogressive
deformity.Thestrongestnegativeimpactiswhenahemivertebraoccursatthelumbosacrallevel7.
Differentialdiagnosis:Forantenatalsonographicappearancesconsiderblockvertebra

(Q.28) Whichisanonessentialmineral
(a)

Copper

(b)

Manganese

(c)

Iron

(d)

Lead

YourResponse:

CorrectAnswer:

Exp:

Lead
(RefGanongReviewofMedicalPhysiology23rdEd,Chapter27.Digestion,Absorption,&Nutritional
Principles)

http://dbmci.examonair.com/Result/ShowAllQuestionInHtml.aspx?testid=3572

17/99

22/03/2016

DrBhatiaMedicalCoachingInstitute:OnlineTestPlatform
Table273TraceElementsBelievedEssentialforLife.
ArsenicManganese
ChromiumMolybdenum
CobaltNickel
CopperSelenium
FluorineSilicon
IodineVanadium
IronZinc
Manganeseisanessentialtracenutrientinallformsoflife.
Theclassesofenzymesthathavemanganesecofactorsareverybroadandincludeoxidoreductases,
transferases,hydrolases,lyases,isomerases,ligases,lectins,andintegrins.
Thereversetranscriptasesofmanyretroviruses(thoughnotlentivirusessuchasHIV)containmanganese.
Thebestknownmanganesecontainingpolypeptidesmaybearginase,thediphtheriatoxin,andMn
containingsuperoxidedismutase(MnSOD).

(Q.29) Nitricoxideisreleasedfrom:
(a)

Pericytes

(b)

Smoothmusclecells

(c)

Endothelialcells

(d)

Mesenchymalcells

YourResponse:

CorrectAnswer:

Exp:

Endothelialcells
REFRobbins8thEdChap2.
NitricOxide
NOisashortlived,soluble,freeradicalgasproducedbymanycelltypesandcapableof
mediatingavarietyoffunctions.Inthecentralnervoussystemitregulatesneurotransmitter
releaseaswellasbloodflow.Macrophagesuseitasacytotoxicmetaboliteforkilling
microbesandtumorcells.Whenproducedbyendothelialcells(whereitwasoriginallynamed
endotheliumderivedrelaxationfactor),itcausessmoothmusclerelaxationandvasodilation.
NOissynthesizeddenovofromlarginine,molecularoxygen,andNADPHbytheenzyme
nitricoxidesynthase(NOS).TherearethreeisoformsofNOS,withdifferenttissue
distributions.TypeI(nNOS)isaconstitutivelyexpressedneuronalNOS,whichdoesnotplay
asignificantroleininflammation.TypeII(iNOS)isaninducibleenzymepresentin
macrophagesandendothelialcells;itisinducedbyanumberofinflammatorycytokinesand
mediators,mostnotablybyIL1,TNF,andIFN,andbybacterialendotoxin,andis
responsibleforproductionofNOininflammatoryreactions.iNOSisalsopresentinmany
othercelltypes,includinghepatocytes,cardiacmyocytes,andrespiratoryepithelium.Type
III(eNOS)isaconstitutivelysynthesizedNOSfoundprimarily(butnotexclusively)within
endothelium.
NOplaysmanyrolesininflammation,including(1)relaxationofvascularsmoothmuscle
(vasodilation),(2)antagonismofallstagesofplateletactivation(adhesion,aggregation,and
degranulation),(3)reductionofleukocyterecruitmentatinflammatorysites,and(4)action
asamicrobicidal(cytotoxic)agent(withorwithoutsuperoxideradicals)inactivated
macrophages.

(Q.30) ChediakHigashisyndromeisassociatedwithdefectin:
(a)

Chemotaxis

(b)

Phagocytosis

(c)

Absorption

(d)

BothAandB

YourResponse:

CorrectAnswer:

Exp:

BothAandB
Thebestanswershouldbeboth.
TheunderlyingdefectinChdiakHigashisyndromeremainselusive,butthedisordercanbeconsidereda
modelfordefectsinvesicleformation,fusion,ortrafficking.

Robins8theditionpage5556
http://dbmci.examonair.com/Result/ShowAllQuestionInHtml.aspx?testid=3572

18/99

22/03/2016

DrBhatiaMedicalCoachingInstitute:OnlineTestPlatform
Chediakhigashisyndromeisautosomalrecessiveconditioncharacterizedbydefectivephagolysosome
function
defectivefusionofphagosomesandlysosomesinphagocytes(causingsusceptibilitytoinfections)
abnormalitiesinmelanocytes(leadingtoalbinism)
abnormalitiesincellsofnervoussystem(nervedefects)
defectiveplateletsfunction
neutropeniaanddefectivedegranulationdelayedmicrobialkilling
leukocytecontaingiantgranules(duetoabberantphagolysosomefusion)
(Q.31) Whichsugaristakenupbyenterocytesbyfacilitateddiffusion?
(a)

Glucose

(b)

Galactose

(c)

Fructose

(d)

Xylose

YourResponse:

CorrectAnswer:

Exp:

RefGanongReviewofMedicalPhysiology23rdEdPage608
Fructoseistakenupbyenterocytesbyfacilitateddiffusion.Bothglucoseandgalactosearetakenupby
enterocytesthroughasodiumdependenttransporter(secondaryactivetransport).Xyloseandsucroseare
nottakenupbyenterocytes.

(Q.32) Clottingfactorpresentinbothplasmaandserumis
(a)

Thrombin

(b)

FactorVIII

(c)

FactorV

(d)

FactorVII

YourResponse:

CorrectAnswer:

Exp:

FactorVII
Fluidportionofblood(Bloodminuscells)isknownasplasmaandifthewholebloodisallowedtoclotand
theclotisremoved,theremainingfluidiscalledserum.
SerumhasessentiallythesamecompositionasplasmaexceptthatfibrinogenandclottingfactorII,Vand
VIIIhavebeenremoved.
Italsohasahighserotonincontentduetobreakdownofplateletsduringclotting.
TiptoremembervitKdependentfactorsdonotgetconsumedduringclottingexceptfactorII.

(Q.33) ThefollowingaremostimportantFreeRadicalScavengersexcept:
(a)

Superoxidedismutase

(b)

Catalase

(c)

Glutathione

(d)

Glutamine

YourResponse:

CorrectAnswer:

Exp:

Glutamine.(RefRobbins8th/pg.20,21;Table13;Fig.120;HarperBiochemistry28th/pg.483)
RemovalofFreeRadicals:
Freeradicalsareinherentlyunstableandgenerallydecayspontaneously.,forexample,isunstableand
decays(dismutates)spontaneouslyintoO2andH2O2inthepresenceofwater.
Inaddition,cellshavedevelopedmultiplenonenzymaticandenzymaticmechanismstoremovefreeradicals
andtherebyminimizeinjury.
Theseincludethefollowing:
Antioxidantseitherblocktheinitiationoffreeradicalformationorinactivate(e.g.,scavenge)freeradicals.
Examples:
LipidsolublevitaminsEandAaswellasVitaminCandglutathioneinthecytosol.
IronandcoppercancatalyzetheformationofROS.Thelevelsofthesereactivemetalsareminimizedby
bindingoftheionstostorageandtransportproteins(e.g.,transferrin,ferritin,lactoferrin,and
ceruloplasmin),therebyminimizingtheformationofROS.
AseriesofenzymesactsasfreeradicalscavengingsystemsandbreaksdownH2O2includethefollowing:
Superoxidedismutases(SODs)OptionAarefoundinmanycelltypesandconvertoxygenfree
radicaltoH2O2.
oCatalase,OptionBpresentinperoxisomes,decomposesH2O2(2H2O2O2+2H2O).
GlutathioneperoxidaseOptionCalsoprotectsagainstinjurybycatalyzingfreeradicalbreakdown
(H2O2+2GSHGSSG[glutathionehomodimer]+2H2O,or2OH+2GSHGSSG+2H2O).Theintracellular
ratioofoxidizedglutathione(GSSG)toreducedglutathione(GSH)isareflectionoftheoxidativestateofthe
cellandisanimportantindicatorofthecell'sabilitytodetoxifyROS.
SummaryRemovaloffreeradicals:
Themajorantioxidantenzymes(freeradicalscavengingsystems)are:

http://dbmci.examonair.com/Result/ShowAllQuestionInHtml.aspx?testid=3572

19/99

22/03/2016

DrBhatiaMedicalCoachingInstitute:OnlineTestPlatform
Superoxidedismutase(SOD):
oInmitochondriaoConvertsO2(superoxide)H2O2.
Catalase
oInperoxisomesoConvertsH2O2H2O+O2.
GlutathionePeroxidase
oInmitochondria.oConvertsOH(hydroxylradical)H2O2H2O+O2.
Peroxiredoxins
oCauseconversiontoHNO2.oIncytosolandmitochondria.
(Q.34) Proofreadingistheroleof?
(a)

DNAprimase

(b)

DNAPolymerase

(c)

ExonucleaseI

(d)

Restrictionendonuclease

YourResponse:

CorrectAnswer:

Exp:

DNAPolymerase.(RefHarperBiochemistry26th/pg.328,311)
PolymeraseII(polII)ismostlyinvolvedinproofreadingandDNArepair.PolymeraseI(polI)completes
chainsynthesisbetweenOkazakifragmentsonthelaggingstrand.Eukaryoticcellshavecounterpartsfor
eachoftheseenzymesplussomeadditionalones.
PROTEINFUNCTION
1DNApolymerasesDeoxynucleotidepolymerization
2HelicasesProgressiveunwindingofDNA
3TopoisomerasesRelievetorsionalstrainthatresultsfromhelicaseinducedunwinding
4DNAprimaseInitiatessynthesisofRNAprimers
5SinglestrandbindingproteinsPreventprematurereannealingofdsDNA
6DNAligaseSealsthesinglestrandnickbetweenthenascentchainandOkazakifragmentson
laggingstrand

(Q.35) ThetypeofmutationthatleadstoreplacementofValineforglutamateinSickleCellDiseaseis?
(a)

Pointmutation

(b)

Silentmutation

(c)

Nonsensemutation

(d)

None

YourResponse:

CorrectAnswer:

Exp:

Pointmutation.
Pointmutations
Pointmutationsresultfromthesubstitutionofasinglenucleotidebasebyadifferentbase,resultinginthe
replacementofoneaminoacidbyanotherintheproteinproduct.
(a).Missensemutations:
Themutationgivingrisetosicklecellanemiaisanexcellentexampleofapointmutationthataltersthe
meaningofthegeneticcode.
Suchmutationsaresometimescalledmissensemutations.
(b).Nonsense"mutations:
Incontrast,certainpointmutationsmaychangeanaminoacidcodontoachainterminationcodon,orstop
codon.
Such"nonsense"mutationsinterrupttranslation,andtheresultanttruncatedproteinsarerapidlydegraded.
Frameshiftmutations
Frameshiftmutationsoccurwhentheinsertionordeletionofoneortwobasepairsaltersthereadingframe
oftheDNAstrand.
Trinucleotiderepeatmutations
Trinucleotiderepeatmutationsbelongtoaspecialcategory,becausethesemutationsarecharacterizedby
amplificationofasequenceof3nucleotides.
Althoughthespecificnucleotidesequencethatundergoesamplificationdiffersinvariousdisorders,all
affectedsequencessharethenucleotidesguanine(G)andcytosine(C).
Forexample,infragileXsyndrome,prototypicalofthiscategoryofdisorders,thereare200to4000tandem
repeatsofthesequenceCGGwithinagenecalledFMR1.Innormalpopulations,thenumberofrepeatsis
small,averaging29.TheexpansionsofthetrinucleotidesequencespreventnormalexpressionoftheFMR1
gene,thusgivingrisetomentalretardation.
Anotherdistinguishingfeatureoftrinucleotiderepeatmutationsisthattheyaredynamic(i.e.,thedegreeof
amplificationincreasesduringgametogenesis).

(Q.36) ThefourPyroleringsinhemoglobinmoleculearejoinedtogetherby?
(a)

Disulphidebridges

(b)

Methylenebridges

http://dbmci.examonair.com/Result/ShowAllQuestionInHtml.aspx?testid=3572

20/99

22/03/2016

DrBhatiaMedicalCoachingInstitute:OnlineTestPlatform
(c)

Hydrogenbonds

(d)

Alphabonds

YourResponse:

CorrectAnswer:

Exp:

Methylenebridges(RefHarpersBiochemistry26th/pg.270278)
StructureofHb
Hb=Heme+globulin(2+2chains)Q
TheironisinFe++stateinthehemeoffunctionalHb.
IroncanformsixCoOrdinatedBonds.
Ironisheldatthecentreofthehemeby4nitrogenofporphyrinrings(pyrrolerings).
Ironbindinternally(by5thlinkage)withaminoacidsofHistidine.Q
ThecoordinatedpositionofFe++isaavailabletobindtoO2.
ThefourPyroleringsarejoinedbyMethyleneorMethylidenebridge(C=bridge).
HbinternallycontainsmostlyHydrophobicaminoacids(formingHemepocket)andHydrophilicamino
acidsontheirsurfaces.
TheHemepocketofsubunitareadequateforentryofO2whilesubunitsHemepocketsblocksentry
ofO2byavalineresidues.

(Q.37) Followingarethephysiologicalactionsofinsulinexcept:
(a)

Increasedlipogenesis

(b)

Increasedaminoacidentryinthecell

(c)

ActivationofLipoproteinlipase

(d)

Activationofkeyenzymesofglycolysis

YourResponse:

CorrectAnswer:

Exp:

ActivationofLipoproteinlipase.(RefHarpersIllustratedBiochemistry26
editionTable191)
PATHWAYEFFECTOFINSULIN
1Glycogensynthesis+
2Glycolysis(liver)+
3Triacylglyccridesynthesis+
4Fattyacidsynthesis+
5Proteinsynthesis+
6Cholesterolsynthesis+
7Glycogenolysis+
8Gluconeogenesis(liver)
9Lipolysis
10Fattyacidoxidation
11Proteinbreakdown+/
12Ketogenesis(liver)
13Ketonebodyutilization(nonhepatictissues)
Themajoractionsofinsulinare:
1.Facilitationofglucosetransportthroughcertainmembranes(e.g.adiposeand
musclecells)
2.Stimulationoftheenzymesystemforconversionofglucosetoglycogen(liver
andmusclecells);
3.Slowdownofgluconeogenesis(liverandmusclecells);
4.Regulationoflipogenesis(liverandadiposecells);and
5.Promotionofproteinsynthesisandgrowth(generaleffect).
Noteinsulinactivateslipoproteinlipaseinadiposetissueanddecreaseslipoproteinlipase
activityinmuscles

(Q.38) Activityofallofthefollowingenzymesisincreasedindiabetesstateexcept?
(a)

Pyruvatecarboxylase

http://dbmci.examonair.com/Result/ShowAllQuestionInHtml.aspx?testid=3572

21/99

22/03/2016

DrBhatiaMedicalCoachingInstitute:OnlineTestPlatform
(b)

PEPCarboxykinase

(c)

Pyruvatedehydrogenase

(d)

Glucose6phosphatase.

YourResponse:

CorrectAnswer:

Exp:

Pyruvatedehydrogenase.
RegulatoryandAdaptiveEnzymesAssociatedwithCarbohydrateMetabolism
Activityin
Carbo
hydrate
Feeding

Fasting Inducer
and
Diabetes

Repressor Activator

Inhibitor

Glycogenolysis,glycolysis,andpyruvateoxidation
Glycogensynthase

Increases Decreases

Hexokinase

Noeffecteither

Glucokinase

Increases Decreases Insulin

Phosphofructokinase Increases Decreases Insulin


1

Insulin,
glucose6
phosphate

Glucagon

Glucose6
phosphate
Glucagon
Glucagon 5'AMP,
Citrate,ATP,
fructose6
glucagon
phosphate,
fructose2,6
bisphosphate,
Pi

Pyruvatekinase

Increases Decreases Insulin,fructose Glucagon Fructose1,6 ATP,alanine,


bisphosphate, glucagon,
insulin
norepinephrine

Pyruvate
dehydrogenase

Increases Decreases

CoA,NAD+, AcetylCoA,
insulin,ADP, NADH,ATP
(fattyacids,
pyruvate
ketonebodies)

Gluconeogenesis
Pyruvatecarboxylase Decreases Increases Glucocorticoids, Insulin
glucagon,
epinephrine

AcetylCoA

Phosphoenolpyruvate Decreases Increases Glucocorticoids, Insulin


carboxykinase
glucagon,
epinephrine

Glucagon?

Glucose6
phosphatase

ADP

Decreases Increases Glucocorticoids, Insulin


glucagon,
epinephrine

(Q.39) Intermsofenzymekineticsthefollowingplotisof1/vi(vi=Velocityofenzymereaction)asyasafunctionof1/[S](S=Substrate

concentration)asx.Thisplotiscorrectlynamedas?

(a)

Doublereciprocalplot

(b)

Hillequationplot

(c)

LineweaverBurkplot

(d)

BothAandC.

YourResponse:

CorrectAnswer:

Exp:

REFHarper'sIllustratedBiochemistry,28eChapter8.Enzymes:Kinetics
Equation(35)istheequationforastraightline,y=ax+b,wherey=1/viandx=1/[S].Aplotof1/viasyasa
functionof1/[S]asxthereforegivesastraightlinewhoseyinterceptis1/VmaxandwhoseslopeisKm
/Vmax.SuchaplotiscalledadoublereciprocalorLineweaverBurkplot.Settingtheytermofequation
(36)equaltozeroandsolvingforxrevealsthatthexinterceptis1/Km.
ThegreatestvirtueoftheLineweaverBurkplotresidesinthefacilitywithwhichitcanbeusedtodetermine
thekineticmechanismsofenzymeinhibitors(seebelow).However,inusingadoublereciprocalplotto

http://dbmci.examonair.com/Result/ShowAllQuestionInHtml.aspx?testid=3572

22/99

22/03/2016

DrBhatiaMedicalCoachingInstitute:OnlineTestPlatform
determinekineticconstants,itisimportanttoavoidtheintroductionofbiasthroughtheclusteringofdata
atlowvaluesof1/[S].Toachievethis,prepareasolutionofsubstratewhosedilutionintoanassaywill
producethemaximumdesiredconcentrationofsubstrate.Nowusethesamevolumeofsolutionsprepared
bydilutingthestocksolutionbyfactorsof1:2,1:3,1:4,1:5,etc.Thedatawillthenfallonthe1/[S]axisat
intervalsof1,2,3,4,5,etc.Alternatively,asinglereciprocalplotsuchastheEadieHofstee(viversusvi/[S])
orHanesWoolf([S]/viversus[S])plotcanbeusedtominimizeclustering.
(Q.40) EnzymedeficientinAlkaptonuria?
(a)

Phenylalaninehydroxylase

(b)

Cystathioninesynthase

(c)

Homogentisicacidoxidase

(d)

Tyrosinase

YourResponse:

CorrectAnswer:

Exp:

Homogentisicacidoxidase.(RefHarpersIllustratedBiochemistry26edition/pg.255)
TheprobablemetabolicdefectintypeItyrosinemia(tyrosinosis)isatfumarylacetoacetatehydrolase.
Therapyemploysadietlowintyrosineandphenylalanine.Untreatedacuteandchronictyrosinosisleadsto
deathfromliverfailure.AlternatemetabolitesoftyrosinearealsoexcretedintypeIItyrosinemia(Richner
Hanhartsyndrome),adefectintyrosineaminotransferase,andinneonataltyrosinemia,duetoloweredp
hydroxyphenylpyruvatehydroxylaseactivity.Therapyemploysadietlowinprotein.
Alkaptonuriawasfirstdescribedinthe16thcentury.Characterizedin1859,itprovidedthebasisfor
Garrodsclassicideasconcerningheritablemetabolicdisorders.Thedefectislackofhomogentisateoxidase.
Theurinedarkensonexposuretoairduetooxidationofexcretedhomogentisate.Lateinthedisease,there
isarthritisandconnectivetissuepigmentation(ochronosis)duetooxidationofhomogentisateto
benzoquinoneacetate,whichpolymerizesandbindstoconnectivetissue.
Alkaptonuria(ochronosis)
Congenitaldeficiencyofhomogentisicacidoxidaseinthedegradativepathwayoftyrosine.
Resultingalkaptonbodiescauseurinetoturnblackonstanding.
Also,theconnectivetissueisdark.
Benigndisease,butmayhavedebilitatingarthralgias.
ALLENZYMEDEFICIENCYDISEASES:
DISEASE

DEFICIENTENZYME

VonGierkesDiseaseIa

Glucose6phosphatase

VonGierkesDiseaseIb

Glucose6phosphatasetranslocase

PompesDisease

Acidmaltase/Lysosomalacidalphaglucosidase

CorisDisease/Forbes/Limitdextrinosis

Liverandmuscledebrancherenzyme

AndersonsDisease/Amylopectinosis

Branchingenzyme

McArdlesDisease

Musclephosphorylase

HersDisease

Hepaticphosphorylase

TaruiDisease

Phosphofructokinase

Fanconi/BickelsDisease

GlucosetransporterII

Albinism

Tyrosinase

NeimanPicksDisease

Acidsphingomyelinase

Metachromaticleukodystrophy

Arylsulphatase

GauchersDisease

Acidbetaglucosidase

Oroticaciduria

Oroticphosphoribosyltransferase

Galactosemia

Galactose1phosphateuridyltransferase
Galactokinase
UDPgalactose4epimerase

Fructoseintolerance

Fructose1,6biphosphatase

Essentialfructosuria

Fructokinase

KrabbesDisease

Betagalactosidase

FabrysDisease

Alphagalactosidase

FarbersDisease

Acidceramidase

ForbesDisease

Debranchingenzyme

MPSI

AlphaLIduronidase

MPSII

Iduronatesulphatase

TaysachsDisease

BetaHexosaminidaseA

Sandhoff;sDisease

BetaHexosaminidaseAandB

TyrosinemiaI

Fumarylacetoacetatehydrolase

TyrosinemiaII

Transaminase

MSUD

Branchedchainalphaketoaciddecarboxylase

HomocystinutiaI

Cystathioninesynthetase

HomocystinuriaII

N5MethylTHFmethyltransferase

http://dbmci.examonair.com/Result/ShowAllQuestionInHtml.aspx?testid=3572

23/99

22/03/2016

DrBhatiaMedicalCoachingInstitute:OnlineTestPlatform
Sialidosis

Neuraminidase

GM1Gangliosidosis

Betagalactosidase

Essentialpentosuria

NADPDependantLXylitoldehydrogenase

RefsumsDisease

Phytanatealphaoxidase

cystinosis

Cysteinereductase

LeschNyhanSyndrome

HGPRTaseCompletedeficiency

KelleySeegmillersyndrome

HGPRTasePartialdeficiency

Acuteintermittentporphyria

UroporphyrinogenIsynthetase

Congenitalerythropoeticporphyria

UroporphyrinogenIIIsynthetase

Porphyriacutaneatarda

Uroporphyrinogendecarboxylase

Variegateporphyria

Protoporphyrinogenoxidase

Erythropoeticprotoporphyria

Ferrochelatase

HyperammonemiaI

CarbamoylphosphatasesynthetaseI

HyperammonemiaII

Transcarbamoylase

Citrullemia

Arginosuccinatesynthetase

Arginosuccinicaciduria

Arginosuccinase

Arginemia

Arginase

Fucosidosis

Alphafucosidase

CanavansDisease

NAspertoAcylase

Adrenoleucodystrophy

AcylCoAsynthetase

(Q.41) FalseregardingGroupItypeofhormonesis?
(a)

Theyarelipophilic

(b)

Haveaverylongplasmahalflife

(c)

Theirreceptorsaremainlylocatedontheplasmamembrane

(d)

SteroidhormonesareexamplesofGroupItype.

YourResponse:

CorrectAnswer:

Exp:

Theirreceptorsaremainlylocatedontheplasmamembrane
REF:Harper'sIllustratedBiochemistry,28eChapter41
GeneralFeaturesofHormoneClasses
GroupI

GroupII

Types

Steroids,iodothyronines,
calcitriol,retinoids

Polypeptides,proteins,glycoproteins,catecholamines

Solubility

Lipophilic

Hydrophilic

Transport
proteins

Yes

No

Plasmahalflife Long(hourstodays)

Short(minutes)

Receptor

Intracellular

Plasmamembrane

Mediator

Receptorhormonecomplex

cAMP,cGMP,Ca2+,metabolitesofcomplex
phosphinositols,kinasecascades

ThehormonesingroupIarelipophilic.Aftersecretion,thesehormonesassociatewithplasmatransportor
carrierproteins,aprocessthatcircumventstheproblemofsolubilitywhileprolongingtheplasmahalflifeof
thehormone.Therelativepercentagesofboundandfreehormonearedeterminedbytheamount,binding
affinityandbindingcapacityofthetransportprotein.Thefreehormone,whichisthebiologicallyactive
form,readilytraversesthelipophilicplasmamembraneofallcellsandencountersreceptorsineitherthe
cytosolornucleusoftargetcells.Theligandreceptorcomplexisassumedtobetheintracellularmessenger
inthisgroup.
Thesecondmajorgroupconsistsofwatersolublehormonesthatbindtotheplasmamembraneofthe
targetcell.Hormonesthatbindtothesurfacesofcellscommunicatewithintracellularmetabolicprocesses
throughintermediarymoleculescalledsecondmessengers(thehormoneitselfisthefirstmessenger),
whicharegeneratedasaconsequenceoftheligandreceptorinteraction.Thesecondmessengerconcept
arosefromanobservationthatepinephrinebindstotheplasmamembraneofcertaincellsandincreases
intracellularcAMP.ThiswasfollowedbyaseriesofexperimentsinwhichcAMPwasfoundtomediatethe
effectsofmanyhormones.Atrialnatriureticfactor(ANF),usescGMPasitssecondmessenger(groupII.B).
Severalhormones,manyofwhichwerepreviouslythoughttoaffectcAMP,appeartouseioniccalcium
(Ca2+)ormetabolitesofcomplexphosphoinositides(orboth)astheintracellularsignal.Theseareshownin
groupII.Cofthetable.TheintracellularmessengerforgroupII.Disaproteinkinasephosphatasecascade.
Severalofthesehavebeenidentified,andagivenhormonemayusemorethanonekinasecascade.Afew
hormonesfitintomorethanonecategory,andassignmentschangeasnewinformationisbroughtforward.
(Q.42) Vitamindeficiencyassociatedwithincreasexanthurenicacidexcretioninurine?
(a)

Thiamine

(b)

Pyridoxine

http://dbmci.examonair.com/Result/ShowAllQuestionInHtml.aspx?testid=3572

24/99

22/03/2016

DrBhatiaMedicalCoachingInstitute:OnlineTestPlatform
(c)

Ascorbicacid

(d)

Niacin

YourResponse:

CorrectAnswer:

Exp:

Pyridoxine(RefHarper,Biochemistry,26thed.,258)
Conversionoftryptophanmetabolite3hydroxykinurenineto2hydroxyantranilateisimpairedinvitamin
B6deficiency;hencelargeamountofxanthinurateisformed.
ThisisbasicprinciplebehindconductingtryptophanloadtestwhichmainlyprecipitatesearlyvitaminB6
deficiency.
PYRIDIOXINEDEFICIENCY
Deficiency
CONVULSIONS,
Hyperirritability(deficiencyinduciblebyINHandoralcontraceptives),
Peripheralneuropathy.
Function
A.Convertedtopyridoxalphosphate,
B.Acofactorusedin:
1.Transamination(e.g.,ALTandAST),
2.Decarboxylationreactions,
3.Glycogenphosphorylase,and
4.Hemesynthesis.
C.Requiredforthesynthesisofniacinfromtryptophan.
Vitamin

Enzymesinwhichitactsascoenzyme

Deficiencydisease

1.Vit.B1(Thiamine)

Pyruvatedehydrogenasecomplex

BeriBeridisease

Oxidativedecarboxylationof

Wernickeencephalopathy

ketoacid
Transketolasereaction
Tryptophanpyrrolase
2.Vit.B2(Riboflavin)(heat
stablebutlightsensitive)

FMNandFADare2coenzymesofthisvit. Cheilosis
ConstituentofcytochromeC

Glossitis

reductase

Seborrheicdermatitis

Warburgyellowenzyme
Fumaratedehydrogenase
3.Vit.B3(Niacin)

NADandNADPareits

PellagraC/B3Ds

activeformswhich

Diarrhea

operateashydrogenand

Dermatitis

electrontransferagents

Dementia

Theyarecoenzymesforvarioustypesof
dehydrogenaseslike
Alcoholdehydrogenase
Lactatedehydrogenase
Malatedehydrogenase
Pyruvatedehydrogenase
4.Vit.B6(Pyridoxine)

Cotransaminase

Epileptiformconvulsionin
infants

Codecarboxylase
Deaminase

Sideroblasticanemia[Vit
B6isusedint/tofoxalate

InvolvedinsynthesisofCoA

stoneofkidneyand
homocystinuria]

CoenzymeforKynureninase
[vitB6deficiencyKynureninelevels,
whichisconvertedtoxanthurenicacid

http://dbmci.examonair.com/Result/ShowAllQuestionInHtml.aspx?testid=3572

25/99

22/03/2016

DrBhatiaMedicalCoachingInstitute:OnlineTestPlatform
whichisareliablecriteriaforB6deficiency]
5.Pantothenicacid

ActiveformisCoA

G.I.symptoms

FormationofAcetylCoA

Changedsleeppatterns

FormationofsuccinylCoA

Easyfatigue

OxidationofFattyacids

Burningfootsyndrome

SynthesisofFattyacids
Carboxylaseenzyme
6.Biotin

Carboxylaseenzyme

Leinersdisease
Exfoliativedermatitis

7.Vit.B12(cyanocobalamine

ToconvertMethylmalonylCoA

Methylmalonylurea

Homocystinuria

Homocysteinemethionine

Perniciousanemia

SCID

Ribonucleotide

Deoxyribonucleotide

(Q.43) Whichofthefollowingarechaperones?
(a)

Amyloidprecursorprotein

(b)

BenceJonesprotein

(c)

Conjugatedprotein

(d)

Heatshockprotein

YourResponse:

CorrectAnswer:

Exp:

Heatshockprotein(RefHarperBiochemistry26thed.35,36)
Heatshockproteinanyofagroupofproteinsfirstidentifiedassynthesizedinresponsetohyperthermia,
hypoxia,orotherstressesandbelievedtoenablecellstorecoverfromthesestresses.Manyhavebeen
foundtobemolecularchaperonesandaresynthesizedabundantlyregardlessofstress.
CHAPERONES
Aretheproteinthatpreventfaultyfoldingandunproductiveinteractionsofotherproteins
Playaroleintheassemblyorproperfoldingofotherproteins.
ie.preventwrongfoldingandallowsfolding,onlyinthecorrectdirection.
Theyhelpintheassemblyofthetertiaryandquantimetrystructureofproteins.
Theyarenotpartofthematureprotein,andtheyhaveATPaseactivity.
Somepropertiesofchaperoneproteins
Presentinawiderangeofspeciesfrombacteriatohumans.
Manyaresocalledheatshockprotein(Hsp)
Someareinduciblebyconditionsthatcauseunfoldingofnewlysynthesizedprotein(eg.temp.,and
variouschemicals)
Theybindstopredominantlyhydrophobicregionsofunfoldedandaggregatedproteins.
Theyactinpartasaqualitycontrolor,editingmechanism.Fordetectingmisfoldedorotherwisedefective
proteins.
MostchaperonsshowsassociatedATPaseactivity,withATPorADPbeinginvolvedintheprotein
chaperoneinteraction
Foundinvariouscellularcomponentssuchas,cytosol,mitochondriaandthelumenoftheendoplasmic
reticulum.
Somechaperonesandenzymesinvolvedinfoldingthatarelocatedintherough
endoplasmicreticulum:
BIP(Igheavychainbindingprotein
GRP94(glucoseregulatoryprotein)
Calnexin(Ca++bindingprotein)
Calreticulum(Ca++bindingprotein)
PDI(Proteindisulfideisomerase)
PPI(Peptidylprolylcistransisomerase)

http://dbmci.examonair.com/Result/ShowAllQuestionInHtml.aspx?testid=3572

26/99

22/03/2016

DrBhatiaMedicalCoachingInstitute:OnlineTestPlatform
(Q.44) TruestatementsregardingVitaminB6arethefollowingEXCEPT:
(a) VitaminB6DeficiencyIsRare.
(b) VitaminB6Deficiencyresultsinabnormalitiesoftryptophanandmethioninemetabolism.
(c) VitaminB6StatusisAssessedbyAssayingErythrocyteAminotransferases.
(d) InExcess,VitaminB6CausesMotorNeuropathy.
YourResponse:

CorrectAnswer:

Exp:

(RefHarpersBiochemistry26thed.166,491)
VitaminB6DeficiencyIsRare.
Althoughclinicaldeficiencydiseaseisrare,thereisevidencethatasignificantproportionofthepopulation
havemarginalvitaminB6status.
Moderatedeficiencyresultsinabnormalitiesoftryptophanandmethioninemetabolism.
Increasedsensitivitytosteroidhormoneactionmaybeimportantinthedevelopmentofhormone
dependentcancerofthebreast,uterus,andprostate,andvitaminB6statusmayaffecttheprognosis.
VitaminB6StatusIsAssessedbyAssayingErythrocyteAminotransferases.
ThemostwidelyusedmethodofassessingvitaminB6statusisbytheactivationoferythrocyte
aminotransferasesbypyridoxalphosphateaddedinvitro,expressedastheactivationcoefficient.
InExcess,VitaminB6CausesSensoryNeuropathy.
Thedevelopmentofsensoryneuropathyhasbeenreportedinpatientstaking27gofpyridoxineperdayfor
avarietyofreasons(thereissomeslightevidencethatitiseffectiveintreatingpremenstrualsyndrome).
Therewassomeresidualdamageafterwithdrawalofthesehighdoses;otherreportssuggestthatintakesin
excessof200mg/dareassociatedwithneurologicdamage.

(Q.45) Presenceofwhichofthefollowingintheexpressionofvectorensuresincreaseintheyieldofrecombinantproteinproduced?
(a)

Induciblepromoter

(b)

Genecodingforproteaseinhibitor

(c)

Translationinitiationsignals

(d)

TranscriptionandTranslationterminationsignals

YourResponse:

CorrectAnswer:

Exp:

(RefHarperBiochemistry28th/pg.)
Theprimaryadvantageofaninduciblepromoterexpressionsystemisthatproductionoftherecombinant
proteincanbebiochemicallycontrolled,allowingfortheseparationofuniquegrowthandproduction
phasesoftheculture.Duringthegrowthphase,thecultureisrapidlygrowntohighcelldensitypriorto
inductionwithouttheextrametabolicburdenofexogenousproteinproduction,thusminimizingthe
nonproductiveperiodoftheculture.Inductionofthecultureathighcelldensityensuresthatthevolumetric
productionwillbemaximized.
Thus,aInduciblepromoterintheexpressionofvectorincreasestheyieldofrecombinantproteinproduced.
Giventheimportanceofproteaseinhibitors,characterizationofproteaseinhibitors,whichcanplayarolein
preventing,amelioratingorcorrectingduysfunctionsanddiseases.

(Q.46) Twoplantsgrowningardensoil,onetoexpressGreenFluoroscentProtein(GFP)andotherfireflyluciferase.Indark,whichofthe

followingwillbeseen?
(a)

PlantexpressingGPFwillglow

(b)

Plantexpressingfireflyluciferasewillglow

(c)

Boththeplantswillglow

(d)

Boththeplantswillnotglow

YourResponse:

CorrectAnswer:

Exp:

Plantexpressingfireflyluciferasewillglow.
Fireflyluciferaseisbyfarthemostcommonlyusedbioluminescentreported.Fireflyluciferaseisslightly
differentfromGFP,aphotoprotein.Luciferaseisagenerictermfortheclassofoxidativeenzymesusedin
bioluminescenceandisdistinctfromaphotoprotein.
Thegreenfluorescentprotein(GFP)isaproteincomposedof238aminoacidresidues(26.9kDa)that
exhibitsbrightgreenfluorescencewhenexposedtobluelight.Photoproteins(GPF)dontrequireanyspecial
pigments,enzymesorchemicalstosetitoff.Onceexpressed,itsimplyneedsmineralsubstratestoemit
fluorescence.GFPwasoriginallyrecruitedfromjellyfish.Ithasthesamepurposeastheluciferaseenzymein
molecularbiology,tomarktheexpressionofselectgenes.Atleastinthelab,itGFPalsorequirestoshine
bluelight(orsometimesUV)forittoemitthegreenfluorescence.Itssuchastandardtoolnow,inmice,
plants,flies,fishstudies.

(Q.47) Whichofthefollowingisrequiredfortransferofaminogroup?
(a)

Transferase

(b)

Oxidase

(c)

Transaminase

(d)

Anhydrase

http://dbmci.examonair.com/Result/ShowAllQuestionInHtml.aspx?testid=3572

27/99

22/03/2016

DrBhatiaMedicalCoachingInstitute:OnlineTestPlatform
YourResponse:

CorrectAnswer:

Exp:

Transaminase.(RefHarperbiochemistry26th/pg.135)
Aminotransferase(transaminase)reactionsform:
Pyruvatefromalanine,
Oxaloacetatefromaspartate,and
ketoglutaratefromglutamate.
Becausethesereactionsarereversible,thecyclealsoservesasasourceofcarbonskeletonsforthe
synthesisoftheseaminoacids.
Otheraminoacidscontributetogluconeogenesisbecausetheircarbonskeletonsgiverisetocitricacidcycle
intermediates.
Alanine,cysteine,glycine,hydroxyproline,serine,threonine,andtryptophanyieldpyruvate;
Arginine,histidine,glutamine,andprolineyieldketoglutarate;
Isoleucine,methionine,andvalineyieldsuccinylCoA;
Tyrosineandphenylalanineyieldfumarate.

(Q.48) FalseregardingCytochomeP450s?
(a) Theyarehemoproteins
(b) Theyarepresentinhighestamountsinlivercellsandenterocytes
(c) AtleastsixdifferentspeciesofcytochromeP450arepresentintheendoplasmicreticulumofhumanliver
(d) NADHisinvolvedinthereactionmechanismofcytochromeP450
YourResponse:

CorrectAnswer:

Exp:

NADHisinvolvedinthereactionmechanismofcytochromeP450
REF:Harper'sIllustratedBiochemistry,28e>Chapter53.
RememberCytochromeP450isaveryimportanttopicforyourentrance.Hereisadetailedaccountof
thesewproteins:
Becauseofthelargenumberofisoforms(about150)thathavebeendiscovered,itbecameimportantto
haveasystematicnomenclatureforisoformsofP450andfortheirgenes.Thisisnowavailableandinwide
useandisbasedonstructuralhomology.TheabbreviatedrootsymbolCYPdenotesacytochromeP450.This
isfollowedbyanArabicnumberdesignatingthefamily;cytochromeP450sareincludedinthesamefamily
iftheyexhibit40%ormoreaminoacidsequenceidentity.TheArabicnumberisfollowedbyacapitalletter
indicatingthesubfamily,iftwoormoremembersexist;P450sareinthesamesubfamilyiftheyexhibit
greaterthan55%sequenceidentity.TheindividualP450sarethenarbitrarilyassignedArabicnumerals.
Thus,CYP1A1denotesacytochromeP450thatisamemberoffamily1andsubfamilyAandisthefirst
individualmemberofthatsubfamily.ThenomenclatureforthegenesencodingcytochromeP450sis
identicaltothatdescribedaboveexceptthatitalicsareused;thus,thegeneencodingCYP1A1isCYP1A1.
Likehemoglobin,theyarehemoproteins.
Theyarewidelydistributedacrossspecies,includingbacteria.
Theyarepresentinhighestamountinlivercellsandenterocytesbutareprobablypresentinalltissues.In
liverandmostothertissues,theyarepresentmainlyinthemembranesofthesmoothendoplasmic
reticulum,whichconstitutepartofthemicrosomalfractionwhentissueissubjectedtosubcellular
fractionation.Inhepaticmicrosomes,cytochromeP450scancompriseasmuchas20%ofthetotalprotein.
P450sarefoundinmosttissues,thoughofteninlowamountscomparedwithliver.Intheadrenal,theyare
foundinmitochondriaaswellasintheendoplasmicreticulum;thevarioushydroxylasespresentinthat
organplayanimportantroleincholesterolandsteroidbiosynthesis.ThemitochondrialcytochromeP450
systemdiffersfromthemicrosomalsysteminthatitusesanNADPHlinkedflavoprotein,adrenodoxin
reductase,andanonhemeironsulfurprotein,adrenodoxin.Inaddition,thespecificP450isoformsinvolved
insteroidbiosynthesisaregenerallymuchmorerestrictedintheirsubstratespecificity.
AtleastsixdifferentspeciesofcytochromeP450arepresentintheendoplasmicreticulumofhumanliver,
eachwithwideandsomewhatoverlappingsubstratespecificitiesandactingonbothxenobioticsand
endogenouscompounds.ThegenesformanyisoformsofP450(frombothhumansandanimalssuchasthe
rat)havebeenisolatedandstudiedindetailinrecentyears.Thecombinationoftherebeinganumberof
differenttypesandeachhavingarelativelywidesubstratespecificityexplainswhythecytochromeP450
familycanmetabolizethousandsofdifferentchemicals.
NADPH,notNADH,isinvolvedinthereactionmechanismofcytochromeP450.Theenzymethatuses
NADPHtoyieldthereducedcytochromeP450,shownatthelefthandsideoftheaboveequation,iscalled
NADPHcytochromeP450reductase.ElectronsaretransferredfromNADPHtoNADPHcytochromeP450
reductaseandthentocytochromeP450.Thisleadstothereductiveactivationofmolecularoxygen,and
oneatomofoxygenissubsequentlyinsertedintothesubstrate.Cytochromeb5,anotherhemoprotein
foundinthemembranesofthesmoothendoplasmicreticulum,maybeinvolvedasanelectrondonorin
somecases.
LipidsarealsocomponentsofthecytochromeP450system.Thepreferredlipidisphosphatidylcholine,
whichisthemajorlipidfoundinmembranesoftheendoplasmicreticulum.
MostisoformsofcytochromeP450areinducible.Forinstance,theadministrationofphenobarbitalorof
manyotherdrugscauseshypertrophyofthesmoothendoplasmicreticulumandathreetofourfold
increaseintheamountofcytochromeP450within45days.Themechanismofinductionhasbeenstudied
extensivelyandinmostcasesinvolvesincreasedtranscriptionofmRNAforcytochromeP450.However,
certaincasesofinductioninvolvestabilizationofmRNA,enzymestabilization,orothermechanisms(eg,an
effectontranslation).InductionofcytochromeP450hasimportantclinicalimplications,sinceitisa
biochemicalmechanismofdruginteraction.Adruginteractionhasoccurredwhentheeffectsofonedrug
arealteredbyprior,concurrent,orlateradministrationofanother.Asanillustration,considerthesituation
whenapatientistakingtheanticoagulantwarfarintopreventbloodclotting.Thisdrugismetabolizedby
CYP2C9.Concomitantly,thepatientisstartedonphenobarbital(aninducerofthisP450)totreatacertain

http://dbmci.examonair.com/Result/ShowAllQuestionInHtml.aspx?testid=3572

28/99

22/03/2016

DrBhatiaMedicalCoachingInstitute:OnlineTestPlatform
typeofepilepsy,butthedoseofwarfarinisnotchanged.After5daysorso,thelevelofCYP2C9inthe
patient'sliverwillbeelevatedthreetofourfold.Thisinturnmeansthatwarfarinwillbemetabolizedmuch
morequicklythanbefore,anditsdosagewillhavebecomeinadequate.Therefore,thedosemustbe
increasedifwarfarinistobetherapeuticallyeffective.Topursuethisexamplefurther,aproblemcouldarise
lateronifthephenobarbitalisdiscontinuedbuttheincreaseddosageofwarfarinstaysthesame.The
patientwillbeatriskofbleeding,sincethehighdoseofwarfarinwillbeevenmoreactivethanbefore,
becausethelevelofCYP2C9willdeclineoncephenobarbitalhasbeenstopped.Anotherexampleofenzyme
inductioninvolvesCYP2E1,whichisinducedbyconsumptionofethanol.Thisisamatterforconcern,
becausethisP450metabolizescertainwidelyusedsolventsandalsocomponentsfoundintobaccosmoke,
manyofwhichareestablishedcarcinogens.Thus,iftheactivityofCYP2E1iselevatedbyinduction,thismay
increasetheriskofcarcinogenicitydevelopingfromexposuretosuchcompounds.
CertainisoformsofcytochromeP450(eg,CYP1A1)areparticularlyinvolvedinthemetabolismofpolycyclic
aromatichydrocarbons(PAHs)andrelatedmolecules;forthisreasontheywereformerlycalledaromatic
hydrocarbonhydroxylases(AHHs).ThisenzymeisimportantinthemetabolismofPAHsandin
carcinogenesisproducedbytheseagents.Forexample,inthelungitmaybeinvolvedintheconversionof
inactivePAHs(procarcinogens),inhaledbysmoking,toactivecarcinogensbyhydroxylationreactions.
Smokershavehigherlevelsofthisenzymeinsomeoftheircellsandtissuesthandononsmokers.Some
reportshaveindicatedthattheactivityofthisenzymemaybeelevated(induced)intheplacentaofa
womanwhosmokes,thuspotentiallyalteringthequantitiesofmetabolitesofPAHs(someofwhichcouldbe
harmful)towhichthefetusisexposed.
CertaincytochromeP450sexistinpolymorphicforms(geneticisoforms),someofwhichexhibitlow
catalyticactivity.Theseobservationsareoneimportantexplanationforthevariationsindrugresponses
notedamongmanypatients.OneP450exhibitingpolymorphismisCYP2D6,whichisinvolvedinthe
metabolismofdebrisoquin(anantihypertensivedrug)andsparteine(anantiarrhythmicandoxytocic
drug).CertainpolymorphismsofCYP2D6causepoormetabolismoftheseandavarietyofotherdrugsso
thattheycanaccumulateinthebody,resultinginuntowardconsequences.Anotherinteresting
polymorphismisthatofCYP2A6,whichisinvolvedinthemetabolismofnicotinetoconitine.Three
CYP2A6alleleshavebeenidentified:awildtypeandtwonullorinactivealleles.Ithasbeenreportedthat
individualswiththenullalleles,whohaveimpairedmetabolismofnicotine,areapparentlyprotected
againstbecomingtobaccodependentsmokers.Theseindividualssmokeless,presumablybecausetheir
bloodandbrainconcentrationsofnicotineremainelevatedlongerthanthoseofindividualswiththewild
typeallele.IthasbeenspeculatedthatinhibitingCYP2A6maybeanovelwaytohelppreventandtotreat
smoking.
(Q.49) TrueaboutmanagementofLungcancerisallexcept:
(a) SurgicalresectionisnotroutinelyrecommendedforNSCLC.
(b) PalliativeradiotherapyisavitalRxofSCLCpatients.
(c) ErlotinibisusedinsecondlinetherapyofNSCLC.
(d) ChemoradiationisthetreatmentofchoiceforbulkystageIIIAorIIIBdiseasewithoutpleuraleffusion.
YourResponse:

CorrectAnswer:

Exp:

SurgicalresectionisnotroutinelyrecommendedforNSCLC(NonSmallCellLungCancer).(RefHarrison
17th/pg.560)
RxofCaLung:
SurgicalresectionisnotroutinelyrecommendedforSCLC.(smallcelllungcancer).butitisdoneveryfrequentlyfornon
smallcell.

Onecanrememberthislikethatallsmalllungproblemsdosentneedknife.
ChemotherapyplusradiationtherapyisthetreatmentofchoiceforpatientswithbulkystageIIIAorIIIB
diseasewithoutpleuraleffusion(referredtoasdryIIIB).
PalliativeradiationtherapyisanimportantcomponentofthemanagementofSCLCpatients.
EGFRTargetedTherapy(ErlotinibanoralinhibitoroftheEGFRkinase)isusedinsecondlinetherapyof
NSCLC.
Squamouscell

CentralLocation(2/3),
Atelectasis,
Postobstructivepneumonia,
Maycavitate.

Adenocarcinoma

Usuallyperipheral,
Foundinscars,
Solitarynodule(52%),
Upperlobedistribution(69%).

Smallcell

Mediastinaladenopathy,
Hilarmass,
Smallorinvisiblelungnodule,
Highmetastaticpotential,
Rapidgrowth.
MaybeassociatedwithHypoglycemia,Cushing'ssyndrome,Inappropriate
secretionofADH,excessivegonadotropinsecretion.

Largecell
Largeperipheralmass
undifferentiated(10%) Pleuralinvolvement.
(Q.50) A51yearoldmanpresentswithbilateralenlargementofhisbreasts.Physicalexaminationisotherwiseunremarkable,andthe

diagnosisofgynecomastiaismade.Whichofthefollowinghistologicfeaturesismostlikelytobeseenwhenexaminingabiopsy

http://dbmci.examonair.com/Result/ShowAllQuestionInHtml.aspx?testid=3572

29/99

22/03/2016

DrBhatiaMedicalCoachingInstitute:OnlineTestPlatform
specimenfromthismansbreasttissue?
(a)

Atrophicductalstructureswithincreasednumbersoflipocytes

(b)

Dilatedductsfilledwithgranular,necrotic,acidophilicdebris

(c)

Expansionoflobulesbymonotonousproliferationofepithelialcells

(d)

Proliferationofductsinhyalinizedfibroustissuewithperiductaledema

YourResponse:

CorrectAnswer:

Exp:

Proliferationofductsinhyalinizedfibroustissuewithperiductaledema
Gynecomastia(enlargementofthemalebreast)histologicallyrevealsepithelialhyperplasiawithinthe
ductsthatissurroundedbyhyalinizedfibroustissue.Itiscausedbyanincreaseintheestrogentoandrogen
ratio.Thisabnormalitymaysometimesbefoundinmalesatthetimeofpuberty.Othercausesof
gynecomastiaincludeKlinefelterssyndrome(decreasedsecretionoftestosterone),testicularfeminization
(androgeninsensitivity),testiculartumors,cirrhosisoftheliver,alcoholabuse,increasedgonadotropin
levels(suchaschoriocarcinomaofthetestis),increasedprolactinlevels,drugs(suchasdigoxin),or
hyperthyroidism.Testicularneoplasmsthatareassociatedwithgynecomastiaaretumorsthatsecrete
humanchorionicgonadotropin(hCG),whichincreasesthesynthesisofestradiol.Testiculartumorsassoci
atedwiththeproductionofhCGincludegermcelltumors(choriocarcinomaandseminoma),Leydigcell
tumors,andSertolicelltumors.
Notemalebreastdoesnthavelobulessonoterminalductlobarunit(TDLU)iseverseenin

malebreast
Nolobulessonolobularhyperplasia.malebreastenlargementmeans.ductandonlyductproliferation
alongwithstromalhyperplasiaandhyalinizationofstroma
(Q.51) Trueaboutprionsis?
(a)

Theyareencodedbyviralgenome

(b)

Associatedwithmisfoldingofproteins

(c)

NonInfectious

(d)

Immunogenic

YourResponse:

CorrectAnswer:

Exp:

Associatedwithmisfoldingofproteins.
(RefRobbinspathology8th/pg.1308;Harper28th/pg.3940;Ananthanarayan7th/pg.567)
Proteinaceousparticlesdevoidofnucleicacid,thatcausesslowlyevolvingspongiformdegenerationof
centralnervoussystemwithoutproductionofimmuneresponseandduetoabnormalityofproteinfolding.
Prionsareinfectiousagents,proteininnature,devoidofDNAandRNA,andunusuallyresistanttophysical
andchemicalagentslikeheat,irradiation,andformalin.
NormalPrPisa30kDcellularproteinpresentinneurons.DiseaseoccurswhenthePrPundergoesa
conformationalchangefromitsnormalhelixcontainingisoform(PrPc)toanabnormalpleatedsheet
isoform,usuallytermedPrPsc(forscrapie).Associatedwiththeconformationalchange,PrPacquires
resistancetodigestionwithproteases,suchasproteinaseK.AccumulationofPrPscinneuraltissueseemsto
bethecauseofthepathologyinthesediseases,buthowthismaterialinducesthedevelopmentof
cytoplasmicvacuolesandeventualneuronaldeathisstillunknown.Westernblottingoftissueextractsafter
partialdigestionwithproteinaseKallowsdetectionofPrPsc,whichisdiagnostic.
TheconformationalchangeresultinginPrPscmayoccurspontaneouslyatanextremelylowrate
(resultinginsporadiccases)oratahigherrateifvariousmutationsarepresentinPrPc,suchasoccursin
familialformsofCJDandinGSSandfatalfamilialinsomnia.PrPsc,independentofthemeansbywhichit
originates,thenfacilitates,inacooperativefashion,theconversionofotherPrPcmoleculestoPrPsc
molecules.ItisthisactivityofPrPscthataccountsforthenatureofpriondiseases.
ThegeneencodingPrP,termedPRNP,showsahighdegreeofconservationacrossspecies.Avarietyof
mutationsinPRNPhavebeenfoundtounderliefamilialformsofpriondiseases.

(Q.52) Anautopsydoneinapatientwhosuccumbedtomyocardialinfarctionrevealswelldevelopedphagocytosisofdeadcellsandearly

fotmationoffibrovasculargranulationtissueatmargins.Whatisthetimeofdeathsincetheeventofinfarction?
(a)

12to24hrs

(b)

1to3days

(c)

3to7days

(d)

7to10days

YourResponse:

CorrectAnswer:

Exp:

7to10days
REFRobbins8thEdChap11
Anotherveryfrequentlyaskedtopic.Youmustknowthefollowingtablebyheart.
EVOLUTIONOFMORPHOLOGICCHANGESINMI:

TIME

GROSS
FEATURES

http://dbmci.examonair.com/Result/ShowAllQuestionInHtml.aspx?testid=3572

LIGHTMICROSCOPY

ELECTRONMICROSCOPY

30/99

22/03/2016

DrBhatiaMedicalCoachingInstitute:OnlineTestPlatform
REVERSIBLE
INJURY:
00.5hrs

None

None

Relaxationof
myofibrils,glycogenloss,mitochondrial
swelling

14hrs

None

Usuallynone,variablewavinessof
fibresatborder

Sarcollemaldisruption;
mitochondrialamorphousdensities

412hrs

Occaisionally Beginningcoagulation
darkmottling necrosis;edema;hemorrhage

1224hrs

Darkmottling Ongoingcoagulationnecrosis;
pyknosisofnuclei;
myocytehypereosinophilia;marginal
contractionbandnecrosis;beginning
neutrophilicinfiltrate

13days

Mottlingwith Coagulationnecrosiswithlossof
yellowtan
nucleiandstriations;interstitial
infarctcentre infiltrationofneutrophils

37days

Hyperemic
border;
centralyellow
tansoftening

710days

Maximally
Welldevelopedphagocytosisofdead
yellowtanand cells;earlyfotmationoffibrovascular
soft;with
granulationtissueatmargins
depressedred
tanmargins

1014days

Redgrey
Wellestablishedgranulationtissue
depressed
withnewbloodvesselsandcollagen
infarctborders deposition

23weeks

Graywhite
Increasedcollagendepositionwith
scar;
decreasedcellularity
progressive
fromborder
towardscore
ofinfarct

2monthsand
later

Scarring
complete

IRREVERSIBLE
INJURY

Beginningdisintegrationofdead
myofibres;withdeadneutrophils;
earlyphagocytosisofdeadcellsby
macrophagesatinfarctborder

Densecollagenousscar

(Q.53) CD4isnotimportantforwhichofthefollowing
(a)

Antibodyproduction

(b)

CytotoxicityofTcells

(c)

MemoryBcells

(d)

Opsonization

YourResponse:

CorrectAnswer:

Exp:

OpsonizationRefRobbins,8thed.,page395397
CD4performsthefollowinghelperfunctions
i.TheyhelpBcellsdevelopintoantibodyproducingplasmacells.SomeactivatedBcellsalsoformmemory
cells,whichcanremainquiescentforlongperiodsbutcapturebybeingactivatedrapidlyuponreexposure
toantigen
ii.TheyhelpCD8TcellstobecomeactivatedcytotoxicTcells
ThesefunctionareperformedbytwosubpopulationofCD4cells:ThlcellsactivatecytotoxicTcellsby
producingIL2andhelpinitiatedelayedhypersensitivityresponsebyproducingprimarilyIL2&yinterferon.
whereasTh2cellsperformBcellhelperfunctionbyproducingprimarilyIL4andIL5.

(Q.54) Ectopicrestofnormaltissueisknownas?
(a)

Choristoma

(b)

Hamartoma

(c)

Pseudotumor

(d)

Lymphoma

YourResponse:

CorrectAnswer:

Exp:

Choriostma.(RefRobbins8th/pg.174)
Hamartomaisamalformationthatpresentsasamassofdisorganizedtissueindigenoustotheparticular
site.Onemayseeamassofmaturebutdisorganizedhepaticcells,bloodvessels,andpossiblybileducts
withintheliver,ortheremaybeahamartomatousnoduleinthelungcontainingislandsofcartilage,
bronchi,andbloodvessels.Anothermisnomeristhetermchoristoma.Thiscongenitalanomalyisbetter
describedasaheterotopicrestofcells.Forexample,asmallnoduleofwelldevelopedandnormally
organizedpancreatictissuemaybefoundinthesubmucosaofthestomach,duodenum,orsmallintestine.

http://dbmci.examonair.com/Result/ShowAllQuestionInHtml.aspx?testid=3572

31/99

22/03/2016

DrBhatiaMedicalCoachingInstitute:OnlineTestPlatform
ThisheterotopicrestmayberepletewithisletsofLangerhansandexocrineglands.ThetermChoristoma,
connotinganeoplasm,impartstotheheterotopicrestagravityfarbeyonditsusualtrivialsignificance.
Althoughtheterminologyofneoplasmsisregrettablynotsimple,itisimportantbecauseitisthelanguage
bywhichthenatureandsignificanceoftumorsarecategorized.
(Q.55) Schaumannbodiesareseenin?
(a)

Sarcoidosis

(b)

Chronicbronchitis

(c)

Asthma

(d)

Syphilis

YourResponse:

CorrectAnswer:

Exp:

Sarcoidosis.(RefRobbins8th/pg.738)
Sarcoidosisisasystemicdiseaseofunknowncausecharacterizedbynoncaseatinggranulomasinmany
tissuesandorgans.
AssociationwithcertainHLAgenotypes(e.g.,classIHLAA1andHLAB8).
Nounequivocalevidencetosuggestthatsarcoidosisiscausedbyaninfectiousagent.
Histologically,allinvolvedtissuesshowtheclassicnoncaseatinggranulomas,eachcomposedofan
aggregateoftightlyclusteredepithelioidcells,oftenwithLanghansorforeignbodytypegiantcells.Central
necrosisisunusual.Withchronicity,thegranulomasmaybecomeenclosedwithinfibrousrimsormay
eventuallybereplacedbyhyalinefibrousscars.
Twoothermicroscopicfeaturesareoftenpresentinthegranulomas:
laminatedconcretionscomposedofcalciumandproteinsknownasSchaumannbodiesand
stellateinclusionsknownasasteroidbodiesenclosedwithingiantcellsfoundinapproximately60%ofthe
granulomas.
Althoughcharacteristic,thesemicroscopicfeaturesarenotpathognomonicofsarcoidosisbecauseasteroid
andSchaumannbodiesmaybeencounteredinothergranulomatousdiseases(e.g.,tuberculosis).
Pathologicinvolvementofvirtuallyeveryorganinthebodyhasbeencitedatonetimeoranother.
REMEMBER:ListofvariousBODIESseeninvariousdiseases:
BODIES

DISEASE

Hematoxylinbody

LibmanSacsendocarditis

MeyersKouvenaarsbody

Filariasis

Ricebodies

FibrousloosebodiesinjointsinTBarthritis

MichalisGuttmanbody

Malakoplakia

Herringbody

Posteriorpituitarybodies

HalberstaedlerProwazekbody

Trachomatosis

Picksbody

FrontoTemporaldementia/Picksdisease

Moosersbody

Endemictyphus

Loforabody

Familialmyoclonicepilepsy

Tuftstonebody

Metachromaticleukodystrophy

Cystoidbody

CMVretinitis

Paschenbody

Elementarybodyofvariola(Smallpox)

Guarneribody

Inclusionbodyofvariolavaccinia

Molluscumbody

Inclusionbodyofmolluscumcontagiosum

LDbody

Leishmaniasis(Itisaamastigotewithinamacrophage)

Negribody

Rabies(Intracytoplasmic)

Bollingerbody

Fowlpox

HendersonPetersonbody

Molluscumcontagiosum

Torresbody

Yellowfever

WarthinFinkeledybody

Measles

Miyagawabody

LGV

LevinthalColeLillybody

Psittacosis

Copperpennybody

HistologiccharacteristicofChromoblastomycosis

Parappenheimerbody

Sideroblasticanemia

Gammagandybody

Sicklecellanemia

Massonbody

Cryptogenicorganizingpneumonia

Asteriodbody,Schaumannbody

Sarcoidosis

RusselandDutcherbody

Multiplemyeloma

Globoidbody

Krabbesdisease

Aschoffbody

Rheumaticheartdisease

(Q.56) LibmanSachsendocarditisisassociatedwith?
(a)

Rheumaticheartdisease

(b)

SLE

http://dbmci.examonair.com/Result/ShowAllQuestionInHtml.aspx?testid=3572

32/99

22/03/2016

DrBhatiaMedicalCoachingInstitute:OnlineTestPlatform
(c)

Carcinoma

(d)

Fungalinfection

YourResponse:

CorrectAnswer:

Exp:

SLE(RefRobbinspathology8th/pg.408
LibmanSacksEndocarditis
LibmanSacksendocarditisreferstosterilevegetationsthatcandeveloponthevalvesofpatientswith
systemiclupuserythematosus.
Theselesionspresumablyoccurbecauseofimmunecomplexdepositionandthushaveassociated
inflammation.
Withincreasinguseofsteroidsfortreatmentoflupus,LibmanSacksendocarditishasbecomefairly
uncommon.
VARIOUSTYPESOFVEGETATIONSSEENONVALVES:
characterizedbymicrobialinvasionofheartvalvesormuralendocardiumoften
withdestructionoftheunderlyingcardiactissues(valveperforation,ring
abscess).

Infectiveendocarditis

Bulky,friablevegetationscomposedofnecroticdebris,thrombus,and
organisms.
Thromboemboismcommon.
LibmanSacks
endocarditis

SLEcanbeassociatedwithLibmanSacksendocarditis
inwhichtherearemanyflat,reddishtanvegetationsspreadingoverthemitral
valveandchordae.
vegetationsonbothsidesofvalve
donotembolize.

NBTE(nonbacterial
typicallyexhibitssmall,blandvegetations,usuallyattachedatthelineofclosure.
thromboticendocarditis) Easilydetachable.
NBTEindebilitatedcancerandsepsispatientsismaranticendocarditis.
Rheumaticcarditis

Vegetationslargerthan10mm(notwellpenetratedbyantibiotics)
Alongthelineofclosure
Aschoffbodies

(Q.57) VirchowsTriadincludethefollowingEXCEPT?
(a)

Hypercoagulability

(b)

Stasisofblood

(c)

Injurytovessels

(d)

VenousThrombosis

YourResponse:

CorrectAnswer:

Exp:

VenousThrombosis
FactorswhichleadtovenousthrombosisThesewereoriginallydescribedbyVirchowoveracenturyago.He
suggestedthatthefollowingfactorsmayleadtoclottingintheveins.
VIRCHOWTRIAD:
Changesinthevesselwallwithdamagetotheendotheliumduetoinjuryorinflammation.Thisisknownto
happenfollowingpreviousdeepveinthrombosis.
Diminishedrateofbloodflowintheveins.Inmodernmedicalpracticethisoccursduringandafter
operations,andindebilitatingconditionssuchasstrokesandmyo
cardialinfarction.
Increasedcoagulabilityoftheblood.Thisalsooccursfollowingsurgeryandinthepresenceofinfectionor
systemicmalignancy.
FEWCLASSICALTRIADS:
AUSTRIANTRIAD

TheAustriantriadisnamedafterRobertAustrianwhoinitiallyidentifiedand
describedit.It'scomponentsare:
1.Pneumococcalpneumonia2.Meningitis3.Endocarditis(classicallyaorticvalve
endocarditisassociatedwithaorticregurgitation)

WADDELsTRIAD

Waddell'striadisrecognizedinclinicalpracticeasassociatedwithhighvelocity
accidentssuchasmotorvehicle,autopedestrian,orbicyclecrashes
Waddell'striadconsistsof
1.Femurfracture2.Intraabdominalorintrathoracicinjury3.Headinjury,

CHARCOTsTRIAD

TherearetwosetsofCharcot'striadsrelatingtoquiteseparatediseases.
A.Charcot'striadformultiplesclerosis
1.Nystagmus2.Intentiontremor3.Scanningorstaccatospeech
Whilethistriadischaracteristicofmultiplesclerosis,itisnotconsidereddefinitely
diagnostic.

http://dbmci.examonair.com/Result/ShowAllQuestionInHtml.aspx?testid=3572

33/99

22/03/2016

DrBhatiaMedicalCoachingInstitute:OnlineTestPlatform
B.Charcot'striadforascendingcholangitis
1.Jaundice2.Fever,usuallywithrigors3.Rightupperquadrantabdominalpain.
Charcot'striadforascendingcholangitisisaresultofascendingcholangitis(an
infectionofthebileductusuallycausedbybacteriaascendingfromitsjunction
withtheduodenum).Whenthepatientpresentsalsowithhypotensionandmental
statuschanges,itisknownasReynolds'pentad
Charcot'striadsarenamedfortheFrenchneurologistwhofirstdescribedthese
combinationsofsignsinrelationtothesediseases,JeanMartinCharcot(1825
1893.
VIRCHOWsTRIAD

Virchow'striadpresentsthethreebroadcategoriesoffactorsthatarethoughtto
contributetothrombosis.
Thetriadconsistsof:
1.Alterationsinnormalbloodflow2.Injuriestothevascularendothelium3.
Alterationsintheconstitutionofblood(hypercoagulability)
Alterationinbloodflowcanincludeturbulence,stasis,mitralstenosis,andvaricose
veins.Injuriestothevascularendotheliumcanbecausebydamagetotheveins
arisingfromshearstressorhypertension.Hypercoagubilitycanbeaconsequence
ofnumerouspossibleriskfactorssuchashyperviscosity,deficiencyofantithrombin
III,nephroticsyndrome,changesafterseveretraumaorburn,disseminatedcancer,
latepregnancyanddelivery,race,age,smoking,andobesity.
Virchow'striadwasfirstformulatedbytheGermanphysicianRudolfVirchow
(18211902)in1856.

BECKsTRIAD

Beck'striadwasdescribedbythethoracicsurgeonCaludeS.Beckin1935.It's
componentsare:
1.Distendedneckveins2.Distantheartsounds3.Hypotension
i.e.risingvenouspressure,fallingarterialpressure,anddecreasedheartsounds
foundinthepresenceofpericardialtamponade.

BERGMANsTRAID

Bergmanstriadisseenwithfatembolisyndrome.It'scomponentsare:
1.Mentalstatuschanges2.Petechiae(oftenintheaxilla/thorax)3.Dyspnea

CARCINOIDTRIAD

Theelementsofthecarcinoidtriadare:
1.Flushing2.Diarrhea3.Rightsidedheartfailure
Bronchospasmissometimesalsopresent.

CUSHINGsTRIAD

Cushing'striad(nottobeconfusedwiththeCushingreflex)isasignofincreased
intracranialpressure.Itisthetriadof:
1.Hypertension(progressivelyincreasingsystolicbloodpressure)
2.Bradycardia
3.Wideningpulsepressure(anincreaseinthedifferencebetweensystolicand
diastolicpressureovertime)
Cushing'striadsuggestsacerebralhemorrhageinthesettingoftraumaoranspace
occupyinglesion(e.g.braintumor)thatisgrowingandapossibleimpendingfatal
herniationofthebrain.Cushing'striadisnamedafteranAmericanneurosurgeon
HarveyWilliamsCushing(18691939).

HUTCHINSONTRIAD

Hutchinson'striadisacommonpatternofpresentationforcongenitalsyphilis.It
consistsof:
1.Interstitialkeratitis2.Hutchinsonincisors3.Eighthnervedeafness
Hutchinson'striadisnamedafterSirJonathanHutchinson(18281913).

MACDONALDTRIAD

TheMacdonaldtriadisasetofthreebehavioralcharacteristicswhichare
associatedwithsociopathicbehavior.Thesebehavioralcharacteristicsarefoundin
thechildhoodhistoriesofindividualswithsociopathicbehavior:
1.Enuresis(bedwetting)2.Firesetting3.Torturingsmallanimals
TheMacdonaldtriadisalsoknownasthetriadofsociopathy.Itwasfirstidentified
byaforensicpsychiatrist,JohnMarshallMacdonald(November7,1920
December16,2007),ina1963paperintheAmericanJournalofPsychiatrytitled
"TheThreattoKill".
TheMacdonalttriadisconsideredpredictiveoffuturecriminalbehavior.

Meltzerstriad

Purpura,arthralgiaandmyalgia.
IsgenerallyseenwithpolyclonalCryoglobulinemiasseeninessential,viral,
orconnectivetissuediseaseassociatedCryoglobulinemias.

(Q.58) Nitrobluetetrazolium(NBT)dyetestisdoneforassessingdefectin?

http://dbmci.examonair.com/Result/ShowAllQuestionInHtml.aspx?testid=3572

34/99

22/03/2016

DrBhatiaMedicalCoachingInstitute:OnlineTestPlatform
(a)

Tcells

(b)

Bcells

(c)

Phagocytes

(d)

Platelets

YourResponse:

CorrectAnswer:

Exp:

Phagocytes(RefRobbinsPathology7thed.Pg.63)
NBT:
.Nitrobluetetrazoliumisusedinadiagnostictest,particularlyforchronicgranulomatousdisease

andotherdiseasesofphagocytefunction.WhenthereisanNADPHoxidasedefect,thephagocyte
isunabletomakereactiveoxygenspeciesorradicalsrequiredforbacterialkilling.Asaresult,
bacteriamaythrivewithinthephagocyte.Thehigherthebluescore,thebetterthecellisat
producingreactiveoxygenspecies.Normally,whitebloodcellscalledneutrophilsmakeachemicalthat
killsbacteria.Inchronicgranulomatousdisease,thischemicalismissing.Thechemicalthatkillsthebacteria
isthesameonethatcausesNBTtochangefromcleartodeepblue.Ifthechemicalismissing,thewhite
bloodcellswillnotchangecolorwhenNBTisadded.Nitrobluetetrazoliumisachemicalcompound

composedoftwotetrazolemoieties.Itisusedinimmunologyforsensitivedetectionofalkaline
phosphatase(withBCIP).NBTservesastheoxidantandBCIPistheAPsubstrate(andgivesalso
darkbluedye).
(Q.59) Duringoxidativeburstinphagocytosiswhichenzymesplayskeyrole?
(a)

Oxidase

(b)

Peroxidase

(c)

Carboxylase

(d)

Hydrolase

Your
Response:
Correct
Answer:
Exp:

A
Oxidase(RefRobbinsPathology7thed.Pg.63)
OXYGENDEPENDENTRESPIRATORYBURST:
NADPHoxidase(deficiency=chronicgranulomatous
disease)
Superoxidedismutase
Myeloperoxidase
Catalase
Glutathionereductase
Glucose6phosphatedehydrogenase(G6PD)

(Q.60) Incongenitaldystrophicepidermolysisbullosa,mutationisseeningenecodingfor?
(a)

Laminin4

(b)

CollagentypeVII

(c)

Alpha6integrin

(d)

Keratin14

YourResponse:

CorrectAnswer:

Exp:

CollagentypeVII(RefRobbinspathology7th/pg.106;table32)
COLLAGEN

TISSUEDISTRIBUTION

GENETICDISORDERS

TYPE
I

Ubiquitousinhardandsofttissues OsteogenesisImperfectaQ
EhlersDanlossyndromearthrochalasias
type

II

Cartilage,intervertebraldisk,
vitreous

AchondrogenesistypeII,spondyloepiphyseal
dysplasiasyndrome

III

Holloworgans,softtissues

VascularEhlersDanlossyndrome

IV

Basementmembranes

AlportsyndromeQ

Softtissues,bloodvessels

ClassicalEhlersDanlossyndrome

VI

Ubiquitousinmicrofibrils

Bethlemmyopathy

VII

Anchoringfibrilsatdermal
epidermaljunctions

DystrophicepidermolysisbullosaQ

IX

Cartilage,intervertebraldisks

Multipleepiphysealdysplasias

IX

Cartilage,vitreous

Sticklersyndrome

XVII

Transmembranecollagenin
epidermalcells

Benignatrophicgeneralizedepidermolysis
bullosa

XVandXVIII

Endostatinformingcollagens,

Knoblochsyndrome(typeXVIIIcollagen)

http://dbmci.examonair.com/Result/ShowAllQuestionInHtml.aspx?testid=3572

35/99

22/03/2016

DrBhatiaMedicalCoachingInstitute:OnlineTestPlatform
endothelialcells
(Q.61) Filariformlarvaeareseenwith?
(a)

Enterobiusvermicularis

(b)

Ascarislumbricoides

(c)

Necatoramericans

(d)

Trichuristrichura

YourResponse:

CorrectAnswer:

Exp:

Ascarislumbricoides.(RefMedparasitologybyArora2nd/1321)
Filariformlarva:
Isinfectivethirdstagelarvaofthehookworm,Ascaris,andothernematodeswithpenetratinglarvaeorwith
larvaethatmigratethroughthebodytoreachtheintestine.

ExtraEdge:LaboratoryFindings(Harrison,17theditionpage1320)
MostcasesofascariasiscanbediagnosedbymicroscopicdetectionofcharacteristicAscariseggs(65by45
m)infecalsamples.Occasionally,patientspresentafterpassinganadultwormidentifiablebyitslarge
sizeandsmoothcreamcoloredsurfaceinthestoolorthroughthemouthornose.
Duringtheearlytranspulmonarymigratoryphase,wheneosinophilicpneumonitisoccurs,larvaecanbe
foundinsputumorgastricaspiratesbeforediagnosticeggsappearinthestool.
Theeosinophiliathatisprominentduringthisearlystageusuallydecreasestominimallevelsinestablished
infection.Adultwormsmaybevisualized,occasionallyserendipitously,oncontraststudiesofthe
gastrointestinaltract.
Aplainabdominalfilmmayrevealmassesofwormsingasfilledloopsofbowelinpatientswithintestinal
obstruction.
Pancreaticobiliarywormscanbedetectedbyultrasoundandendoscopicretrograde
cholangiopancreatography;thelattermethodalsohasbeenusedtoextractbiliaryAscarisworms.
(Q.62) AllofthefollowingserotypesofB.cereusareassociatedwithemetictypeoffoodpoisoningexcept?
(a)

(b)

(c)

(d)

10

YourResponse:

CorrectAnswer:

Exp:

10
DISEASESANDSPECIFICASSOCIATEDSEROTYPES
B.cereusfoodpoisoning

Emetic1,3,5
Diarrhoeal2,6,8,9,10,12

Humanbotulism

A,B,EandrarelyF.toxinproductionisdeterminedbybacteriophage
intypeC&D

Clostrialfoodpoisoning

Cl.PerfringestypeA

Enteritisnecroticans/
pigbel

Cl.PerfringestypeC(betatoxin)

Listeriamonocytogens
humaninfection

1/2a,1/2b,4b

Yersiniapseudotuberculosis 01
Yersiniaenerocolitica

03,08,09

S.pyogens
glomerulonephritis

49,5355,5964,1,12

Lateonsetneonatal
meningitisbygroupb
streptococcus

CapsulartypeIII

Pneumococcalpneumonia Type1totype8.Mostvirulenttype3
Meningococcalmeningitis A,B,C,W,Y
Legionellapneumophilla

http://dbmci.examonair.com/Result/ShowAllQuestionInHtml.aspx?testid=3572

Mostcommonserotype1.Hospitalacquireddiseaseand
poorprognosisserotype6

36/99

22/03/2016

DrBhatiaMedicalCoachingInstitute:OnlineTestPlatform
Chlamydiatrachomatis

EndemicblindingtrachomaA.B.Bo,CInclusionconjunctivitis,
genitalinfection,infantpneumoniaDto
RLymphogranulomavenerumL1,L2,L3

Cryptococcusneoformans Immunocompromised=vargrubii(serotype
A)immunocompetent=vargatti(serotypeB
andC)
Poliovirus

Mostcommonandmostepidemictype1Mostantigenicand
endemictype2Vaccineinducedparalysismutatedtype3

Influenzavirus

AntigenicvariationA>B
NoAntigenicvariationC

Parainfluenzavirus

Mostcommongroupinchildrentype1
Mostcommongroupinlowerrespiratorytractinfectionininfants
type3

Rotavirus

Humanillness=A>B>C
Adultdiarrohealrotavirus=grpB
Smallpopulationofpediatricdiarrhoealdiseases=group
c
MostcommonserotypesofgroupAinhumans=G1toG4,G9

Astrovirus

Mostcommon=1

Rhabdoviridea

Rabiesvirus=1,rabiesrelatedvirus=2,3,4

Adenovirus

Follicular(swimmingpool)conjunctivitis=
3,7epidemickeratoconjunctivitis
(shipyardeye)=8,9,37
Respiratorydiseaseinchildren=1,2,5,6Sorethroat,
pneumonia=3,4,7,14,21
Acuterespiratorydistressinmilitaryrecruits=
4,7,21
Diarrhoea=40,41Hemmorhagiccystitis=11,21

H.influenza

Meningitis=Biotype1Invasivedisease=capsular
subtypeb

Coxsackievirus

Acutehemmorhagiccystitis=A24Generaliseddiseaseof
newborn=B2to5Handfootmouthdiseaseandherpangina=A>
B
Pleurodynia=B>AJuvenilediabetes=
B4
Postviralfatiguesyndrome=B

Enterovirus

Acutehemmorhagiccystitis=ent70
Handfootmouthdiseaseandherpangina=ent71

HPV

Commonwarts/verrucavulgaris=
1,2,3,4Condylomaacuminata=
6,11
CIN(LOWRISK)=6,11Carcinoma(HIGHRISK)=
16,18,31,33,45Epidermodyspl.Verruciformis=
5,8

(Q.63) WhichofthefollowingshowRegressivemetamorphosis?
(a)

Hydatidcyst

(b)

Cysticercoid

(c)

Cysticercusbovis

(d)

Cysticercuscellulosae

YourResponse:

CorrectAnswer:

Exp:

Hydatidcyst.(RefMedicalparasitologybyRajeshKaryakarte1sted/pg.117&256)

http://dbmci.examonair.com/Result/ShowAllQuestionInHtml.aspx?testid=3572

37/99

22/03/2016

DrBhatiaMedicalCoachingInstitute:OnlineTestPlatform
RegressivemetamorphosisisshownbyHydatidcyst.
itisseeninhydatiddisease.....therupturedhydatidcystreleasesproscolicesintotissueswhichdonot
developintoadultformsaccordingtolawofmigrationofcystodes....buttheyformanewhydatidtissue
againstthelaw....thisiscalledregressivemetamorphosis.
Secondaryechinococcosisoccursasacomplicationofaprimarycyst.Theimportanceofoperative
techniqueduringtheremovaloftheprimarycystisemphasized,becausetheintraoperativedisemination
occursmostfrequentlyafterasurgicalperformance.Secondaryechinococcosisgeneratesbyasexual
regressivemetamorphosisoflarvalelementintrolarvalforms.
LAWOFMIGRATIONOFCYSTODE:Acystodehastocompleteitslifecyclewithalterationofgeneration
alongwithalterationofhosts.
Whichmeansnormalistoformadultafterlarvabutinechinococcus(hydatidcyst)whathappensis
regressivemetamorphosis,larvaformslarva.
Diagnosis(Harrison,17theditionpage1339)
Radiographicandrelatedimagingstudiesareimportantindetectingandevaluatingechinococcalcysts.Plain
filmswilldefinepulmonarycystsofE.granulosususuallyasroundedmassesofuniformdensitybutmay
misscystsinotherorgansunlessthereiscystwallcalcification(asoccursintheliver).
MRI,CT,andultrasoundrevealwelldefinedcystswiththickorthinwalls.Whenoldercystscontainalayer
ofhydatidsandthatisrichinaccumulatedprotoscolices,theseimagingmethodsmaydetectthisfluidlayer
ofdifferentdensity.However,themostpathognomonicfinding,ifdemonstrable,isthatofdaughtercysts
withinthelargercyst.
Thisfinding,likeeggshellormuralcalcificationonCT,isindicativeofE.granulosusinfectionandhelpsto
distinguishthecystfromcarcinomas,bacterialoramebicliverabscesses,orhemangiomas.Incontrast,
ultrasoundorCTofalveolarhydatidcystsrevealsindistinctsolidmasseswithcentralnecrosisandplaquelike
calcifications.
AspecificdiagnosisofE.granulosusinfectioncanbemadebytheexaminationofaspiratedfluidsforproto
scolicesorhooklets,butdiagnosticaspirationisnotusuallyrecommendedbecauseoftheriskoffluid
leakageresultingineitherdisseminationofinfectionoranaphylacticreactions.
Serodiagnosticassayscanbeuseful,althoughanegativetestdoesnotexcludethediagnosisof
echinococcosis.Cystsintheliverelicitpositiveantibodyresponsesin~90%ofcases,whereasupto50%of
individualswithcystsinthelungsareseronegative.Detectionofantibodytospecificechinococcalantigens
byimmunoblottinghasthehighestdegreeofspecificity.
Secondaryechinococcosisoccursasacomplicationofaprimarycyst.Theimportanceofoperativetechnique
duringtheremovaloftheprimarycystisemphasized,becausetheintraoperativediseminationoccursmost
frequentlyafterasurgicalperformance.Secondaryechinococcosisgeneratesbyasexual
regressivemetamorphosisoflarvalelementintrolarvalforms
(Q.64) Livepartofhydatidcyst?
(a)

Pericyst

(b)

Ectocyst

(c)

Endocyst

(d)

Noneoftheabove

YourResponse:

CorrectAnswer:

Exp:

Endocyst(RefBaileyandLove25th/pg.56)
EndocystisLivepartofhydatidcyst.
Etiology(Harrison,17theditionpage1338)
HydatiddiseaseisaparasiticinfectioncausedbyEchinococcusgranulosus.Thecystlodgesmost
commonlyintheliverandthelungs,respectively.
Morphologically,hydatidcystconsistsofthreelayersandhydatidfluid.Thefirstlayeristhepericystor
adventitiawhichisthehosttissueformedbythelungasareactiontotheforeignbody(parasite).The
othertwolayers,thelaminatedmembrane(externallayerofthecyst)andthegerminativelayer(inner
layerofthecyst),belongtotheparasite.Thecystfluidresembleswaterinappearancewhichmaycontain
daughtervesicles.
Theinnerlayerisalsocalledendocystwhichisthelivingpartwhichmeans,theyareinfectious.thatis
whyrupturedhydatidcystissuchaseriousproblem.thefluidinendocystisspilledinsurroundinghost
tissueanddaughtercystswilldevelopedinlarva.

Thecystsexistindifferentforms:intactorrupturedsingleormultiple,unilateralorbilateral,solely
locatedinthelungorconcomitantlyinotherorganlodgements(especiallyintheliver)
(Q.65) Informoletherconcentrationtechnique,whichlayercontaintheparasites?
(a)

Ether

(b)

Fecaldebris

(c)

Formalwater

(d)

Sediment

YourResponse:

CorrectAnswer:

Exp:

Sediment(RefMedicalparasitologybyRajeshKaryakarte1sted/pg.215;Fig.122)
Concentrationtechniquesinclude:
Sedimentationtechniques
Formoletherconcentrationtechnique

http://dbmci.examonair.com/Result/ShowAllQuestionInHtml.aspx?testid=3572

38/99

22/03/2016

DrBhatiaMedicalCoachingInstitute:OnlineTestPlatform
Formoldetergentconcentrationtechnique
Floatationtechniques
Sodiumchloridefloatationtechniques
Zincsulphatefloatationtechnique
Sugarfloatationtechnique
Thediagnosisofparasiticinfectionsinhumansischallengingandrequirestherecognitionofparasitestages
basedonsize,morphology,color,andmovement.Sizeandmorphologyarethemajordiagnostic
parameters,(Foreyt,1989).Aconcentration
procedureshouldbeperformedasaroutinepartofacompleteexaminationforparasites.Concentration
permitsthedetectionoforganismspresentinsmallnumbers:thesemaybemissedbyusingdirectwet
mounts.Organismsthatcangenerallybeidentified
usingaconcentrationprocedureinclude:helmintheggsandlarvae;cystsofGiardialamblia,Entamoeba
histolytica/Entamoebadispar,Entamoebacoli,Endolimaxnana,Blastocystishominis,Iodamoebabutschii;
oocystsofIsosporabelli.twotofivegrams
ofstoolwereusedforeachtest.Thestoolwaspouredontoadoublelayerofwettedgauzetakingspecial
caretoincludeanybloodormucus.Tenmillilitersofapreparationofeithernormalsalineor0.1NNaOH
weremixed.Themixturewasleftfor1hourbeforecentrifugationfor5minutesat2,000rpm.The
supernatantwaspouredoffandmixedwiththereagentinstep2again.Repeatcentrifugation,atthesame
rateandforthesametime,wasconducted.Thesupernatantwaspouredoffand10mlof0.1%formalin
wereadded.Themixturewasleftfor10minutes.Threemillilitersofetherwereadded;thetubewasclosed
andmixingtookplacefor30seconds.Theupperseparatedsupernatantwaspouredoff;thesedimentwas
usedforslidepreparation.
(Q.66) InHotairOven,forholdingperiodofonehourtemperaturerequiredis?
(a)

100oC

(b)

120oC

(c)

140oC

(d)

160oC

YourResponse:

CorrectAnswer:

Exp:

160oC.
(RefTextbookofMicrobiologybyAnanthanarayan8thed.25)
Hotairovenismostwidelyusedmethodofsterilizationbydryheat.Holdingperiodof160ocforonehouris
usedtosterilizethefollowing:
a.Glasswaresb.Forcepsc.Scissord.Scalpele.Glasssyringe
f.Swabsg.Liquidparaffinh.Dressingpowderi.Fats,oilsandgrease.
ThesporesofanontoxigenicstrainofClostridiumtetaniareusedasmicrobiologicaltestofdryheat
efficiency.

(Q.67) WhichofthefollowingisthepathogenresponsibleforblindnessinadvancedHIVinfections?
(a)

Cytomegalovirus

(b)

EpsteinBarrvirus

(c)

Fungus

(d)

Toxoplasma

YourResponse:

CorrectAnswer:

Exp:

Cytomegalovirus
ThepredominantcausesofmorbidityandmortalityamongpatientswithlatestageHIVinfectionare
opportunisticinfections.
Thesearedefinedassevereinfectionsinducedbyagentsthatrarelycausediseaseinimmunecompetent
individuals.OpportunisticinfectionsusuallydonotoccuruntilCD4Tcellcountsdropfromnormal(1000
cellspermicroliter)tolessthan200cellspermicroliter.
ThecommonopportunisticinfectionsinuntreatedAIDSpatientsarecausedbyprotozoa,fungi,bacteria,
andotherviruses.CoinfectionwithDNAvirusesarereportedtoleadtoenhancedexpressionofHIVincells
invitro.
HerpesvirusinfectionsarecommoninAIDSpatients,andCytomegalovirus(CMV)hasbeenshownto
produceaproteinthatactsasachemokinereceptorandisabletohelpHIVinfectcells.CMVretinitisisthe
mostcommonsevereocularcomplicationsofAIDS.

(Q.68) TheFcpieceofwhichImmunoglobulinfixesC1complements?
(a)

IgA

(b)

IgG

(c)

IgM

IgE

(d)
YourResponse:

CorrectAnswer:

http://dbmci.examonair.com/Result/ShowAllQuestionInHtml.aspx?testid=3572

39/99

22/03/2016

DrBhatiaMedicalCoachingInstitute:OnlineTestPlatform
Exp:

IgG
(RefH17th/Table30811pg.708)
Physical,Chemical,andBiologicPropertiesofHumanImmunoglobulins
Property

IgG

IgA

IgM

IgD

IgE

Usualmolecularform Monomer

Monomer,dimer Pentamer,
hexamer

Monomer Monomer

Otherchains

None

Jchain,SC

Jchain

None

None

Subclasses

G1,G2,G3,G4 A1,A2

None

None

None

Heavychainallotypes Gm(=30)

NoA1,A2m(2) None

None

None

Molecularmass,kDa 150

160,400

950,1150

175

190

Sedimentation
constant,Sw20

6.6S

7S,11S

19S

7S

8S

Carbohydrate
content,%

10

13

Serumlevelin
9.512.5
averageadult,mg/mL

1.52.6

0.71.7

0.04

0.0003

Percentageoftotal
serumIg

715

510

0.3

0.019

Serumhalflife,days 23

2.5

Synthesisrate,mg/kg 33
perday

65

0.4

0.016

Antibodyvalence

7585

2,4

10,12

Classicalcomplement +(G1,2?,3)
activation

++

Alternate

+(G4)

BindingcellsviaFc

Macrophages,
neutrophils,
largegranular
lymphocytes

Lymphocytes

Lymphocytes None

Biologicproperties

Placental
transfer,
secondaryAb
formost
antipathogen
responses

Secretory
PrimaryAb
immunoglobulin responses

complement
activation
Mastcells,
basophils,B
cells

Markerfor Allergy,
matureB antiparasite
cells
responses

(Q.69) TrueaboutC.jejuni:
(a)

Grampositivebacillus

(b)

Symptomsmayinitiallymimiccholecystitis

(c)

Mostoftenoccursseveraldaysafterconsumptionofundercookedchicken.

(d)

Penicillinindrugofchoice

YourResponse:

CorrectAnswer:

Exp:

Mostoftenoccursseveraldaysafterconsumptionofundercookedchicken.
Campylobacterspeciesaresmallmotile,nonsporeforming,commashaped,gramnegativebacilli,best
growninamicroaerophilicenvironmentat42C(107.6F).
GuillainBarreisarareneurologicalcomplicationofC.jejunigastroenteritis.
C.jejunigastroenteritisisusuallyselflimited;however,ifnecessary,erythromycinisthedrugofchoice.
Campylobacterinfectionmostoftenoccursseveraldays(upto7d)afterconsumptionofundercooked
chicken.
Symptomoffeverandabdominalpainmayinitiallymimicappendicitispseudoappendicitis.

(Q.70) HPVrelatedcervicalintraepithelialneoplasiacanbediagnosedbythepresenceofwhichofthefollowinghistologicfeatures?
(a)

Central,basophilicintranuclearcellularinclusions

(b)

CowdrytypeAintranuclearcellularinclusions

(c)

Enlargedmultinucleatedcells

(d)

Cytoplasmicvacuolizationandnuclearenlargementofcells

YourResponse:

CorrectAnswer:

Exp:

Cytoplasmicvacuolizationandnuclearenlargementofcells

HPVproducescharacteristiccytoplasmicvacuolizationandnuclearenlargementofsquamousepithelial
cells,referredtoaskoilocytosis.BothCowdrytypeAintranuclearinclusionsandenlargedmultinucleated
cellscanbeseenwithherpessimplexvirus(HSV)andvaricellazostervirus(VZV)infections;central,

http://dbmci.examonair.com/Result/ShowAllQuestionInHtml.aspx?testid=3572

40/99

22/03/2016

DrBhatiaMedicalCoachingInstitute:OnlineTestPlatform
basophilicintranuclearinclusionbodiesareseeninCMVinfections,whereasthepresenceofatypical
lymphocytesisseenspecificallyinEpsteinBarrvirus(EBV)infections.
(Q.71) Whichofthefollowingviralfamiliesisknowntobecausallyassociatedwithtumorformationinhealthyappearinghumanadults?
(a)

Flaviviruses

(b)

Papovaviruses

(c)

Paramyxoviruses

(d)

Polyomaviruses

YourResponse:

CorrectAnswer:

Exp:

Papovaviruses

HPVisamemberofthePapovavirusfamilyandiscausallyassociatedwithcervicalcancerinotherwise
healthyindividuals;HepatitisCvirusisamemberoftheFlavivirusesfamilyandcauseschronichepatitisand
inseverecasesisafactorinlivercancerdevelopment;Paramyxovirusesincludeagentssuchasrespiratory
syncytialvirusandmeaslesvirusandarenotassociatedwithcarcinomas;humanpolyomavirusesincludeBK
andJCviruses,whichhavebeenassociatedwithimmunocompromisedpatients,andtheirroleinformation
ofhumantumorsisstillunderinvestigation.
(Q.72) Hemagglutinin,(AntiAandAntiB)arewhichtypeofantibodies?
(a)

IgG

(b)

IgM

(c)

IgA

(d)

IgE

YourResponse:

CorrectAnswer:

Exp:

IgM

(RefH17th/pg.708709;Wintrobe'sclinicalhematology13th/pg.979)
Isohemagglutinins,substancesthatagglutinatetheredbloodcellsofothersofthesamespecies,arealso
foundinhumans.Thus,therearefourmainbloodgroups,whichdifferwithrespecttotwoantigens,Aand
B,intheredbloodcellsandtwoisohemagglutinins,antiAandantiB,intheserum.Thus,inhumans,typeO
hasneitherantigenbutbothagglutinins,typeAhasAantigenandantiB.
AntiAandantiBantibodiesareusuallyIgManddonotpassthroughtheplacenta.
(Q.73) Cellfusionisaninnovativetechniqueofpreparingspecificantibodies.Techniquestoinducethecellfusionincludethefollowing

except:
(a)

Byadheringsomeinactivatedvirusestosurfacemembrane

(b)

Byaddingcomponentpolyethyleneglycol

(c)

Bymildelectricshock

(d)

Byreducingtheviscosityofthemembrane

YourResponse:

CorrectAnswer:

Exp:

Byreducingtheviscosityofthemembrane.
Cellfusionisaninnovativetechniqueofpreparingspecificantibodies.
Techniquestoinducethecellfusioninclude:
Byadheringsomeinactivatedvirusestosurfacemembrane
Byaddingcomponentpolyethyleneglycol
Bymildelectricshock.

(Q.74) Whichofthefollowingprecautionsisadvisablewhenusinglovastatin?
(a)

Serumtransaminasemeasurements

(b)

Renalfunctionstudies

(c)

Acousticmeasurements

(d)

Monthlycompletebloodcounts

YourResponse:

CorrectAnswer:

Exp:

Serumtransaminasemeasurements
Lovastatinshouldnotbeusedinpatientswithsevereliverdisease.Withroutineuseoflovastatin,serum
transaminasevaluesmayrise,andinsuchpatients,thedrugmaybecontinuedonlywithgreatcaution.
Lovastatinhasalsobeenassociatedwithlenticularopacities,andslitlampstudiesshouldbedonebefore
and1yearafterthestartoftherapy.Thedrugisnottoxictotherenalsystem,andreportsofbonemarrow
depressionareveryrare.Thereisasmallincidenceofmyopathy,andlevelsofcreatininekinaseshouldbe
measuredwhenunexplainedmusclepainoccurs.Combinationwithcyclosporineorclofibratehasledto
myopathy.Thereisnodangerinusewithbileacidsequestrants.

(Q.75) Allopurinolshouldbeavoided,orreduceddosesgivenifwhichofthefollowingchemotherapeuticagentisbeinggiven?

http://dbmci.examonair.com/Result/ShowAllQuestionInHtml.aspx?testid=3572

41/99

22/03/2016

DrBhatiaMedicalCoachingInstitute:OnlineTestPlatform
(a)

Bleomycin

(b)

Cisplatin

(c)

Cyclophosphamide

(d)

Mercaptopurine

YourResponse:

CorrectAnswer:

Exp:

Mercaptopurine
Mercaptopurineandotherthiopurinesarepurineantimetabolitesthataremetabolicallyinactivated
(detoxified)byxanthineoxidase.Thispurinedegradationpathwayofmetabolismnotonlyleadsto
formationofuricacid,butalsoisimportanttoreducinghostcelltoxicitytothethiopurines.Thus,
concomitantuseofallopurinolincreasestheriskofhostcelltoxicityNotethatazathioprine(aninhibitorofB
andTlymphocyteproliferation,andtypicallyusedasanimmunosuppressant)ismetabolizedto
mercaptopurine.Asaresult,ittooisaninteractantwithallopurinol.Themetabolismoftheotherdrugs
listedisnotxanthineoxidasedependent.

(Q.76) DrugswhichinducecytochromeP450areallexcept:
(a)

Griseofulvin

(b)

Ketoconazole

(c)

Barbiturates

(d)

Phenytoin

YourResponse:

CorrectAnswer:

Exp:

Ketoconazole
Microsomalenzymeinhibitors
.Ketoconazole.Ciprofloxacin.Allopurinol.Omeprazole
.Erythromycin.Cimetidine.Phenylbutazone
Microsomalenzymeinduction
Anticonvulsant

Antitubercularsterid

Other

Phenobarbitone

Rifampicin

Chloralhydrate

Phenytoin

Isoniazid

Phenylbutazone

Glucocorticoids

Griseofulvin
DDT

(Q.77) DrugnotgiveninthecontinuationphaseofcategoryIIis:
(a)

Isoniazid

(b)

Rifampicin

(c)

Ethambutol

(d)

Pyrazinamide

YourResponse:

CorrectAnswer:

Exp:

Pyrazinamide
ThetreatmentregimernsfollowedinIndiaundertherevisednationaltuberculosiscontrolprogramme
(RNTCP1997)
TBCategory

Initialphase

Continuationphase

Totalduration

2H3R3Z3E3

4H3R3

II

2H3R3Z3E3S3+1H3R3Z3E3

5H3R3E3

III

2H3R3Z3

4H3R3

Thenumeralbeforeaphaseisthedurationofthatphaseinmonths.
Thenumeralinsubscript(e.gH3R3)isnumberofdosesofthatdrugperweek.Itthereisnosubscript
numeral,thenthedrugisgivendaily.
(Q.78) Whichofthefollowingdrugdoesnotreducespreload:
(a)

GTN

(b)

Captopril

(c)

Hydralazine

(d)

Nitroprusside

YourResponse:

CorrectAnswer:

Exp:

Hydralazine
HYDRALAZINEisadirectactingvasodilatorthatactsdirectlyonthearteriolesandreducestotalperipheral
resistance(afterloadreduction).Itincreasesheartrate,strokevolume,andcardiacoutput.

(Q.79) Chloroquineismosteffectiveagainstwhichstageofmalarialparasite:

http://dbmci.examonair.com/Result/ShowAllQuestionInHtml.aspx?testid=3572

42/99

22/03/2016

DrBhatiaMedicalCoachingInstitute:OnlineTestPlatform
(a)

Merozoites

(b)

Tissueschizonts

(c)

Bloodschizonts

(d)

Gametocytes

YourResponse:

CorrectAnswer:

Exp:

Bloodschizonts
CHLOROQUINEinhibitstheerythrocyticstageofPlasmodium.Itinterruptserythrocyticschizogony(blood
schizonts).
Chloroquineinhibitsthepolymerizationofhaemtoinerthaemozoin.Freehaemmonomersarethoughtto
catalyzeoxidativedamagetothePlasmodiumspp.trophozoite,thestagewhenhaemoglobincatabolismis
maximal.bloodschizontsaremoresensitiveforhisdrugthantissueschizonts.

(Q.80) SodiumreabsorptioninPCTispreventedby:
(a)

Thiazide

(b)

Triamterene

(c)

Mannitol

(d)

Ethacrynicacid

YourResponse:

CorrectAnswer:

Exp:

Mannitol
Mannitolisapharmacologicallyinertsubstancethatisfreelyfilteredofglomerulusandnotabsorbedin
appreciableamount.Principalsiteofactionisproximalconvolutedtubule.PassivereabsorptionofNa+and
CIisdecreasedduetoprogressivelyincreasingconcentrationgradient.

(Q.81) Whichofthefollowingistrueregardingbromocriptine:
(a)

Dopamineagonist

(b)

Dopamineantagonist

(c)

Increasesprolactinlevel

(d)

Galactopoietic

YourResponse: a
CorrectAnswer:

Exp:

Dopamineagonist
Actionsofbromocriptine
1.Dopamineagonist2.Antidyskinetic3.Growthhormone
suppressant
4.Antihyperprolactinemic5.Lactationinhibitor6.Neuroleptic
malignantsyndrome

(Q.82) Trueaboutroutesofdrugadministration:
(a)

80%bioavailabilitybyI.V.injection

(b)

I/Dinjectionproduceslocaltissuenecrosisandirritation

(c)

Inhalationproducesdelayedsystemicbioavailability

(d)

S/Crouteisusedinthetreatmentofhypotensiveshockpatient.

YourResponse:

CorrectAnswer:

Exp:

Intradermalinjectionproduceslocaltissuenecrosisandirritation
ThroughIV.route,thedrugdirectlyreachesthesystemiccirculationandeffectsareproducedimmediately
(greatvalueinemergency).
Bioavailabilityis100%.
Theintimaofveinsisinsensitiveanddruggetsdilutedwithblood,Therefore,evenhighlyirritantdrugscan
beinjectedIV.
Onlyaqueoussolutions(notsuspensions)canbeinjectedIV.andtherearenopreparationsforthisroute.
Doseofthedrugrequiredissmallest.
NotonlyIV.,butalltheparenteralroutessuchassubcutaneous(S.C.),intramuscular(I.M.)intravenous(IV.)
andintradermal(I.D.)havethefollowingdisadvantages:
Preparationandtechniqueneedstobesterile.
Itiscostlier.
Itisinvasive,painfulandcancauselocaltissuenecrosisandirritation.
Assistanceofanotherpersonismostlyneeded.
Therearechancesoflocaltissueinjury.
Itisingeneral,morerisky.
Ininhalation,absorptiontakesplacefromthevastsurfaceofalveoliimmediatesystemicbioavailability
andveryrapidaction.

http://dbmci.examonair.com/Result/ShowAllQuestionInHtml.aspx?testid=3572

43/99

22/03/2016

DrBhatiaMedicalCoachingInstitute:OnlineTestPlatform
Volatileliquidsandgasesaregivenbyinhalationforsystemicactione.g.,amylnitrate.
Whenadministrationisdiscontinued,thedrugdiffusesbackandisrapidlyeliminatedinexpiredair.
(Q.83) Carisoprodolactivationformswhichmetabolite:
(a)

Amphetamine

(b)

Meprobamate

(c)

Doxylamine

(d)

Dimethadione

YourResponse:

CorrectAnswer:

Exp:

.Meprobamate
Carisoprodolisarapidlyactingandisacentrallyactingmusclerelaxant.
Thedrugblockstheelectricalcommunicationamongthecentralnervesinbrain.
Thedrugstartsactingwithin20minutesandtheeffectslastfor26hours.
Itismetabolizedquicklybycytochromeenzymestomeprobamate,whichisanindependentmuscle
relaxant.
However,meprobamatehasabusepotential.Usualdoseofcarisoprodolis350mgdailyandsedationis
mainsideeffect.
Carisoprodolisbelievedtohavelessabusepotential.
Meprobamatewasusedinthepastasaselectiveantianxietydrugwithinsomnia.
However,nowadays,benzodiazepinesarethedrugsofchoiceforthiscondition.
Presently,though,carisoprodoltypeofdrugsareusedorallysometimesinorthopaedicconditions;theyare
oftengivenincombinationwithNSAIDsastopicalformulationsforbackpain,sprains,lumbagoetc.

(Q.84) Widmarksformulaisusedforestimationof:
(a)

Alcohol

(b)

Timesincedeath

(c)

Timessinceinjury

(d)

Consciousness

YourResponse:

CorrectAnswer:

Exp:

Alcohol.(RefReddy24thed.Pg.491)
WIDMARKFORMULA
Ittakesintoaccountthesizeandsexofthepersonandthetypeofalcoholic
liquorconsumed:
a=pre
Where:
a=wt.ofAlcoholinbodyingm
p=bodywtinkg.
e=conc.ofalcoholintheblood,inmg/kg
r=isaconstant0.6formen;0.5forwomen.
ForurineAnalvsis
A=%pr.qq=Alcoholicconc.inurineinmg/kg.

(Q.85) Incisedwoundresembles?
(a)

Splitlaceration

(b)

Stretchlacerations

(c)

Chopwounds

(d)

Internallacerations

YourResponse:

CorrectAnswer:

Exp:

Splitlaceration(RefParikhFMT6th/pg.4.10;4.13)
Incisedwound
Anincisedwoundisaninjurycausedbyaweaponwithasharpcuttingedgewhenitisdrawnacrossthe
skin.
Splitlacerationsthatareusuallyfoundinthepartsoverlyingboneswithoutmuchtissueinbetween(e.g.
scalp,face,hands,andlowerlegs)areincisedlikeorincisedlookingwounds.

(Q.86) Whichisnotacontentofblackgunpowder?
(a)

Sulphur

(b)

Charcoal

(c)

Sodiumnitrate

(d)

Potassiumnitrate

http://dbmci.examonair.com/Result/ShowAllQuestionInHtml.aspx?testid=3572

44/99

22/03/2016

DrBhatiaMedicalCoachingInstitute:OnlineTestPlatform
YourResponse:

CorrectAnswer:

Exp:

Sodiumnitrate(RefParikhFMT6th/pg.4.29)
Blackpowderorthegunpowder
Theblackpowderorthegunpowderconsistof:
Charcoal(15%),
Sulphur(10%),and
Postassium(75%).
GenerallyBlackpowderisusedinmuzzleloaders,blankcartridges,refilledcartridges,andcountrymade
cartridges.
Smokelesspowdercontainsnitrocellulose,orsometimesnitroglycerine.Smokelesspowdrisusedinall
moderncartridgeslikeofshotgun,revolver,rifle,etc.
Ablackpowdersubstitute,Pyrodex,isrecentlymanufacturedforuseinoldfirearms.

(Q.87) Whichchelatingagentisobtainedfromorganisms?
(a)

Desferrioxamine

(b)

BAL

(c)

EDTA

(d)

DTPA

YourResponse
c
:
CorrectAnswer A
:
Exp:

Desferrioxamine(RefKDT,Pharmacology,6thed.,867)
Desferrioxamine
Itisobtainedfromactinomycete.
1gmofdesferrioxamineiscapableofchelating85mgof
elementaliron.
Use:
AcuteIronpoisoning
Topreventhemochromatosisinpatientreceivingmultipleblood
transfusion
Adverseeffects:
HistaminereleaseRetinopathy,cataractHypertension
Deferiprone:
Orallyactingironchelator
Mainsideeffectsarearthropathyandbonedepression.

(Q.88) Chocolatecoloredpostmortemstainingisseenin?
(a)

Carbonmonoxidepoisoning

(b)

KCNpoisoning

(c)

Phosphoruspoisoning

(d)

Potassiumchloratepoisoning

YourResponse:

CorrectAnswer:

Exp:

Potassiumchlorate(RefParikhFMT,p3.10)
LIVORMORTIS(HYPOSTASIS,POSTMORTEMLIVIDITY,POSTMORTEMSUGGILLATIONS)
Lividityisadarkpurplediscolourationoftheskinresultingfromthegravitationalpoolingofbloodinthe
veinsandcapillarybedsofthedependentpartsofthebodyfollowingcessationofthecirculation.
Theprocessbeginsimmediatelyafterthecirculationstops,andinapersondyingslowlywithcirculatory
failure,itmaybepronouncedveryshortlyafterdeath.
Thisisduetothereleaseoffibrinolysins,especiallyfromsmallcalibrevessels,e.g.capillaries,andfrom
seroussurfaces,e.g.thepleura.Clotsmaypersistwhenthemassofclotistoolargetobeliquifiedbythe
fibrinolysinavailableatthesiteofclotformation.
Insomedeathsassociatedwithinfectionandcachexia,thisfibrinolyticeffectmayfailtodevelop,explaining
thepresenceofabundantclotintheheartandlargecalibrevessels.
Thus,incasesofsuddendeaththebloodremainsspontaneouslycoagulableonlyduringabriefperiod
immediatelyfollowingdeath;itthenbecomescompletelyfreefromfibrinogenandwillneveragainclot.
Thisincoagulabilityofthebloodisacommonplaceobservationatautopsy.
Thenormalcolourofareasofpostmortemlividityisacyanotichue,butthisdescriptionshouldnotbeused
sinceitismisleading.
Thedevelopmentoflividityistoovariabletoserveasausefulindicatorofthetimeofdeath.
Typically,lividityhasa

Purpleorreddishpurple
colouration.

Carbonmonoxidepoisoning

Cherryred.

Potassiumchlorate,nitrates,andanilinepoisoning(Incaseswhere
methaemoglobinisformedinthebloodduringlife)

Chocolatebrown.

http://dbmci.examonair.com/Result/ShowAllQuestionInHtml.aspx?testid=3572

45/99

22/03/2016

DrBhatiaMedicalCoachingInstitute:OnlineTestPlatform
Cyanidepoisoning

Pink,brightscarlet,and
violet.

Deathsfromexposuretocold

Brightpink

(Q.89) ThefollowingsectionsofIndianPenalCode(IPC)dealwithsexualoffencesexcept?
(a)

Sec375

(b)

Sec376

(c)

Sec377

(d)

Sec351

YourResponse:

CorrectAnswer:

Exp:

FEWIMPORTANTIPCsTOBEREMEMBERED:
Sec118IPC

Concealingdesigntocommitoffencepunishablewithdeath
orimprosonmentforlife.

Sec176,IPC

Omissiontogivenoticeorinformationtopublicservantby
personlegallyboundtogiveit.

Sec177IPC

Furnishingfalseinformation.

Sec182IPC

Falseinformationwithintenttocausepublicservanttouse
lawfulpowertotheinjuryofanotherperson.

Sec191IPC

Givingfalseevidence

Sec192IPC

Fabricatingfalseevidence.

Sec193IPC

Punishmentforfalseevidence

Sec197IPC

Issuingorsigningfalsecertificates.

Sec204IPC

Destructionofdocumenttopreventitsproductionas
evidence.

Sec284IPC

Negligentconductwithrespecttopoisonoussubstances.

Sec299IPC

Definesculpablehomicidenotamountingtomurder

Sec300IPC

Definesmurder

SEC302IPC

punishmentformurderdeath/life+fine

Sec304IPC

SECTION304AIPC:Causingdeathbynegligence2years/
fineorboth.
SECTION304BIPC:Dowrydeath.incaseofdeathofa
marriedwomanwithin7yearsofmarriage,duetoother
thannaturalcauses,withhistoryofcrueltyorharassment
priortodeath.thechargeisframedagainsttheaccused
undersection304BIPC.

Sec309IPC

Attempttocommitsuicide

Sec312316IPC

Dealswithcriminalabortion

Sec317IPC

Abandoningofchildbyparents

Sec318IPC

Concealmentofdeathofinfant

Sec313IPC

Causingmiscarriagewithoutwoman'sconsent

Sec320IPC

Definitionofgrievoushurt

SEC321IPC

Voluntarilycausinghurt

SEC325IPC

Grievoushurtispunishableundersection325IPC.

SEC338IPC:

Theactdonerashlyandnegligentlycausingminoroffenceor
grievoushurtwillbechargedundersection338IPC

Sec351IPC

Assault.

Sec354IPC

Molestation:Considerstheassaultorcriminalforceto
womanwiththeintentiontooutragehermodesty.This
offenseisconsideredlessseriousthanRape.

Sec362IPC

Abduction.

Sec375IPC

DefinitionofRape.

Sec376IPC

Section376A:PunishmentforRape
RestallclausesofSec376dealwithCustodialrape.
Section376B:dealswiththeintercourseofpublicservant
withwomaninhiscustody.
Section376C:dealswithintercourseofsuperintendentof
jail,remand,hostel,privatehousesetc.
Section376D:dealswithintercoursebyanymemberofthe

http://dbmci.examonair.com/Result/ShowAllQuestionInHtml.aspx?testid=3572

46/99

22/03/2016

DrBhatiaMedicalCoachingInstitute:OnlineTestPlatform
managementorstaffofahospitalwithanywomaninthat
hospital.
Sec377IPC

Unnaturaloffences

Sec497IPC

Adultery.

Sec509IPC

Word,gestureoractintendedtoinsultthemodestyofa
woman.Whoever,intendingtoinsultthemodestyofany
woman,uttersanyword,makesanysoundorgesture,or
exhibitsanyobject,intendingthatsuchwordorsoundshall
beheard,orthatsuchgestureorobjectshallbeseen,by
suchwoman,orintrudesupontheprivacyofsuchwoman,
shallbepunishedwithsimpleimprisonmentforaterm
whichmayextendtooneyear,orwithfine,orwithboth.

(Q.90) ThepostmortemfindingseeninSmotheringincludes
(a)

Bruisingininneraspectofupperlip

(b)

Fracturebodyofhyoid

(c)

Fractureofcricoid

(d)

Curvedmarksontheneck

YourResponse:

CorrectAnswer:

Exp:

Bruisingininneraspectofupperlip
Thelips,gums&tonguemayshowbruisingorlaceration
Slightbruisingmaybefoundinthemouthandnose,whichshouldbeconfirmedbymicroscopy.
Theasphyxialsignsandsymptomsaresevere,becausedeathusuallyresultsduetoslowasphyxiaandoften
thefatalperiodisthreetofiveminutes

(Q.91) AllarecharacteristicsofEntrytypeFirearmwoundexcept?
(a)

Smallinsize

(b)

Invertededges

(c)

Abrasioncollarpresent

(d)

Leadringabsent

YourResponse:

CorrectAnswer:

Exp:

Leadringabsent
REFParikhs6thEdPage4.47
ENTRYWOUND

EXITWOUND

Smallinsize(exceptwhenatcontactrangeskin Sharplydefinedoutwardlysplitedges
tornedbyblast)
InflatboneCleancutonoutersurface,chipped InflatboneBevelledandeverted.
inward.
Edgesinverted,exceptinfattypersonsmaybe
evertedduetoprotrusionoffat.

Edgeseverted.

Abrasioncollarpresent.

Absentexceptinshoredexitwoundswhen
irregularabrasionwillbeseen.

Greasecollarmayormaynotbepresent

Absent

Burning,Blackening,Singeingandtattooingat
appropriatedistances.

Absentexceptwhenanotheroverlappingexit
woundcausesitsoverlappingpattern

Clothingturnedinandcarriedinward.

Mayormaynotbeturnedout.

Tracknearwoundmaybebrightpinkdueto
carboxyhemoglobin.

Notusuallyso.

Leadringonradiologicalormicrochemical
examination.

Absent

(Q.92) Whenusingtheplanemirrortechniqueduringretinoscopy,whichoneofthefollowingstatementsstandswrong?
(a)

A'with'movementisneutralizedwithapluslens

(b)

An'against'movementisneutralizedwithaminuslens

(c)

A'with'movementalwaysindicateshypermetropia

(d)

An'against'movementalwaysindicatesmyopia

YourResponse:

CorrectAnswer:

Exp:

A'with'movementalwaysindicateshypermetropia.
Withtheplanemirrortechnique,pluslensisusedtoneutralizewithmovementandminuslensforagainst
movement.Whileanagainstmovementalwaysindicatesmyopia,awithmovementmaybeseenin
myopicpatientifthemyopiaislessthanthedioptricvalueoftheobserver'sworkingdistance(forexample
atadistanceof2/3m,awithmovementisseenifthemyopiaislessthan1.50D.Theneutralpointoccurs

http://dbmci.examonair.com/Result/ShowAllQuestionInHtml.aspx?testid=3572

47/99

22/03/2016

DrBhatiaMedicalCoachingInstitute:OnlineTestPlatform
whenthepatient'sfarpointcoincideswiththeobserver'snodalpoint..
(Q.93) CauseofEnlargedcornealnervesis?
(a)

Fuchscongenitaldystrophy

(b)

Congenitalglaucoma

(c)

Leprosy

(d)

Allofabove

YourResponse:

CorrectAnswer:

Exp:

Allofabove
Causesofcorneal
Anaesthesia(Neurotrophickeratopathy) Hyposthesia

Enlargednerves

Congenital

MnemonicFindoldcongenital
LINKtoRefsumsMEN

Familialdysautonomia(RileyDay
syndrome)
Anhidroticectodermaldysplasia

Absolute
glaucoma

Keratoconus
FuchsCornealdystrophy
anterior(Axenfeld
sign)
Oldage

Congenitalinsensitivitytopain

Congenitalglucoma

Acquired

Leprosy(Hensensdisease)

Alcoholblock,electrocoagulationor
sectionofgasserianganglionorsensory
rootoftrigeminalnervefortrigeminal
neuralgia

Icthyosis
Neurofibromatosis

Herpessimplex

Keratoconus

Herpeszoster

Trauma

Syphilitic(Luetic)neuropathy

Refsumsdisease

Leprosy

MENtypeIIB

Diabetes
Acousticneuroma
Abuseoftopicalanaesthetics

(Q.94) Inelectroretinogram(ERG),awave,largenegativewaveisgeneratedby:
(a)

Retinalpigmentepithelium

(b)

Rods&conesreceptoractivity.

(c)

Responseofamacrinecells

(d)

All

YourResponse:

CorrectAnswer:

Exp:

Rods&conesreceptoractivity.(RefBasak,Ophthalmology,2nded.,21)
ERG
Electroretinographymeansgrossrecordofelectricalpointial,changesintheretinaoften
stimulationwithlength.
InitialpositivedeflectionR1isfollowedbyR2isduetophotochemicalreactioninrods&
conesoutersegments.
Thisisfollowedbyalargenegativeawavewhichreflectsphotoreceptor(rods&cones)
activity.
Thenalargepositivebwavefollowsduetoresponseofbipolarcells.
Lastly,asmallpositivecwaveisgeneratedbyretinalpigmentepitheliallayer.
Ganglioncelllayer,nervefiberlayer&opticnerveplaynopart.
Durationofentireresponse<250ms.
USES:
Usefulindistinguishingretinalandopticnervedysfunctionfrommaculardiseases.
Assessesretinalfunctioninpresenceofopacityinmediainsiderosisbulbi.
Usefulin
Corticalandhystericalblindness,
Retinitispigmentosa,
Chorioretinitis,
Rodandconedysfunction,and
Toxicretinopathy.

http://dbmci.examonair.com/Result/ShowAllQuestionInHtml.aspx?testid=3572

48/99

22/03/2016

DrBhatiaMedicalCoachingInstitute:OnlineTestPlatform
(Q.95) Retinoscopyisusedto?
(a)

TodeterminedegreeofametropiA.

(b)

ToknowconditionofRefraction

(c)

Testingvisualacuity

(d)

Tomeasureastigmatismofanteriorcornealsurface.

YourResponse:

CorrectAnswer:

Exp:

Forestimatingconditionofrefraction(RefBasak,Ophthalmology,2nded.,72)
RETINOSCOPY
Mostpracticalmethodofestimatingconditionofrefractionobjectively,withaccommodation
atrest.
Principleistomakeeveryobservingeyeemmetropic,sothatemergingraysshouldform
parallelbeam.
Streakretinoscopyeasilydeterminesaxisofastigmatism,canbedoneinanyposition,children
andpreoperatively.
Cycloplegiamayberequiredchildren,howevernotadvisableundercycloplegia.
Itshouldpreferablybeconductedinadarkroom.
Sourceoflightshouldbebehindthepatient.
Inhypermetropia,emmetropia&myopia<1Dreflexmoves,reflexsamedirection.
Inmyopiaof1Dnomovement.
Inmyopia>1Dshadowmovesoppositedirection.

(Q.96) Indacryocystorhinostomy(DCR)lacrimalsacsaredirectlyopenedinto:
(a)

Superiornasalmeatus

(b)

Middlenasalmeatus

(c)

Inferiornasalmeatus

(d)

Nasolacrimalduct

YourResponse:

CorrectAnswer:

Exp:

Middlenasalmeatus

(RefBasaks,Opthalmology,2nded.,pg,282,336)
DACROCYSTORHINOSTOMY(DCR)issurgicalprocedureofchoiceinchronicdacryocystitis,inwhicha
communicationismadebetweenlacrimalsacandmiddlemeatusofnose.
Contraindications:oDNSoAtrophicrhinitisoCarcinomaoflacrimalgland
(Q.97) Amongtheretinalganglioncells,thefastestsignaltransmissiontothebrainandpromptresponsivenessforrapidchangesinvisual

imageisthefunctionof:
(a)

Wcells

(b)

Ycells

(c)

Xcells

(d)

Horizontalcells

YourResponse:

CorrectAnswer:

Exp:

Ycells.(RefAldersphysiologyofeye10thed.718)
Theconesrespondtobrightlightandmediatehighresolutionvisionandcolorvision.
Therodsrespondtodimlightandmediatelowerresolution,blackandwhite,nightvision.
Humanshavethreedifferenttypesofcones(trichromaticvision).
Whenlightfallsonareceptoritsendsaproportionalresponsesynapticallytobipolarcells,whichinturn
signaltheretinalganglioncells.Thereceptorsarealsocrosslinkedbyhorizontalcellsandamacrinecells,
whichmodifythesynapticsignalbeforetheganglioncells.
Intheretinalganglioncellstherearetwotypesofresponse,dependingonthereceptivefieldofthecell.In
ONcells,anincrementinlightintensityinthecenterofthereceptivefieldcausesthefiringratetoincrease.
InOFFcells,itmakesitdecrease.Beyondthissimpledifference,chromaticsensitivityandthetypeofspatial
summationalsodifferentiateganglioncells.
GanglionCells
Theganglioncellsarethelastneuronsinthechain,beforetheoutputleavestheeyeandgoestothebrain.
Thereareseveraltypesofganglioncell,andit'sbelievedeachtypeisusedforadifferentpurposes.TheW
cellisexcitedmainlybyrods,anditsfunctionistoperceivedirectionalmovementallovertheretina.Asan
objectmovesacrossthefield,thewcellsarestimulatedandpasstheinformationonviatheintegration
chain.TheXcellsareresponsibleformostcolorvision.Theyreceiveinputfromthecones;andtheyarealso
themostnumeroustypeofganglioncell.Theyprobablyaccountforthepinpointingoftheimageinits
preciselocationontheretina.TheYcellsarethelargestandleastnumeroustypeofganglioncell.They
appeartobededicatedtotheperceptionofchangesinlightintensityandperhapsveryrapidmovement
ofanimageacrossthevisualfield.Ganglioncells,unliketheotherintegratorneurons,dohaveaction
potentials,andevenwhenunstimulatedtheyfireatamoreorlessconstantrate.Itisthechangesinlight
intensityandtheshiftingoftheimageoverthefieldofvisionthatcausesachangeinthefiringratein
ganglioncells.These"on"or"off"responsesofindividualganglioncells,whichtaketheformofchangesof

http://dbmci.examonair.com/Result/ShowAllQuestionInHtml.aspx?testid=3572

49/99

22/03/2016

DrBhatiaMedicalCoachingInstitute:OnlineTestPlatform
frequencyoffiring,areinterpretedbythebrainasthefinaloutputofthesystem.Byswitchingtheganglion
cellsonoroffasdictatedbytheintegrationoftheimageintheretina,theeyehasanamazingabilityto
detectmotionofevenverysmallobjects.
(Q.98) TruestatementsaboutJacksonscrosscylinderareallEXCEPT:
(a)

Doesnotblurtheimagewhenplacedbeforeanemmetropiceye

(b)

Doesnotalterthesphericalequivalentofanametropiceye

(c)

Isusedtochecktheaxisofthecylindersubjectively

(d)

Isusedtocheckthepowerofthecylindersubjectively

YourResponse:

CorrectAnswer:

Exp:

Doesnotblurtheimagewhenplacedbeforeanemmetropiceye(RefKhurana3rded.683)
JACKSONSCROSSCYLINDER
Thecrosscylinderisusedforsubjectiverefinementofaxisandpowerofcylinderafterplacingthebest
availableestimateofrefractionbeforetheeye(retinoscopy,astigmaticdialtest,orpreviousrefraction).
Itisusedtochecktheaxisofthecylinderbeforeitspower.
Thepowerofthecylinderistwicethatofthesphereandofoppositesign.
A0.50Dcrosscylinderhasatotalcylindricalpowerof0.50D.A0.50Dcrosscylinderhasanetspherical
powerorsphericalequivalentpowerof0.A0.50Dcrosscylindercanbewrittenupas+0.25DS/0.50DCor
0.25DS/+0.50DC.Axisisnotspecified.
Placedbeforeanemmetropiceye,thecrosscylinderblurstheimage.
Placedbeforeanametropiceye,thecrosscylinderdoesnotalterthesphericalequivalent,butitwillenlarge
orcontracttheintervalofSturm,blurringorclarifyingtheimage,asitincreasesordecreasesthenet
astigmaticametropia.

(Q.99) Vagalstimulationcausesincreasein
(a)

Heartrate

(b)

RRintervalinEC

(c)

Cardiacoutput(COP)

(d)

Forceofcontraction

YourResponse: b
CorrectAnswer:

Exp:

RRinterval
RRintervalistheintervalbetweenthepeaksofRwaveof2consecutive
QRScomplexes.
Obviously,theRRintervalbearsaninverserelationshipwiththeheart
rate.
VagalstimulationcausesbradycardiaandthereforeanincreaseinRR
interval.
Vagalstimulationleadstobradycardia,hatiswhyinsymptomatic
bradycardiacases.
Patientsaretreatedbygivinginjectionatropine.
VagalstimulationleadstodecreaseCOP&ejectionfraction.

(Q.100) Mostcommonlocationofiriscoloboma:
(a)

12oClockposition

(b)

9oClockposition

(c)

6oClockposition

(d)

3oClockposition

YourResponse:

CorrectAnswer:

Exp:

6oClockposition
COLOBOMAOFTHEIRIS
Itisacongenital(presentsincebirth)defectoftheirisoftheeye.
Mostcommonlylocatedinferonasally.
Itisvisibleasahole,split,orcleftintheiris.
Colobomaoftheirismayappearasablack,roundholelocatedinoradjacenttotheiris(coloredportionof
theeye).
Itcanappearasablacknotchofvaryingdepthattheedgeofthepupil,givingthepupilanirregularshape.
Itcanalsoappearasasplitintheirisfromthepupiltotheedgeoftheiris.
Asmallcoloboma,especiallyifitisnotattachedtothepupil,mayallowasecondaryimagetofocusonthe
backoftheeye,causingaghostimage,blurredvision,ordecreasedvisualacuity.
Colobomamaybeassociatedwithhereditaryconditions,traumatotheeye,oreyesurgery.
Thedefectmayextendtotheretina,choroid,oropticnerve.
CommonCauses:

http://dbmci.examonair.com/Result/ShowAllQuestionInHtml.aspx?testid=3572

50/99

22/03/2016

DrBhatiaMedicalCoachingInstitute:OnlineTestPlatform
Cateyesyndrome
Trisomy13
Trisomy18
Marfanssyndrome
RubinsteinTaybisyndrome
SturgeWeberdisease
Basalcellnevussyndrome
Normalvariant(canbeinheritedasanautosomaldominanttrait)
Colobomasaregenerallydiagnosedat,orshortlyafterbirth.
Childhaswhatappearstobeaholeintheirisoranunusualshapedpupil.
Visionbecomesblurred,ordecreasedvisionisnoted.
(Q.101) Mostreliablelandmarkinotoscopyis:
(a)

Coneoflight

(b)

Lateralprocessofmalleus

(c)

Handleofmalleus

(d)

Umbo

Your
Response:
Correct
Answer:
Exp:

B
Lateralprocessofmalleus
Mostreliablelandmarkinotoscopyislateralprocess
ofmalleus.

(Q.102) Semontmaneuver(MSM)andmodifiedEpleyprocedure(MEP)areusedfortreating:
(a)

BPPV

(b)

Barotrauma

(c)

Menieresdisease

(d)

Traumaticnystagmus

YourResponse:

CorrectAnswer:

Exp:

BPPV(RefPLDhingraENT,5th/pg.51)
EpleyprocedureortheSemontmaneuver
Benignparoxysmalpositionalvertigo(BPPV)occursasaresultofdisplacedotoconia,whicharesmall
crystalsofcalciumcarbonate(alsoreferredtoas"otoliths"orcanaliths)thatarenormallyattachedto
theotolithicmembraneintheutricleoftheinnerear.
Becauseoftrauma,infection,orevensimpleaging,canalithscandetachfromtheutricleandcollectwithin
thesemicircularcanals.Headmovementsshiftthedetachedcanalithsandstimulatesensitivenervehairs
tosendfalsesignalstothebrain,causingdizzinessandothersymptoms.Thegoalofthecanalith
repositioningprocedure(CRP),aformofvestibularrehabilitationtherapy,istomovethedisplaced
canalithstostopthesefalsesignalsandthedebilitatingsymptomstheycancause.
SingleapplicationsoftheEpleyprocedureortheSemontmaneuverhavebeenreportedtorelieve
symptomsinthemajorityofpatients.
Repeatedtreatmentsessionsmaybenecessarybeforesymptomsresolve.

(Q.103) Carhartsnotchisseenat:
(a)

1000Hz

(b)

2000Hz

(c)

4000Hz

(d)

6000Hz

YourResponse:

CorrectAnswer:

Exp:

2000Hz(RefPLDhingra5th/Pg.99,458)
OtosclerosisandAudiometry
AirConduction:decreasedmoreforlowerfrequency.
BoneConduction:
Normal
Insamecasesthereisdipinboneconductioncurvemaximumat2000Hzandiscalled
asCarhartznotch.
Itisduetoinertialpropertyofstapes.
ItdisappearsafterStapedectomy.
FrequencyNotchinboneconduction
2000Hz=Otosclerosis
3000Hz=AcousticNeuroma

http://dbmci.examonair.com/Result/ShowAllQuestionInHtml.aspx?testid=3572

51/99

22/03/2016

DrBhatiaMedicalCoachingInstitute:OnlineTestPlatform
4000Hz=Noiseinduceddeafness
(Q.104) Vestibularfunctiontestedbyfollowingexcept:
(a)

Galvanicstimulation

(b)

Fistulatest

(c)

Impedanceaudiometry

(d)

Coldcalorictest

YourResponse:

CorrectAnswer:

Exp:

Impedanceaudiometry(RefPLDhingra5th/pg.28,47,49)
Galvanictestistheonlyvestibulartest,whichhelpsindifferentiatinganendorganlesionfromvestibular
nerve.
Fistulatest:
UsedtodetectfistulainbonywallofLabyrinth.
Increasingpressureinbonycanal;causesgiddinessandnystagmus;aspressureistransmittedtothe
labyrinththroughfistula(positiveFistulatest).
ImpedanceAudiometry:measuresresistanceinthemiddleearcavity.
Thecoldcalorictestisatestofthevestibuloocularreflexthatinvolvesirrigatingcoldorwarmwaterorair
intotheexternalauditorycanal

(Q.105) WhichofthefollowingisdifficulttovisualizeorexamineonIndirectlaryngoscopy?
(a)

Truevocalcord

(b)

Anteriorcommissure

(c)

Epiglottis

(d)

Falsevocalcord

YourResponse
d
:
CorrectAnswer: B
Exp:

Anteriorcommissure
(RefPLDhingra5th/pg.447;DiseasesofENTbyBhargava5thed.
270)
StructuresvisualizedbyIDLare:
Posteriorthirdofthetongue
Valleculae
Epiglottis
Aryepiglotticfolds
Inletoflarynx
Posteriorcommissure
Interioroflarynx
Truevocalcords
Posteriorpharyngealwall
MedialwallofPyriformfossa
Postcricoidregion.
Falsevocalcord

(Q.106) TobeyAyertestisusefulfordiagnosing:
(a)

Lateralsinusthrombosis

(b)

Cavernoussinusthrombosis

(c)

Serousotitismedia

(d)

Eustachiantubedefect

YourResponse:

CorrectAnswer:

Exp:

(Lateralsinusthrombosis)(RefPLDhingra5th/pg.94)
SIGMOIDORLATERALSINUSTHROMBOSIS
HecticPicketfencetypeoffeverwithrigors
Thisisduetosepticaemia,oftencoincidingwithreleaseofsepticemboliintobloodstream.Feverisirregularhaving
oneormorepeaksaday.Itisusuallyaccompaniedbychillsandrigors.Profusesweatingfollowsfalloftemperature.
Clinicalpictureresemblesmalariabutlacksregularity.
Inbetweentheboutsoffever,patientisalertwithasenseofwellbeing.Patientsreceivingantibioticsmaynotshow
thispicture.
Headache.Inearlystage,itmaybeduetoperisinusabscessandismild.Later,itmaybeseverewhenintracranial
pressurerisesduetovenousobstruction.
Progressiveanaemiaandemaciation.
Griesinger'ssign
Thisisduetothrombosisofmastoidemissaryvein.Oedemaappearsovertheposteriorpartofmastoid.
PapilloedemaItspresencedependsonobstructiontovenousreturn.Itisoftenseenwhenrightsinus(whichislarger

http://dbmci.examonair.com/Result/ShowAllQuestionInHtml.aspx?testid=3572

52/99

22/03/2016

DrBhatiaMedicalCoachingInstitute:OnlineTestPlatform
thanleft)isthrombosedorwhenclotextendstosuperiorsagittalsinus.Fundusmayshowblurringofdiscmargins,
retinalhemorrhagesordilatedreins.Funduschangesmaybeabsentwhencollateralcirculationisgood.
TobeyAyertestThisistorecordCSFpressurebymanometerandtoseetheeffectofmanualcompressionofoneor
bothjugularveins.
Compressionofveinonthethrombosedsideproducesnoeffectwhilecompressionofveinonhealthysideproduces
rapidriseinCSFpressurewhichwillbeequaltobilateralcompressionofjugularveins.
CroweBecktestPressureonjugularveinofhealthysideproducesengorgementofretinalveins(seenby
ophthalmoscopy)andsupraorbitalveins.Engorgementofveinssubsideonreleaseofpressure.
TendernessalongjugularveinThisisseenwhenthrombophlebitisextendsalongthejugularvein.Theremaybe
associatedenlargementandinflammationofjugularchainoflymphnodesandtorticollis
(Q.107) Griesingerssignisseenin:
(a)

Lateralsinusthrombosis

(b)

Meningitis

(c)

Brainabscess

(d)

Mastoiditis

YourResponse:

CorrectAnswer:

Exp:

Lateralsinusthrombosis(RefPLDhingra5th/pg.94)
Griesingerssignisoneoftheclassicalsignoflateralsinusthrombosischaracterizedbyedemaover
mastoidprocessduetothrombosisofmastoidemissaryvein.

(Q.108) WhichofthefollowingisFalseaboutcholesteatomais:
(a)

Lymphaticpermeation

(b)

Boneerosion

(c)

Epithelialdesquamatedsinus

(d)

Deafness

YourResponse:

CorrectAnswer:

Exp:

Lymphaticpermeation(RefCurrentOtolaryngologyChapter49;PLDhingra5th/pg.75)
Cholesteatoma
Congenital/primaryAcquired/secondary
Cholesteatomaisthepresenceofsquamousepitheliuminthemiddleear,mastoid,orepitympanum.The
mostcommonformofcholesteatomaistheacquiredvariety,whichisclassifiedasprimaryandsecondary
acquiredcholesteatoma.
Primaryacquiredcholesteatomaisthemostcommonofthesetypesandformsasaretractionofthe
tympanicmembrane.Inmostcases,theretractionoccursintheparsflaccida,althoughparstensa
retractionscanalsooccur.
Secondaryacquiredcholesteatomaformsasaresultofeithersquamousepithelialmigrationfromthe
tympanicmembraneorimplantationofsquamousepitheliumintothemiddleearduringsurgery,suchas
ventilationtubeplacementortympanoplasty.
Themostcommonlocationofcholesteatomainthemiddleearisintheareaaroundthestapes
superstructureandincuslongprocess.Thisareaisusuallydifficulttodissectbecauseofthepresenceof
thefacialnerveandossicularchain.

(Q.109) Inobstructiveazoospermia
(a)

FSH&LHBothincrease

(b)

FSH&LHBothnormal

(c)

FSHdecreasebutLHincreases

(d)

FSH&LHbothdecrease

YourResponse:

CorrectAnswer:

Exp:

FSH&LHBothnormal
FSH&LHBothnormal.BothFSH&LHareamarkeroftesticularfunction,whichtendstobenormalinmostcases
ofobstructiveazoospermiaso,thechancesoffindingnormalFSH&LHlevelsaremaximum.Infactsomestudies
alsopinpointthesamefact.

Formostinfertilemen,thesemenanalysisistheonlytestwhichneedstobedoneafterall,theonlyjobofaman

http://dbmci.examonair.com/Result/ShowAllQuestionInHtml.aspx?testid=3572

53/99

22/03/2016

DrBhatiaMedicalCoachingInstitute:OnlineTestPlatform
istoprovidespermtofertilizetheegg!Formenwithalowspermcount,thereisnoneedtodoanyothertests,
sincethesedonotprovideanyusefulinformation.
However,manydoctorsstilldobloodtestsformeasuringthelevelsofkeyreproductivehormones,suchas
prolactin,FSH,LHandtestosterone.
Thesearejustawasteoftimeandmoneysincetheyprovidenousefulinformationanddonotalterthetreatment
plan.
Formenwithazoospermia(zerospermcount),additionalbloodtestsmaybeuseful.
TheserumFSH(folliclestimulatinghormone)leveltestisausefuloneforassessingtesticularfunction.
Ifthereasonfortheazoospermiaistesticularfailure,thenthisisreflectedinaraisedFSHlevel.
Thisisbecause,inthesepatients,thetestisalsofailstoproduceahormonecalledinhibin(whichnormally
suppressesFSHlevelstotheirnormalrange).
AhighFSHlevelisusuallydiagnosticofprimarytesticularfailure,aconditioninwhichtheseminiferoustubulesin
thetestesdonotproducespermnormally,becausetheyaredamaged.
Thistestisdonebyaradioimmunoassayorchemiluminescentassay,andsinceitisasophisticatedtest,itisbest
doneinaspecializedlaboratory.
Abnormaltestresultsshouldberepeatedandrecheckedforconfirmation.
TheotherreasonforahighFSHlevelinsomemenistheconsumptionofclomiphene(amedicineoftenprescribed
fortheempirictreatmentofoligospermia).
Thisiswhythetestshouldbedoneonlywhennomedicationisbeingtaken.
WhileahighFSHlevelisdiagnosticoftesticularfailure,anormalFSHlevelprovidesnousefulinformation.
Thus,menwithcompletetesticularfailuremayalsohavenormalFSHlevels.
WhileahighFSHlevelsuggestsprimarytesticularfailure,itcannotdifferentiatebetweenpartialtesticularfailure
andcompletetesticularfailure.
ThismeansthatevenmenwithveryhighFSHlevelscanhaveoccasionalareasofspermproductionintheirtestes,
andthesetesticularspermcanbeusedforTESAICSI(testicularspermaspirationandintracytoplasmicsperm
injection)treatment.
Rarely,theFSHlevelmaybelow.AlowFSHlevelisfoundinpatientswithhypogonadotropichypogonadism.
Hypogonadotropichypogonadismisanuncommon(buttreatable!)causeofazoospermia.
AlongwithanFSHleveltest,mostdoctorsalsodoaLH(luteinizinghormone)leveltest,whichprovidesmostlythe
sameinformation.
Atestosteroneleveltestprovidesinformationonwhetherornotthetestesareproducingadequateamountsof
themalehormone,namely,testosterone.
Mostinfertilemenhavenormaltestosteronelevels,becausethecompartmentfortestosteroneproductionis
separatefromthecompartmentwhichproducessperm,andisusuallyintactininfertilemen.
Alowtestosteronelevelcausesadecreasedlibidoandthiscanbetreatedbytestosteronereplacementtherapyin
theformoftabletsorinjections.
Ofcourse,thistherapywillnotincreasethespermcount.
Formenwithazoospermiaanderectiledysfunction,measuringtheprolactinlevelwillhelptodetectmenwho
havehyperprolactinemia(highprolactinlevels).
Thoughthisisarareproblem,theycanbeeffectivelytreatedwithmedicaltherapywithbromocriptineandthe
resultsareverygratifying.
(RefMaleInfertilityTests.BeyondtheSemenAnalysis(Page2):HowtoHaveaBaby:OvercomingInfertilitybyDr.
AniruddhaMalpani,MDandDr.AnjaliMalpani,MD)
(Q.110) WhichofthefollowingisnotincludedintheWHOMemberStatesmandated"official"globalpublichealthcampaigns?
(a)

24thMarchWorldTBday

(b)

7thAprilWorldHealthday

(c)

1stDecemberWorldAIDSday

(d)

4thFebruaryWorldCancerday

YourResponse:

CorrectAnswer:

Exp:

4thFebruaryWorldCancerday
Globalpublichealthcampaignsoffergreatpotentialtoraiseawarenessandunderstandingabouthealth
issuesandmobilizesupportforaction,fromthelocalcommunitytotheinternationalstage.Therearemany
worlddaysobservedthroughouttheyearrelatedtospecifichealthissuesorconditionsfromAlzheimer's
tozoonoses.However,WHOfocusesparticularattentiononthesevendaysandoneweekthatWHO
MemberStateshavemandatedas"official"globalpublichealthcampaigns.Theseare:
WorldTBDay

24March

WorldHealthDay

7April

WorldImmunizationWeek

lastweekofApril

WorldMalariaDay

25April

WorldNoTobaccoDay

31May

WorldBloodDonorDay

14June

WorldHepatitisDay

28July

WorldAIDSDay

1December

OtherCommemorativedaysare
WorldCancerDay

http://dbmci.examonair.com/Result/ShowAllQuestionInHtml.aspx?testid=3572

February4

54/99

22/03/2016

DrBhatiaMedicalCoachingInstitute:OnlineTestPlatform
InternationalDayofZeroTolerancetoFemale
GenitalMutilation
NationalScienceDay(India)
Zerodiscriminationday
InternationalWomen'sDay
InternationalDayfortheEliminationofRacial
Discrimination
WorldDownSyndromeDay
WorldWaterDay
WorldTuberculosisDay
Worldautismawarenessday
WorldHemophiliaDay
EarthDay
WorldMalariaDay
WorldThallasemiaDay
WorldHypertensionDay
WorldMilkDay
WorldEnvironmentDay
WorlddayagainstChildLabour
WorldBloodDonorDay
WorldPopulationDay
NelsonMandelaInternationalDay
WorldHepatitisDay
WorldBreastfeedingWeek
InternationalYouthDay
WorldHumanitarianDay
WorldContraceptionDay
WorldBreastCancerAwarenessMonth
WorldVegetarianDay
WorldMentalHealthDay
WorldFoodDay
UnitedNationsDay
WorldDiabetesDay
HumanRightsDay

February6

February28
1March
March8
March21

March21(RememberasDownsis21Trisomy(3rd
month)i;e21/03)
March22
March24
April2
April17
April22
April25
May8
May17
June1
June5
June12
June14
July11
July18
July28
August01to07
August12
August19
September26
October
October1
October10
October16
October24
November14
December10

(Q.111) MaximumabsorptionofHCO3occursis:
(a)

PCT

(b)

DCT

(c)

CT

(d)

ALH

YourResponse:

CorrectAnswer:

Exp:

PCT
About70%offilteredHCO3occuratPCT.70%offilteredwater.NaClabsorptionoccuratPCT.100%of
filteredglucoseandaminoacidoccuratPCT

(Q.112) WHOdefinitionofLowvision?
(a)

Visionmorethan6/60andlessthan6/18inbettereye

(b)

Visionmorethan6/60andlessthan3/60inbesteye

http://dbmci.examonair.com/Result/ShowAllQuestionInHtml.aspx?testid=3572

55/99

22/03/2016

DrBhatiaMedicalCoachingInstitute:OnlineTestPlatform
(c)

Visionmorethan3/60andlessthan6/18inbettereye

(d)

Visionmorethan3/60andlessthan3/18inbesteye

YourResponse:

CorrectAnswer:

Exp:

Visionmorethan6/60andlessthan6/18inbettereye.(RefParkstextbookofPSM19thed.360)
VISION2020:
TheRighttoSightisthecommonagendalaunchedbytheWHOandTaskForceofInternational
organizationtocombatblindness,as80%ofglobalblindnessisavoidable.
FiveconditionsthathavebeenidentifiedasimmediateprioritieswithinVISION2020areCataract,
trachoma,Onchocerciasis,childhoodblindnessandrefractiveerrorsandlowvision.
TheSAFEstrategyhasbeendevelopedtocombatTRACHOMAandisbeingappliedinaffectedareas.
ItisexpectedthatthroughSAFEstrategyitwillbepossibletoeliminatetrachomabyyear2020.

CATEGORIESOFVISUAL MAXIMUMVISUALACUITYLESSTHAN
IMPAIRMENT
Lowvision
Blindness

MINIMUMVISUALACUITYEQUAL
TOORBETTERTHAN

6/18

6/60

6/60

3/60

3/60(fingercountingat3metres)

1/60(fingercountingat3metres)

1/60(fingercountingat3metres)

Lightperception

Nolightperecption

(Q.113) Accordingtotheconceptoftheicebergphenomenonofdisease,thewaterlinerepresentsthedemarcationbetweenwhichofthe

following?
(a)

Undiagnosedandundiagnosed

(b)

Symptomaticandpresymptomatic

(c)

Carrierandsufferers

(d)

Apparentandinapparent

YourResponse:

CorrectAnswer:

Exp:

Apparentandinapparent.(RefParkstextbookofPSM19thed.36)
ICEBERGOFDISEASE
Aconceptcloselyrelatedtothespectrumofdiseaseistheconceptoftheicebergphenomenonofthe
disease.
Accordingtothisconcept,diseaseinacommunitymaybecomparedwithaniceberg.
Thefloatingtipoftheicebergrepresentswhatthephysicianseesinthecommunityi.e.clinicalcases.
Thevastsubmergedportionofthehiddenmassofdisease,i.e.latent,inapparent,presymptomatic,and
underdiagnosedcasesandcarriersinthecommunity.
Thewaterlinerepresentsthedemarcationbetweenapparentandinapparent.

(Q.114) ThepowerfulconceptofKAPispracticedinIndiaforwhichofthefollowing?
(a)

HIV/AIDS

(b)

Familyplanning

(c)

Cancer

(d)

Malaria

YourResponse:

CorrectAnswer:

Exp:

Familyplanning(RefParkstextbookofPSM19thed.406)
Theconceptthateventuallybecameunmetneedforfamilyplanningwasfirstexploredin1960s,when
datafromsurveysofcontraceptiveknowledge,attitudeandpractices(KAP)showedagapbetweensome
womensreproductiveintensionandtheircontraceptivebehavior.
ThetermthatcametopopularusedescribingthisgroupwasKAPgap.
Oneofthefirstpublisheduseofthetermunmetneedappearedin1977.
In1978,basedonWorldFertilitySurveydatafromfiveAsiancountries,CharlesWestoffpublishedfirst
comparativeestimatesofunmetforlimitingbirths.
Unmetneedisdefinedonthebasisofwomensresponsetosurveyquestions.
Accordingtothenationalfamilyhealthsurvey2(19981999),about16%ofcurrentlymarriedwomenin
Indiahaveanunmetneedforfamilyplanning.
Unmetneedforfamilyplanningishighest(27%)amongwomenbelowage20yearsandisalmostentirely
forspacingbirthsratherthanforlimitingthebirth.
Itrangesfrom7%inPunjabto25%inUttarPradeshandBihar,amongthemajorstatesofIndia.

(Q.115) Standarderroris
(a)

S.D/n

http://dbmci.examonair.com/Result/ShowAllQuestionInHtml.aspx?testid=3572

56/99

22/03/2016

DrBhatiaMedicalCoachingInstitute:OnlineTestPlatform
(b)

S.Dxn

(c)

SD/2n

(d)

S.D/squarerootofn

YourResponse:

CorrectAnswer:

Exp:

S.D/squarerootofn)(RefAnintroductiontomedicalstatisticsbyBland2nded.60)
Standarderror
Thestandarderrorofamethodofmeasurementorestimationistheestimatedstandarddeviationofthe
errorinthatmethod.
Thestandarderrorofthemeanofasamplefromapopulationisthestandarddeviationofthesampling
distributionofthemean,andmaybeestimatedbytheformula:

Where
isanestimateofthestandarddeviationofthepopulation,andnisthesize(numberofitems)ofthe
sample.

(Q.116) WhichofthefollowingareincludedinWHOGlobalTargetsinNutrition2025?
(a)

40%reductioninthenumberofchildrenunder5whoarestunted

(b)

30%reductioninlowbirthweight

(c)

noincreaseinchildhoodoverweight

(d)

Allofabove

YourResponse:

CorrectAnswer:

Exp:

Allofabove
REF:http://www.who.int/nutrition/topics/nutrition_globaltargets2025/en/index.html
WHO'sMemberStateshaveendorsedglobaltargetsforimprovingmaternal,infantandyoungchild
nutritionandarecommittedtomonitoringprogress.Thetargetsarevitalforidentifyingpriorityareas
foractionandcatalysingglobalchange.
40%reductioninthenumberofchildrenunder5whoarestunted
50%reductionofanaemiainwomenreproductiveage
30%reductioninlowbirthweight
noincreaseinchildhoodoverweight
increasetherateofexclusivebreastfeedinginthefirst6monthsuptoatleast50%
reduceandmaintainchildhoodwastingtolessthan5%

(Q.117) AboutASHA(AccreditedSocialHealthActivist)trueisallthefollowingEXCEPT:
(a)

Theyarepreferablyfemales.

(b)

ThereisoneASHAworkerper1000population.

(c)

TheyareSkilledbirthattendant

(d)

Providesprimarymedicalcareforminorailments

YourResponse:

CorrectAnswer:

Exp:

TheyareSkilledbirthattendant
ACCREDITEDSOCIALHEALTHACTIVIST(ASHA)
TheGovernmentofIndiahasdecidedtolaunchaNationalRuralHealthMission(NRHM)toaddressthe
healthneedsofruralpopulation,especiallythevulnerablesectionsofsociety.
TheSubcentreisthemostperipherallevelofcontactwiththecommunityunderthepublichealth
infrastructure.Thiscaterstoapopulationnormof5000,butiseffectivelyservingmuchlargerpopulation
attheSubcentrelevel,especiallyinEAGStates.Withonlyabout50%MPWbeingavailableintheseStates,
theANMisheavilyoverworked,whichimpactsoutreachservicesinruralareas.
CurrentlyAnganwadiWorkers(AWWs)undertheIntegratedChildDevelopmentScheme(ICDS)are
engagedinorganizingsupplementarynutritionprogrammesandothersupportiveactivities.Thevery
natureofherjobresponsibilities(withemphasisonsupplementaryfeedingandpreschooleducation)does
notallowhertotakeuptheresponsibilityofachangeagentonhealthinavillage.
Thusanewbandofcommunitybasedfunctionaries,namedasAccreditedSocialHealthActivist(ASHA)is
proposedtofillthisvoid.ASHAwillbethefirstportofcallforanyhealthrelateddemandsofdeprived
sectionsofthepopulation,especiallywomenandchildren,whofinditdifficulttoaccesshealthservices.

(Q.118) About,Aedesaegypti,falseis:
(a) Recognizedbywhitemarkingsonlegsandamarkingoftheformofalyreonthethorax.
(b) ThemalesofAedesspeciesofmosquitoesbitehumansandanimals.

http://dbmci.examonair.com/Result/ShowAllQuestionInHtml.aspx?testid=3572

57/99

22/03/2016

DrBhatiaMedicalCoachingInstitute:OnlineTestPlatform
(c) Theireggscansurviveforverylongperiodsinadrystate
(d) Arenighttimebitersandpreferbreadinginflowingwater
YourResponse:

CorrectAnswer:

Exp:

Arenighttimebitersandpreferbreadinginflowingwater
Aedesaegyptiisamosquitothatcanspreadthedenguefever,Chikungunyaandyellowfeverviruses,and
otherdiseasesaswell.Themosquitocanberecognizedbywhitemarkingsonlegsandamarkingofthe
formofalyreonthethorax.
Theyarerecurrentdaytimebitersandbreedinstatgnantwater.
Aedesaegyptiissmallincomparisontoothers,usuallybetweenthreetofourmillimetresinlength
discountingleglength.
Itistotallyblackapartfromwhite'spots'onthebodyandheadregionsandwhiteringsonthelegs.The
thoraxisdecoratedwithawhite'Lyre'shapeofwhichthe'chords'aretwodullyellowlines.
Itswingsaretranslucentandborderedwithscales.
Themalesofallspeciesofmosquitoesdonotbitehumansoranimalsofanyspecies,theyliveonfruit.
Onlythefemalebitesforbloodwhichsheneedstomaturehereggs.
Theeggsofmostspeciesarelaidtogetherinaraftform,butAedeslayshereggsseparatelythusallowing
themtospreadoverlargesurfacesofwaterifconditionspermit,thiswaytheeggsstandabetterchanceof
survival.
Theeggscansurviveforverylongperiodsinadrystate,oftenformorethanayear.
Sincetheviruscanbepassedfromadulttoeggthenthevirustooisguaranteedsurvivaluntilthenext
summerandheavyrains.
Aedesaegyptibitesduringdaytimeandhencedaytimemosquitobiteisthemainreasonfortransmission.
Aedesisverydomesticated,asmuchasyourpetdogorcat,mostmosquitoescanliveinforestedareasa
longwayfromhumansandliveonanimalblood,notAedes.

(Q.119) Trueis:
(a)

Aldrineisanorganophosphate.

(b)

Tik20isaArylgroupcontainingphosphorouscompounD.

(c)

ACholineesteraseactivatorisusefulinRxofDichlorovaspoisoning

(d)

ThepopularBaygonSparyContainsanOrganophosphorouscompound

YourResponse:

CorrectAnswer:

Exp:

ACholineesteraseactivatorisusefulinRxofDichlorovaspoisoning.
AcarbamatePropoxuristhecontentofBAYGON.
Propoxur,Metformin,PhenforminareArylgroupcontainingphosphorouscompounds.
DichlorovasisaOrganophosphateandpralidoxime(2PAM),acholinesterasereactivatorisusefulinRx.
PAMisusefulforreversalofnicotinicsignsandsymptomsduetoorganophosphates,nervegasesor
unknownanticholineestrases

(Q.120) Integratedmanagementofchildhoodillness(IMCI)targetsthefollowingdiseasesEXCEPT
(a)

Malaria

(b)

Neonataltetanus

(c)

Otitismedia

(d)

HIV

YourResponse:

CorrectAnswer:

Exp:

Neonataltetanus(RefParkstextbookofPSM19thed.372;462)
Integratedmanagementofchildhoodillness(IMCI)
Theextentofchildhoodmorbidityandmortalitycausedbydiarrhoea,Acuterespiratoryinfection,malaria,
measlesandmalnutritionissubstantial.
IthasbeendecidedtolaunchIMCIinfourselecteddistrictseachinUttaranchal,MadhyaPradesh,Orissa,
Rajasthan,Maharashtra,Gujarat,Delhi,HaryanaandTamilNadu.
IMNCIwillbeimplementedinaphasedmannerthroughoutthecountry.
IntheIndiancontextthisstrategyisquitepertinentconsideringtherecentevidencefromNFHSIIreport
highlightingthatARI(17percent),diarrhoea(13percent),fever(27percent)andundernutrition(43per
cent)werethecommonestmorbiditiesobservedinthechildrenagedunder3years.
Coverageofmeaslesvaccinationinchildrenbetween1223monthsisalsolow.
ThemajorhighlightsoftheIndianadaptationare:
Inclusionof07daysageintheprogramme.
Incorporatingnationalguidelinesonmalaria,anaemia,vitaminAsupplementationandimmunization
schedule.
Trainingofthehealthpersonnelbeginswithsickyounginfantsupto2months;
Proportionoftrainingtimedevotedtosickyoungitandsickchildisalmostequal;and
isskillbased.

http://dbmci.examonair.com/Result/ShowAllQuestionInHtml.aspx?testid=3572

58/99

22/03/2016

DrBhatiaMedicalCoachingInstitute:OnlineTestPlatform
(Q.121) AllofthefollowingareinheritedinXlinkedrecessivemannerexcept?
(a)

Hemophilia

(b)

Ducchenemusculardystrophy

(c)

G6PDdeficiency

(d)

Retinitispigmentosa

YourResponse:

CorrectAnswer:

Exp:

Retinitispigmentosa
AgainrememberthatthisisaveryveryfrequentlyaskedQuestiontypeinAIIMSexams.byjustbyhearting
themodeofinheritanceofvariousdiseasesitispossibletoscore1MCQcorrectineachandeveryAIIMS
exam.
IamprovidingyouwithanOvercomprehensivelistofvariousdiseasesandtheirmodeofinheritance.
Rememberingallofthemisimpracticalbutreviseitagainandagainandyouwillnotmissoutonitinthe
exam.
AUTOSOMALDOMINANT

AUTOSOMALRECESSIVE

Hyperlipoprotinemias(excepttypeI)

Abetalipoprotinemia

Achondroplasia

Cysticfibrosis

Neurofibromatosis

Hirshsprungdisease

Adenomatouspolyposiscoli

Phenylketonuria

Marfanssyndrome

Albinism

Retinoblastoma

Taysachsdisease

HereditarySpherocytosis

Alkaptonuria

Tuberoussclerosis

Galactosemia

Myotonicdystrophy

MapleSyrupurinedisease

VonWillebranddisease

Betathallasemia

Osteogenesisimperfecta

Homocystinuria

Acuteintermittentporphyria

Lysosomalstoragedisorders

Holtoramsyndrome

Alpha1Antitrypsindeficiency

MEN

Wilsonsdisease

VHLsyndrome

Hemochromatosis

Wilmstumour

Glycogenstoragedisorders

Otosclerosis

Sicklecelldisease

BRCA1andBRCAIIBreastcancers

Congenitaladrenalhyperplasia

Gardner,Peutzzeigersyndrome

Freidrichsataxia

Retinitispigmentosa

Spinalmuscularatrophy

ALS

HereditaryvitaminDResistantrickets

OslerWeberRendudisease

Kartagenersyndrome

Huntingtonschorea

Turcotssyndrome

ABOBloodgroupsystem

Polycysticliverdisease

Crouzonsyndrome

Fanconisyndrome

Myotonicdystrophy

Gauchersyndrome

Hypophosphatemicrickets(FGF23mutation)

FAXB(Fanconianemia,Ataxiatelangiectasia,Xeroderma
pigmentosum,Bloomsyndrome)

Hemiplegicmigraine

SwisstypeAgammaglobulinemia

CharcotMarieToothdisease

Nonketotichyperglycinemia

Pigmentaryglaucoma

Acrodermatitisenteropathica

Hypokalemicperiodicparalysis
Diaphysialaclasia
XLinkedRecessiveInheritance

XLinkedDominantInheritance

HemophilaAandB

VitaminDResistantrickets

Duchennemusculardystrophy

FamilialHypophosphatemia(PHEX)

G6PDdeficiency

BloodgroupXg

Hydrocephalus

Rettssyndrome

Retinitispigmentosa(mostseverelesscommon OroFacioDigitalsyndrome
form)
BrutonsAgammaglobulinemia

Incontinentiapigmentii

Hunterssyndrome
ChronicGranulomatousdisease
WiskottAldrichsyndrome
Diabetesinsipidus
LeschNyhansyndrome

http://dbmci.examonair.com/Result/ShowAllQuestionInHtml.aspx?testid=3572

59/99

22/03/2016

DrBhatiaMedicalCoachingInstitute:OnlineTestPlatform
FragileXsyndrome
Dentsdisease
Kallmansyndrome
Androgeninsensitivitysyndrome
Fabrysdisease
Alportsyndrome
MITOCHONDRIALINHERITANCE(1QuestionwasaskedinAIIMSNOV09)
Mitochondrialmyopathy
LebersHereditaryOpticNeuropathy(LHON)
MELAS:MetabolicEncephalopathy+LacticAcidosis+Strokelikeepisodes
NARP:Neuropathy+Ataxia+RetinitisPigmentosa
Pearsonsyndrome:BonemarrowandPancreaticfailure
KearnsSayresyndrome:Ophthalmoplegia+Pigmentarydegenerationofretina+Cardiomyopathy
MERRFsyndrome:MyoclonicEpilepsy+RaggedRedFibres
MMC:MaternallyinheritedMyopathyandCardiomyopathy
CEOP:ProgressiveExternalOphthalmoplegia
ADMIMY:AutosomalDominantinheritedMitochondrialMyopathyandMitochondrialDeletion
Leighsdisease:movementdisorder+regression+respiratorydyskinesia
NavajoNeurohepatopathyandMDS
(Q.122) Decisionanalysesoftenincludeapatientsutilitiesinthedeterminationofthebestdecision.Theseutilitiesmeasure?
(a)

Whetherapatientfavorsonedecisionoveranother

(b)

Whetheraphysicianfavorsonedecisionoveranother

(c)

Thedifferencebetweenapatientsdecisionandthephysiciansdecision

(d)

Therelativevalueapatientplacesonaparticularoutcome

YourResponse:

CorrectAnswer:

Exp:

Therelativevalueapatientplacesonaparticularoutcome
Indecisionanalysis,utilitiesrefertotherelativevaluesplacedonvariousoutcomes.Forexample,perfect
healthmightbeassignedautilityof100,anddeathassignedoneof0.What,then,wouldtheutilitybefor
lifewithmoderatebackpain?Withcarefulquestioning,onefindsthatmostpatientsplaceahighervalue
onlifewithdisabilitythanwouldbeanticipated.

(Q.123) Theauthorsofastudystatethatcarefulautopsiesshowthat60percentofallpersonswhodiehaveevidenceofrecentorprevious

pulmonaryembolismandconcludethatpulmonaryembolismistheleadingcauseofdeathinIndiA.Possiblereasonsfordisagreeing
withtheconclusionincludeallthefollowingexcept:
(a)

Confounding

(b)

Selectionbias

(c)

Randomerror

(d)

Leadtimebias

YourResponse:

CorrectAnswer:

Exp:

Leadtimebias
Theexactproportionofdeathsduetopulmonaryembolismisnotknownandcouldonlybedeterminedby
astudyofarandomsampleofallpersonswhodied.Thiswouldavoidtheproblemofselectionbias(only
certainpersonsundergoautopsies).Thestudyshouldbelargeenoughtoavoidrandomerror(perhapsthe
citedstudyfoundemboliinthreeoffiveautopsies),andtheinvestigatorsshouldhavecarefulandprecise
definitionsofpulmonaryembolismtoavoidoverdiagnosisofthecondition.Theassociationbetween
pulmonaryembolismanddeathmaybeduetoconfoundingifsomeotherfactorcausesboth.Thus,
pulmonaryembolism,thoughpresent,maynotbethecauseofdeath.Leadtimebiasreferstoanapparent
increaseinsurvivalamongpersonswhosediseaseisdetectedbyscreening.Forexample,5yearsurvivalof
cancerpatientsidentifiedonscreeningmightappeartobeprolongedsimplyasaresultofstartingtocount
thesurvivaltimeearlierinthecourseofdisease.

(Q.124) Whichstatementistrueconcerningmeasuresofcentraltendency?
(a) Ifmoreoutlyingobservationsaresmallerthantherestofthevalues,thedataareskewedtotheright
(b) Ifmoreoutlyingobservationislargerthantherestofthevalues,themedianwillbesmallerthanthemean
(c) Ifthedataareskewedtotheleft,themeanislargerthanthemedian
(d) Themedianismoresensitivethanthemeanstoextremeobservation
YourResponse:

CorrectAnswer:

Exp:

Ifmoreoutlyingobservationislargerthantherestofthevalues,themedianwillbesmallerthanthe
mean

http://dbmci.examonair.com/Result/ShowAllQuestionInHtml.aspx?testid=3572

60/99

22/03/2016

DrBhatiaMedicalCoachingInstitute:OnlineTestPlatform
Whenmoreoutlyingvaluesarelargerthantherestofthevalues,thedataaresaidtobeskewedtothe
right,andthemedianissmallerthanthemean.Ifmoreoutlyingvaluesaresmallerthantherest,thedata
aresaidtobeskewedtotheleft,andthemedianislargerthanthemean.Themedianismorerobustthan
themeanbecauseitislesssensitivetoextremeobservationsandisamoreappropriatemeasureofcentral
tendencywhenextremevaluesarepartofthedataset.Themode,themean,andthemedianareallthe
sameforasymmetricaldistributionofdata.Thesedistributionscanbeillustratedasfollows:

(Q.125) AccordingtorevisedRCHprogramme,CommunityHealthCentreisreferredas:
(a)

Firstreferralunit

(b)

IIFRU

(c)

IIIFRU

(d)

ItisnotincludedinaccordingtonewRCHprogramme

YourResponse:

CorrectAnswer:

Exp:

Firstreferralunit(RefParkstextbookofPSM19thed.367)
ThemainhighlightsofRCHprogrammeare:
Theprogrammeintegratesallinterventionsoffertilityregulation,maternalandchildhealthwith
reproductivehealthforbothmenandwomen.
Theservicestobeprovidedareclientoriented,demanddriven,highqualityandbasedonneedsof
communitythroughdecentralisedparticipatoryplanningandtargetfreeapproach.,
Theprogrammeenvisagesupgradationoftheleveloffacilitiesforprovidingvariousinterventionsand
qualityofcare.TheFirstReferralUnits(FRUs)beingsetupatsubdistrictlevelprovidecomprehensive
emergencyobstetricandnewborncare.SimilarlyRCHfacilitiesatPHCsaresubstantiallyupgraded.
Facilitiesofobstetriccare,MTPandIUDinsertioninthePHCslevelareimproved.IUDinsertionfacilitiesare
alsoavailableatsubcentres.
SpecialistfacilitiesforSTDandRTIareavailableinalldistricthospitalsandinafairnumberofsubdistric
hospitals.
RCHprogrammeisbasedonadifferentialapproach.Inputsinallthedistrictshavenotbeenkeptuniform.
Whilethecarecomponentsarethesameforalldistricts,theweakerdistrictsgetmoresupportand
sophisticatedfacilitiesareproposedforrelativelyadvanceddistricts.Onthebasisofcrudebirthrateand
femaleliteracyrate,allthedistrictshavebeendividedintothreecategories.CategoryAhaving58districts,
categoryBhaving184districtsandcategoryChaving265districts.Allthedistrictswerecoveredina
phasedmannerover.aperiodofthreeyears.Theprogrammewasformallylaunchedon15thOctober
1997.

(Q.126) TrueregardingNRHMis?
(a) SeekstorevitalizelocalhealthtraditionsandmainstreamAYUSHintothepublichealthsystem
(b) InfantMortalityRatereducedto30/1000livebirthsisanationalleveloutcomeindicator
(c) Itseeksdecentralizationofprogrammesfordistrictmanagementofhealth.
(d) Allofabove
YourResponse:

CorrectAnswer:

Exp:

Ans.D.Allofabove
NATIONALRURALHEALTHMISSION
THEVISION
TheNationalRuralHealthMission(200512)seekstoprovideeffectivehealthcaretoruralpopulation
throughoutthecountrywithspecialfocuson18states,whichhaveweakpublichealthindicatorsand/or
weakinfrastructure.
These18StatesareArunachalPradesh,Assam,Bihar,Chhattisgarh,HimachalPradesh,Jharkhand,Jammu
&Kashmir,Manipur,Mizoram,Meghalaya,MadhyaPradesh,Nagaland,Orissa,Rajasthan,Sikkim,Tripura,
UttaranchalandUttarPradesh.
TheMissionisanarticulationofthecommitmentoftheGovernmenttoraisepublicspendingonHealth
from0.9%ofGDPto23%ofGDP.Itaimstoundertakearchitecturalcorrectionofthehealthsystemto
enableittoeffectivelyhandleincreasedallocationsaspromisedundertheNationalCommonMinimum
Programmeandpromotepoliciesthatstrengthenpublichealthmanagementandservicedeliveryinthe
country.
Ithasasitskeycomponentsprovisionofafemalehealthactivistineachvillage;avillagehealthplan
preparedthroughalocalteamheadedbytheHealth&SanitationCommitteeofthePanchayat;
strengtheningoftheruralhospitalforeffectivecurativecareandmademeasurableandaccountabletothe
communitythroughIndianPublicHealthStandards(IPHS);andintegrationofverticalHealth&Family
WelfareProgrammesandFundsforoptimalutilizationoffundsandinfrastructureandstrengthening
deliveryofprimaryhealthcare.
ItseekstorevitalizelocalhealthtraditionsandmainstreamAYUSHintothepublichealthsystem.
Itaimsateffectiveintegrationofhealthconcernswithdeterminantsofhealthlikesanitation&hygiene,
nutrition,andsafedrinkingwaterthroughaDistrictPlanforHealth.
Itseeksdecentralizationofprogrammesfordistrictmanagementofhealth.
ItseekstoaddresstheinterStateandinterdistrictdisparities,especiallyamongthe18highfocusStates,

http://dbmci.examonair.com/Result/ShowAllQuestionInHtml.aspx?testid=3572

61/99

22/03/2016

DrBhatiaMedicalCoachingInstitute:OnlineTestPlatform
includingunmetneedsforpublichealthinfrastructure.
Itshalldefinetimeboundgoalsandreportpubliclyontheirprogress.
Itseekstoimproveaccessofruralpeople,especiallypoorwomenandchildren,toequitable,affordable,
accountableandeffectiveprimaryhealthcare.
GOALS
ReductioninInfantMortalityRate(IMR)andMaternalMortalityRatio(MMR)
UniversalaccesstopublichealthservicessuchasWomenshealth,childhealth,water,sanitation&
hygiene,immunization,andNutrition.
Preventionandcontrolofcommunicableandnoncommunicablediseases,includinglocallyendemic
diseases
Accesstointegratedcomprehensiveprimaryhealthcare
Populationstabilization,genderanddemographicbalance.
RevitalizelocalhealthtraditionsandmainstreamAYUSH
Promotionofhealthylifestyles
OUTCOMES
NationalLevel:
InfantMortalityRatereducedto30/1000livebirths
MaternalMortalityRatioreducedto100/100,000
TotalFertilityRatereducedto2.1
Malariamortalityreductionrate50%upto2010,additional10%by2012
KalaAzarmortalityreductionrate:100%by2010andsustainingeliminationuntil2012
Filaria/Microfilariareductionrate:70%by2010,80%by2012andeliminationby2015
Denguemortalityreductionrate:50%by2010andsustainingatthatleveluntil2012
JapaneseEncephalitismortalityreductionrate:50%by2010andsustainingatthatleveluntil2012
CataractOperation:increasingto46lakhsperyearuntil2012.
Leprosyprevalencerate:reducefrom1.8/10,000in2005tolessthan1/10,000thereafter
TuberculosisDOTSservices:Maintain85%cureratethroughentireMissionperiod.
UpgradingCommunityHealthCenterstoIndianPublicHealthStandards
IncreaseutilizationofFirstReferralUnitsfromlessthan20%to75%Engaging250,000femaleAccredited
SocialHealthActivists(ASHAs)in10States.
CommunityLevel:
Availabilityoftrainedcommunitylevelworkeratvillagelevel,withadrugkitforgenericailments
HealthDayatAnganwadilevelonafixedday/monthforprovisionofimmunization,ante/postnatal
checkupsandservicesrelatedtomother&childhealthcare,includingnutrition.
AvailabilityofgenericdrugsforcommonailmentsatSubcentreandhospitallevel
Goodhospitalcarethroughassuredavailabilityofdoctors,drugsandqualityservicesatPHC/CHClevel
ImprovedaccesstoUniversalImmunizationthroughinductionofAutoDisabledSyringes,alternatevaccine
deliveryandimprovedmobilizationservicesundertheprogramme

(Q.127) AllofthefollowingareECGfeaturesofhypokalemiaexcept?
(a)

ProlongedPRinterval

(b)

ProminentUwave

(c)

STdepression

(d)

PeakedTwaves

YourResponse:

CorrectAnswer:

Exp:

PeakedTwaves
Thisisanextremelyhighyieldtopichavingatleast1questionineveryexam.
HereisalistofalmostallimportantconditionscausingspecificECGabnormalities.

ECGABNORMALITIES:
DIAGNOSTICECGFEATURES

CONDITION

QTshortening(Earliestfeature)

TherapeuticDigitalization

IncreasedPRinterval
Slowingofheartrate
Decreasedamplitude/invertedTwave
STdepression
ProlongedPRinterval,

Digitalistoxicity

VPCs,VT,VF,AVblock
Sinoatrialblock,
Ventricularbigeminy/trigeminy
Flattening/inversionofTwave

Earlyhypokalemia

STdepression
ProminentUwave
ProlongedQUinterval

http://dbmci.examonair.com/Result/ShowAllQuestionInHtml.aspx?testid=3572

62/99

22/03/2016

DrBhatiaMedicalCoachingInstitute:OnlineTestPlatform
ProlongedPR

Severehypokalemia

DecreasedvoltageandwideningofQRS
Increasedriskofventriculararrhythmias
RarelySAblock
PeakedTwaves

Hyperkalemia

LossofPwave
SINEWAVEPATTERN
WideningofQRSandprolongedPR
Sinusarrest
Cardiacarrestasystole
Ventricularfibrillation
Bradycardia

Hypercalcemia

AVblock
ShortQT
ProlongedQT

Hypocalcemia

WidespreadSTelevationwithupwardconcavity STAGEIAcutePericarditis
involving23standardlimbleadsandV2V6
ReciprocaldepressiononlyinaVRand
sometimesinV1
PRdepressions/oAtrialinvolvement
STsegmentreturnstonormalafterseveraldays STAGEIIAcutePericarditis
OnlythenorevenafteritTwaveinversion
occurs

STAGEIIIAcutePericarditis

WeekstomonthsafteritECGreturnstonormal STAGEIVAcutePericarditis
JwaveorOsbornwave

Hypothermia

HighvoltagetallQRSwithshortPRinterval

Pompesdisease

CVATpattern:

IntracranialblledsespSAH.

MarkedQTprolongationwithdeepwideT
waveinversions
TotalElectricalAlternans

PericardialEffusionwithcardiactamponade
d/tswingingmotionofheartinthepericardial
cavityatafrequencyexactlyhalfthehrtrate

ReplarisationAlternans

Signofelectricalinstabilityandmaysuggest
impendingventriculartachyarrythmias

PeakedTwaves(d/thyperkalemia)

ECGTriadofchronivrenalfailure

LongQTd/tStsegment
lengthening(hypocalcemia)
Leftventricularhypertrophy(d/tsystemic
hypertension)
Rightaxisdeviation

TetralogyofFallot

RightventricularhypertrophywithinvertedT
wavesandPPulmonale
NormalPwavewithshortPR

WPWsyndrome

ASlurredthickenedinitialupstrokeofQRS
DELTAwave
MainQRSdeflection
SecondarySTandTwavechanges
Twavesuppression/inversion(most
characteristicchange)

Imipramine(TCA)toxicity

ProlongedQRS>100ms
RwavegreaterthanSwave
PolymorphicVTpreceededbymarkedQT
prolongation>0.6sec

Torsadesdepointes

BizzareandmultiformQRS
TransientSTdepression

Classicalangina

TransientStelevationwithoutQwave

Noninfarctiontransmural
ischemia(Prinzmetalsangina)

ParadoxicalTwavenormalization
STdepressionorTwaveinversionwithoutQ
wave

NSTEMI

STelevation

HyperacutephaseMI

TallwidenedTwaves
PathologicalQwave

FullyevolvedMI

STelevation
InvertedsymmetricTwave
PathologicalQwaveonly

Oldinfarct

(Q.128) Extinctionphenomenaisseenin?

http://dbmci.examonair.com/Result/ShowAllQuestionInHtml.aspx?testid=3572

63/99

22/03/2016

DrBhatiaMedicalCoachingInstitute:OnlineTestPlatform
(a)

Alzheimerdisease

(b)

Hemispatialneglect

(c)

Antonionsyndrome

(d)

Wernickesencephalopathy

YourResponse:

CorrectAnswer:

Exp:

Hemispatialneglect
RefHarrison,17thedition,page167
HemispatialNeglect
Adaptiveorientationtosignificanteventswithintheextrapersonalspaceissubservedbyalargescale
networkcontainingthreemajorcorticalcomponents.Thecingulatecortexprovidesaccesstoalimbic
motivationalmappingoftheextrapersonalspace,theposteriorparietalcortextoasensorimotor
representationofsalientextrapersonalevents,andthefrontaleyefieldstomotorstrategiesforattentional
behaviors.Subcorticalcomponentsofthisnetworkincludethestriatumandthethalamus.Contralesional
hemispatialneglectrepresentsoneoutcomeofdamagetoanyofthecorticalorsubcorticalcomponentsof
thisnetwork.
Extinctionisaneurologicaldisorderthatimpairstheabilitytoperceivemultiplestimuliofthesametype
simultaneously.Extinctionisusuallycausedbydamageresultinginlesionsononesideofthebrain.Those
whoareaffectedbyextinctionhavealackofawarenessinthecontralesionalsideofspace(towardsthe
leftsidespacefollowingarightlesion)andalossofexploratorysearchandotheractionsnormallydirected
towardthatside.

(Q.129) A36yearoldmalecomestohisprimarycarephysiciancomplainingof3daysofworseningheadache,leftfrontalfacialpain,and

yellownasaldischarge.Thepatientreportsthathehashadnasalstuffinessandcoryzaforabout5days.Pastmedicalhistoryis
notableonlyforseasonalrhinitis.Thephysicalexaminationisnotableforatemperatureof37.9C(100.2F)andtendernessto
palpationovertheleftmaxillarysinus.Theoropharynxhasnoexudates,andthereisnolymphadenopathy.Whichofthefollowing
isthemostappropriatenextintervention?
(a)

Aspirationofthemaxillarysinus

(b)

Nasalfluticasone

(c)

Oralamoxicillin

(d)

Serumantineutrophilcytoplasmicantibodies

YourResponse:

CorrectAnswer:

Exp:

Nasalfluticasone
Thehistoryandclinicalpresentationsuggeststhatpatientisacaseofseasonalrhinitis(hayfever).
Thegoalofrhinitistreatmentistopreventorreducethesymptomscausedbyinflammationofaffected
tissue.
Avoidtheallergenasmuchaspossible
Intranasalcorticosteroidsareprefferedtreatmentegnasalfluticasone
Oralantibioticsareofnouseasitisallergicnotinfectiveetiology
Decongestents
Yogaanddeepbreathingexcercisesaregood
Aspirationshouldbeperformedwhenthereisknownopacificationofasinusandempiricaltherapyhasnot
beeneffectiveorthepatientisatriskofopportunisticinfection

(Q.130) Trueaboutasthma?
(a)

IncreasedFRCanddecreasedresidualvolume

(b)

IncreasedFRCandincreasedresidualvolume

(c)

ReducedFRCandreducedresidualvolume

(d)

DecreasedFRCandincreasedresidualvolume

YourResponse:

CorrectAnswer:

Exp:

increasedFRCincreasedresidualvolume
Asthma
PATHOPHYSIOLOGY
Thepathophysiologichallmarkofasthmaisareductioninairwaydiameterbroughtaboutbycontractionof
smoothmuscle,vascularcongestion,edemaofthebronchialwall,andthick,tenacioussecretions.Thenet
resultisanincreaseinairwayresistance,adecreaseinforcedexpiratoryvolumesandflowrates,
hyperinflationofthelungsandthorax,increasedworkofbreathing,alterationsinrespiratorymuscle
function,changesinelasticrecoil,abnormaldistributionofbothventilationandpulmonarybloodflowwith
mismatchedratios,andalteredarterialbloodgasconcentrations.Thus,althoughasthmaisconsideredto
beprimarilyadiseaseofairways,virtuallyallaspectsofpulmonaryfunctionarecompromisedduringan
acuteattack.
Inaddition,inverysymptomaticpatientstherefrequentlyiselectrocardiographicevidenceofright
ventricularhypertrophyandpulmonaryhypertension.Whenapatientpresentsfortherapy,the1sforced
expiratoryvolume(FEV1)orpeakexpiratoryflowrate(PEFR)istypically_40%ofpredicted.Inkeepingwith
thealterationsinmechanics,theassociatedairtrappingissubstantial.
Inacutelyillpatients,residualvolumefrequentlyapproaches400%ofnormal,whilefunctionalresidual

http://dbmci.examonair.com/Result/ShowAllQuestionInHtml.aspx?testid=3572

64/99

22/03/2016

DrBhatiaMedicalCoachingInstitute:OnlineTestPlatform
capacitydoubles.
(Q.131) GasusedtomeasurediffusionLungCapacity?
(a)

CO

(b)

NO

(c)

CO2

(d)

O2

YourResponse:

CorrectAnswer:

Exp:

CO(RefH17th/pg.1592)
DiffusingCapacity
Theabilityofgastodiffuseacrossthealveolarcapillarymembraneisordinarilyassessedbythediffusing
capacityofthelungforcarbonmonoxide(DLCO).
Inthistest,asmallconcentrationofcarbonmonoxide(0.3%)isinhaled,usuallyinasinglebreaththatis
heldfor~10s.Duringthebreathhold,thecarbonmonoxideisdilutedbythegasalreadypresentinthe
alveoliandisalsotakenupbyhemoglobinastheerythrocytescoursethroughthepulmonarycapillary
system.Theconcentraonofcarbonmonoxideisthenmeasuredinthegasexhaledaerthebreathhold,
andDLCOiscalculatedasthequantyofcarbonmonoxideabsorbedperminutepermmHgpressure
gradientfromthealveolitothepulmonarycapillaries.ThevalueobtainedforDLCOdependsonthe
alveolarcapillarysurfaceareaavailableforgasexchangeandonthepulmonarycapillarybloodvolume.In
addion,thethicknessofthealveolarcapillarymembrane,thedegreeofV
/Q
mismatching,andthe
paentshemoglobinlevelwillaectthemeasurement.Becauseofthiseectofhemoglobinlevelson
DLCO,themeasuredDLCOisfrequentlycorrectedtotakethepaentshemoglobinlevelintoaccount.The
valueforDLCO,ideallycorrectedforhemoglobin,canthenbecomparedwithapredictedvalue,based
eitheronage,height,andgenderoronthealveolarvolume(VA)atwhichthevaluewasobtained.

(Q.132) Apatientisevaluatedintheemergencydepartmentforperipheralcyanosis.Whichofthefollowingisnotapotentialetiology?
(a)

Coldexposure

(b)

Deepvenousthrombosis

(c)

Methemoglobinemia

(d)

Peripheralvasculardisease

YourResponse:

CorrectAnswer:

Exp:

Methemoglobinemia
Thelogicsaysanycausewhichreducesoxygencarryingtoalloverthebodywillbecausingcentral
cyanosisforexamplemethemoglobinemiabecauseitisinbloodcompetingwithnormalhemoglobinto
attatchwithoxygenandwhenmethhemoglobingetstheoxygenitdoesntleaveitsointheendbody
doesntgetoxygen=centralcyanosis
Incontrasttocentralcyanosisperipheralcyanosisoccurswhenthereissomelocalorpartofourbody
getslessoxygenforexampleobstructioninoneparticularbloodvesseletcsohereinthequestiongiven
allotherareperipheralcauseexceptmethemoglobin
CausesofCyanosis
CentralCyanosis
Decreasedarterialoxygensaturation
Decreasedatmosphericpressurehighaltitude
Impairedpulmonaryfunction
Alveolarhypoventilation
Unevenrelationshipsbetweenpulmonaryventilationandperfusion(perfusionofhypoventilatedalveoli)
Impairedoxygendiffusion
Anatomicshunts
Certaintypesofcongenitalheartdisease
Pulmonaryarteriovenousfistulas
Multiplesmallintrapulmonaryshunts
Hemoglobinwithlowaffinityforoxygen
Hemoglobinabnormalities
Methemoglobinemiahereditary,acquired
Sulfhemoglobinemaacquired
Carboxyhemoglobinemia(nottruecyanosis)
PeripheralCyanosis
Reducedcardiacoutput
Coldexposure
Redistributionofbloodflowfromextremities
Arterialobstruction
Venousobstruction

http://dbmci.examonair.com/Result/ShowAllQuestionInHtml.aspx?testid=3572

65/99

22/03/2016

DrBhatiaMedicalCoachingInstitute:OnlineTestPlatform
(Q.133) A35yearoldmaleisseenintheclinicforevaluationofinfertility.Hehasneverfatheredanychildren,andafter2yearsof

unprotectedintercoursehiswifehasnotachievedpregnancy.Spermanalysisshowsanormalnumberofsperm,buttheyare
immotile.Pastmedicalhistoryisnotableforrecurrentsinopulmonaryinfections,andthepatientrecentlywastoldthathehas
bronchiectasis.Chestradiographyislikelytoshowwhichofthefollowing?
(a)

Bihilarlymphadenopathy

(b)

Bilateralupperlobeinfiltrates

(c)

Normalfindings

(d)

Situsinversus

YourResponse:

CorrectAnswer:

Exp:

Situsinversus
Thisparticularcasesuggesttwoclosediagnosisoronecangetconfusedbetweencysticfibrosisand
kartagenersyndromethekeytotheanswerisNormalspermcountandimmotilespermswhichismore
specificallyfoundinkartargenersyndrome.

Thecombinationofinfertilityandrecurrentsinopulmonaryinfectionsshouldpromptconsiderationofan
underlyingdisorderofciliarydysfunctionthatistermedprimaryciliarydyskinesia.Thesedisordersaccount
forapproximately5to10%ofcasesofbronchiectasis.Anumberofdeficiencieshavebeendescribed,
includingmalfunctionofdyneinarms,radialspokes,andmicrotubules.Allorgansystemsthatrequire
ciliaryfunctionareaffected.Thelungsrelyonciliatobeatrespiratorysecretionsproximallyand
subsequentlytoremoveinspiredparticles,especiallybacteria.Intheabsenceofthisnormalhostdefense,
recurrentbacterialrespiratoryinfectionsoccurandcanleadtobronchiectasis.Otitismediaandsinusitis
arecommonforthesamereason.Inthegenitourinarytract,spermrequireciliatoprovidemotilityso
spermsareimmotilealthoughspermcountisnormal.
Kartagenerssyndromeisacombinationofsinusitis,bronchiectasis,andsitusinversusandimmotile
spermswithnormalspermcounts.Itaccountsforapproximately50%ofpatientswithprimaryciliary
dyskinesia.
Cysticfibrosisisassociatedwithinfertilityandbilateralupperlobeinfiltrates,itcausesadecreasednumber
ofspermorabsentspermonanalysisbecauseofthecongenitalabsenceofthevasdeferens.butherein
thisparticularcasethespermcountisnormal
Sarcoidosis,whichisoftenassociatedwithbihilaradenopathy,isnotgenerallyacauseofinfertility.Water
balloonshapedheartisfoundinthosewithpericardialeffusions,whichonewouldnotexpectinthis
patient.

(Q.134) TrueaboutEnteralnutritionis?
(a) Resultsinreductionofinfectiouscomplicationsincriticallyillpatents.
(b) Ismoreexpensivethanparenteralnutrition
(c) Resultsinfasterreturnofbowelfunctioninhealthypatientsaftergastrointestinalsurgery.
(d) Hasahighercomplicationratethanparenteralnutrition.
YourResponse:

CorrectAnswer:

Exp:

Resultsinreductionofinfectiouscomplicationsincriticallyillpatents.
(RefSchwartz:Principlesofsurgery5theditionpageno.96100)
Mostprospectiverandomizedstudiesforsevereabdominal&thoracictrumademonstratessignificant
reductionsininfectiouscomplicationforpatientsgivenearlyenteralnutritionwhencomparedwiththose
whoareunfedorreceivingparenteralnutrition.
Enteralfeedingprovidesphysiologic,metabolic,safety,andcostbenefitsoverparenteralnutrition.
Therearevariouswaysenteralnutritionalisadministeredandscheduled.Themethodofadministration
mustbeindividualizedtoeachpatient'sspecificneeds.ThefunctioningandcapacityoftheGItract,
underlyingdiseasestatesandpatienttolerancemustbeassessedinordertodeterminetheappropriate
administrationmethod.
ENadmini

Indications

Advantages

Intactgagreflex

Easytubeinsertion

Disadvantages

strationroutes
Nasogastric

Noesophagealreflux
Normalgastricemptying
Stomachuninvolvedwith
primarydisease
Nasojejunal

Highestriskofpulmonary
Largerreservoircapacity aspiration
instomach
Patientselfconsciousdueto
appearanceofnasogastrictube

Gastroparesisorimpaired
gastricemptying

TFmaybeinitiated
PotentialGIintolerancetogoalTF
immediatelyafterinjury infusionrate

Esophagealreflux

Reducedaspirationrisk
comparedtoNG

Gastricdysfunctiondueto
tramaorsurgery

Mayrequireendoscopic
placementoftube
Patientselfconsciousdueto
appearanceofnasoenterictube
Tubedisplacementandpotential
aspiration

Jejunostomy

http://dbmci.examonair.com/Result/ShowAllQuestionInHtml.aspx?testid=3572

Longtermfeeding

Reducedriskofaspiration

Highriskofaspiration

PlacedadjunctlywithGI
surgery

PotentialGIintoleranceto
goalTFinfusionrate
Stomacareneeded;potential

66/99

22/03/2016

DrBhatiaMedicalCoachingInstitute:OnlineTestPlatform
Esophagealreflux

Nosurgeryneededfor
InabilitytoaccessupperGI percutaneousendoscopic
tract
jejunostomy(PEJ)
Gastroparesisorimpaired PEJlesscostlythansurgical
gastricemptying
jejunostomy
Gastricdysfunctiondueto TFmaybeinitiated
tramaorsurgery
immediatelyafterinjury

infectionatstomasite
Potentialskinexcoriation
fromleakagesofdigestive
secretionsatstomasite
Potentialfistulaaftertube
removed
Tubeocclusionwithsmall
boretubeorneedlecatheter
Surgeryneededforsurgical
jejunostomies

(Q.135) Inapatientwithseverebullousemphysema,themostappropriatemethodformeasuringlungvolumesis
(a)

Bodyplethysmography

(b)

Diffusingcapacityofcarbonmonoxide

(c)

Spirometry

(d)

Heliumdilution

YourResponse:

CorrectAnswer:

Exp:

Bodyplethysmography
Spirometrydoesnotmeasuretotallungcapacitybecauseitcannotaccountforresidualvolumesoits
alreadyout
DLCOisaffectesinfectdecresasedincaseofCOPDandspeciallyemphysematouscasessothisalsocant
measuretotallungcapacityinbullousemphysema.
Themostfrequentlyusedandaccuratemeasuresoflungvolumesaresteadystateheliumdilutionlung
volumesandbodyplethysmography.
Inheliumdilutionthepatientinspiresaknownconcentrationofheliumfromaclosedcircuitofknown
volume.Afterthepatientrebreathesintheclosedcircuitforaperiodoftime,theconcentrationofhelium
equilibrates,andsubsequentlythelungvolumescanbecalculatedbyusingAvogadroslaw.This
calculationassumesthatgasinthecircuitwillrapidlyequilibratewiththeventilatedportionsofthelung.
However,ifthereareslowlyemptyingareasofthelung,asincysticfibrosispatients,orpartsofthelung
thatdonotparticipateingasexchangeatall,asinbullousemphysemapatients,heliumdilutionwill
underestimatetruelungvolumes.
Subsequently,bodyplethysmographyisthepreferredmethodforlungvolumemeasurementinthese
diseasestates.Toperformbodyplethysmography,thepatientsitsinasealedboxandpantsagainsta
closedmouthpiece.Pantingresultsinchangesinthepressureoftheboxthat,whencomparedwith
changesatthemouthpiece,canbeusedtocalculatelungvolumes.Thismethodmeasurestotalthoracic
gasvolumeandismoreaccuratethanheliumdilution.Heliumlungvolumesareeasiertoperformfor
patientsandstaffandgivereliableresultsinmostcircumstances.Manycentersmeasureasinglebreath
heliumdilutionlungvolumewhenmeasuringthediffusingcapacityofcarbonmonoxide,whichhasthe
sameorgreaterlimitationsastherebreathingmethod.Transdiaphragmaticpressureisusedtomeasure
respiratorymusclestrength,notlungvolumes.

(Q.136) Whatisthediagnosis?

(a)

Istdegreeheartblock

(b)

2nddegreeheartblock

(c)

Rightbundlebranchblock

(d)

Completeheartblock

YourResponse:

CorrectAnswer:

Exp:

Completeheartblock
REF:Ganong'sReviewofMedicalPhysiology>Chapter30.OriginoftheHeartbeat&theElectrical
ActivityoftheHeart
TheAVnodeandotherportionsoftheconductionsystemcan,inabnormalsituations,becomethecardiac
pacemaker.Inaddition,diseasedatrialandventricularmusclefiberscanhavetheirmembranepotentials
reducedanddischargerepetitively.
Asnotedabove,thedischargerateoftheSAnodeismorerapidthanthatoftheotherpartsofthe
conductionsystem,andthisiswhytheSAnodenormallycontrolstheheartrate.Whenconductionfrom
theatriatotheventriclesiscompletelyinterrupted,complete(thirddegree)heartblockresults,andthe
ventriclesbeatatalowrate(idioventricularrhythm)independentlyoftheatria.Theblockmaybedueto
diseaseintheAVnode(AVnodalblock)orintheconductingsystembelowthenode(infranodalblock).In
patientswithAVnodalblock,theremainingnodaltissuebecomesthepacemakerandtherateofthe
idioventricularrhythmisapproximately45beats/min.Inpatientswithinfranodalblockduetodiseasein
thebundleofHis,theventricularpacemakerislocatedmoreperipherallyintheconductionsystemandthe
ventricularrateislower;itaverages35beats/min,butinindividualcasesitcanbeaslowas15beats/min.
Insuchindividuals,theremayalsobeperiodsofasystolelastingaminuteormore.Theresultantcerebral
ischemiacausesdizzinessandfainting(StokesAdamssyndrome).Causesofthirddegreeheartblock

http://dbmci.examonair.com/Result/ShowAllQuestionInHtml.aspx?testid=3572

67/99

22/03/2016

DrBhatiaMedicalCoachingInstitute:OnlineTestPlatform
includeseptalmyocardialinfarctionanddamagetothebundleofHisduringsurgicalcorrectionof
congenitalinterventricularseptaldefects.Whenconductionbetweentheatriaandventriclesisslowedbut
notcompletelyinterrupted,incompleteheartblockispresent.Intheformcalledfirstdegreeheartblock,
alltheatrialimpulsesreachtheventriclesbutthePRintervalisabnormallylong.Intheformcalledsecond
degreeheartblock,notallatrialimpulsesareconductedtotheventricles.Forexample,aventricularbeat
mayfolloweverysecondoreverythirdatrialbeat(2:1block,3:1block,etc).Inanotherformofincomplete
heartblock,therearerepeatedsequencesofbeatsinwhichthePRintervallengthensprogressivelyuntila
ventricularbeatisdropped(Wenckebachphenomenon).ThePRintervalofthecardiaccyclethatfollows
eachdroppedbeatisusuallynormaloronlyslightlyprolonged.
SometimesonebranchofthebundleofHisisinterrupted,causingrightorleftbundlebranchblock.In
bundlebranchblock,excitationpassesnormallydownthebundleontheintactsideandthensweepsback
throughthemuscletoactivatetheventricleontheblockedside.Theventricularrateisthereforenormal,
buttheQRScomplexesareprolongedanddeformed.Blockcanalsooccurintheanteriororposterior
fascicleoftheleftbundlebranch,producingtheconditioncalledhemiblockorfascicularblock.Left
anteriorhemiblockproducesabnormalleftaxisdeviationintheECG,whereasleftposteriorhemiblock
producesabnormalrightaxisdeviation.Itisnotuncommontofindcombinationsoffascicularandbranch
blocks(bifascicularortrifascicularblock).TheHisbundleelectrogrampermitsdetailedanalysisofthesite
ofblockwhenthereisadefectintheconductionsystem.

(Q.137) Maturityonsetdiabetesoftheyoung(MODY)isinheritedas:
(a)

Autosomaldominantdisease

(b)

Autosomalrecessivedisease

(c)

Xlinkeddominantdisease

(d)

Xlinkedrecessivedisease

YourResponse:

CorrectAnswer:

Exp:

Autosomaldominant
Althoughnoninsulindependentdiabetesmellitusdisease(nonketogenic)isfamilial,theexactmodeof
inheritanceisnotknownexceptforthespecificvariantknownasmaturityonsetdiabetesoftheyoung
(MODY),whichismanifestedbymildhyperglycemiawithoutketosis.Onthebasisoffamilystudies,this
diseaseisinheritedinanautosomaldominantfashionwithalmostcompletepenetrance.Therefore,50%of
thechildrenofadiabeticparentwithMODYwilldevelopthedisease.ThereislinkagebetweenMODYand
mutationsintheglucokinasegeneontheshortarmofchromosome7.Thisabnormalityisnotpresentin
ordinarynonketoticdiabetics.Unlikethecaseininsulindependentdiabetes,noHLArelationshipshave
beenidentified.Moreover,anautoimmuneetiologyforthediseaseisnotfelttobeimportant;thisisalsoa
distinctivefeaturecomparedwithtypicaljuvenileonsetinsulindependentdiabetes

(Q.138) A35yroldfemaleduringapreoperativeworkuphadaNormalPTandincreasedaPTTof48secwithnormalcontrol2628.There

wasprevioushistoryofuneventfulcholecystectomydone2yrsback.Nextinvestigationtobefurtherevaluation?
(a)

ClottingFactor8levels

(b)

PlateletAggregationtest

(c)

Russelvipervenomtest

(d)

Ristocetincofactorassay

YourResponse:

CorrectAnswer:

Exp:

Russelvipervenomtest(RefH17th/pg.1795,622,369
Clottingfactor8isnotneededorsayitwonthelpbecausesheisfemale.factor8deficiencyoccursin
malesplusshehaduneventfullcholecystectomy2yearsbackwhichexplainssheisnothavingany
abnormalityinfactor8productionsofar.
NowthisisacaseofisolatedprolongedaPTT.
IsolatedprolongaPTTindicatesdefectsinintrinsicpathwaythesearestandardpaneloftestoneshould
performtodetectthecause
Lupusanticoagulantdetection
Measurementoffactor8,9,11,12
VWFantigenlevels
OptionnumberBandDareoutbecauseplateletaggregationtestandristocetincofactorassaywontaffect
aPTTthisaffectbleedingtime.ifthereisabnormalityinaPTTtherehastobeabnormalityinclottingtime
andnotbleedingtime.
Soweareleftwithoptionrusselviporvenomtestwhichisdoneforlupusanticoagulantandoneshoulddo
itasitisacquiredcauseofaPTTabnormality.
ANTIPHOSPHOLIPID/ANTICARDIOLIPINANTIBODYSYNDROME(APLAS)
AntiphospholipidantibodiesisIgAorIgMagainstenzymesincoagulationcascade,whichareenhancedby
plateletmembraneorphospholipids.
AntibodiesbindtophospholipidsinvitroandprolongsPTT.
ProlongationofRusselvipervenomtimeismoresensitive,itisneutralizedbyaddingplateletmembrane
(lupusanticoagulant).
Bleedingmanifestsinpresenceofthrombocytopeniahypoprothrombinaemia.
Antibodypredisposestothrombosisbymaintainingplateletsininactivatedstateorbyinhibitingfibrinolytic
activityofendothelialcells.
Antiphospholipidsyndromedevelopsinpatientsexpressingantibodiestoanionicphospholipids,
particularly2glycoprotein1.

http://dbmci.examonair.com/Result/ShowAllQuestionInHtml.aspx?testid=3572

68/99

22/03/2016

DrBhatiaMedicalCoachingInstitute:OnlineTestPlatform
Halfofthepatientshavenoobviouscause,afewarepregnant,somearealreadyreceivinghemodialysisor
havearenalallograft,andtheresthaveaprimaryglomerulonephritis(nillesionormembranous
nephropathy)orarheumatologicdiseasesuchasSLE.Lupuspatientsalsooftencoexpressalupus
anticoagulant,withelevationintheactivatedpartialtissuethromboplastintime.
Clinicalpresentationofthecatastrophicformofantiphospholipidsyndromeappearsasmixedthrombosis
ofthearterialandvenouscirculationwithvaryingdegreesofthrombocytopenia,hemolyticanemia,deep
veinthrombosis,transientischemicattacks,pulmonaryembolism,andspontaneousabortions;lesser
degreesofdiseasearemorecommon.
Diagnostictest
isRusselvipervenomPTT.
Treatment
Themainstayoftreatmentforantiphospholipidsyndromeiswarfarin.Thereisalsoevidenceofvasculitisin
manypatientsduetocomplementfixingantiphospholipidantibodies,whichrespondstotheadditionof
steroids.
Acuterenalfailuresometimesrespondstoremovalofantiphospholipidantibodieswithplasmapheresis
andadjustmentofimmunosuppressionwhereclinicallyindicated.
(Q.139) Apatientwithamyelodysplasticsyndrome(subtype,refractoryanemiawithringedsideroblasts)hasbeentransfusiondependent

forthepast2years.Thepatienthasreceivedatotalof50unitsofpackedredbloodcells.Hisphysicalexaminationisnormalexcept
forhyperpigmentation.Laboratoryevaluationrevealsmildglucoseintolerance.Atrialoferythropoietinwasunsuccessful.Whichof
thefollowingwouldbethemostimportanttherapeuticapproachatthistime?
(a)

Granulocytecolonystimulatingfactor(GCSF)

(b)

Phlebotomy

(c)

Ascorbicacid

(d)

Desferrioxamine

YourResponse:

CorrectAnswer:

Exp:

Desferrioxamine
Sinceeachunitoftransfusedbloodcontains200to250mgofironandnormalironexcretionisonly1
mg/d,apatientreceivingabout40unitsofbloodannuallywillaccumulateabout8gofiron,puttinghimor
heratriskforproblemsrelatedtotransfusionalironoverload.Inadditiontotherequirementformany
transfusions,thedisordermustalsohavealongnaturalhistorytoallowforthedevelopmentoftheclinical
sequelaeofchronicironoverload.Thalassemiamajor,myeloproliferativedisorders,myelodysplastic
syndromes(withoutexcessmyeloblasts),pureredcellaplasia,andmoderatelysevereaplasticanemiaare
diseasesthatmaybeassociatedwithtransfusionalironoverload.Thespectrumofproblemsproducedby
irondepositionintissuesincludescardiacdysfunction(arrhythmias,conductiondefects,andrestrictive
cardiomyopathy),hepaticcirrhosis,glucoseintolerance,gonadaldysfunction,andhyperpigmentationdue
toincreasedmelaninproductionsecondarytodermalirondeposition.Theonlyavailabletreatmentfor
transfusionassociatedhemochromatosis(phlebotomyisnotanoptionbecauseofchronicanemia)is
chelationwithdesferrioxamine,whichmustbegivensubcutaneouslyover12to16h/dbyaportable
pump.Whileoralascorbicacidmayenhanceironexcretioninpatientsreceivingdesferrioxamine,ithasno
roleasamonotherapyandmaybeassociatedwithdangerouscardiactoxicity.

(Q.140)

A39yearsoldpatientslaboratoryreportsrevealedaplateletcountof700x109/Lwithabnormalitiesinsize,shape,andgranularity
ofplatelets.WBCcountwas12x109/L,hemoglobinof11g/dlandtheabsenceofthePhiladelphiachromosome.Themostlikely
diagnosisis:
(a)

Polycythemiavera

(b)

Essentialthrombocythemia

(c)

CML

(d)

Leukemoidreaction

YourResponse:

CorrectAnswer:

Exp:

Essentialthrombocythemia
Thenormalhemoglobinlevelsvirtuallyexcludespolycythemiavera,andnormalcytogeneticswiththe
absenceofthePhiladelphiachromosomemakesthediagnosisofchronicmyelogenousleukemiamost
unlikely.
ThetermleukemoidreactionisusedtodescribepersistentneutrophiliawithWBCcountsofmorethanor
equalto30,00050,000/microLiterthatisnotleukemicinetiology.Thecirculatingneutrophilsareusually
matureandnotclonallyderived.Itisusuallyseeninresponsetoinfection,inflammation,ortherapeutic
agentssuchasgrowthfactorsandislesscommonlycausedbymalignancy.
Thusthepatientinthisquestionhasessentialthrombocythemia.

(Q.141) AutosomalDominantgastrointestinalpolyposissyndromeis?
(a)

Turcotsyndrome

(b)

Juvenilepolyposis

(c)

Lynchsyndrome

(d)

Alloftheabove.

YourResponse:

CorrectAnswer:

http://dbmci.examonair.com/Result/ShowAllQuestionInHtml.aspx?testid=3572

69/99

22/03/2016

DrBhatiaMedicalCoachingInstitute:OnlineTestPlatform

Exp:

Alloftheabove.

REFHarrison17thEdChapter87.
Hereditable(AutosomalDominant)GastrointestinalPolyposisSyndromes
Syndrome

Distributionof HistologicType Malignant AssociatedLesions


Polyps
Potential

Familial
adenomatous
polyposis

Largeintestine Adenoma

Common

None

Gardner's
syndrome

Largeandsmall Adenoma
intestines

Common

Osteomas,fibromas,lipomas,
epidermoidcysts,ampullary
cancers,congenital
hypertrophyofretinalpigment
epithelium

Turcot's
syndrome

Largeintestine Adenoma

Common

Braintumors

Nonpolyposis
Largeintestine Adenoma
syndrome(Lynch (often
syndrome)
proximal)

Common

Endometrialandovarian
tumors

PeutzJeghers
syndrome

Smallandlarge Hamartoma
intestines,
stomach

Rare

Mucocutaneouspigmentation;
tumorsoftheovary,breast,
pancreas,endometrium

Juvenile
polyposis

Largeandsmall Hamartoma,
intestines,
rarely
stomach
progressingto
adenoma

Rare

Variouscongenital
abnormalities

(Q.142) CD19positive,CD22positive,CDI03positivemonoclonalBcellswithbrightkappapositivitywerefoundtocomprise60%ofthe

peripheralbloodlymphoidcellsonflowcytometricanalysisina55yearoldmanwithmassivesplenomegalyandatotalleucocyte
countof3.3x109/1.Themostlikelydiagnosis?
(a)

Spleniclymphomawithvillouslymphocytes

(b)

Mantlecelllymphoma

(c)

Bcellprolymphocyticleukemia

(d)

Hairycellleukemia

YourResponse:

CorrectAnswer:

Exp:

Hairycellleukemia(RefRobbinsPathology7thed./683)
Presenceofmassivesplenomegaly,andgranulocytopenia,alongwithcharacteristicimmunophenotypic
features(speciallyCD103)suggeststhediagnosisofHairycellleukemia.CD103isexclusivelypositivein
Hairycellleukemiaandthusleavesnodoubtasregardsthecorrectdiagnosis.
ImmunophenotypicfeaturesofHairycellleukemia:
CDmarkers

Surfaceimmunoglobulins

PanBcellmarkers:CD19,CD20

Heavychain:IgG

Monocyticassociatedantigen:CD11c,CD25,CD103

Lightchain:or

SIg

CD5

CD10

CD11c

CD19

CD20

CD22

CD23

CD25

CD103

Hairycell
leukemia

/+

/+

++

++

/+

++

Mantlecell
lymphoma

++

/+

CLL

+/

++

/+

+/

/+

++

/+

Prolymphocytic
leukemia

++

/+

+/

+/

Splenicmarginal +
lymphoma

/+

/+

+/

/+

Lympho
plasmacytoma

/+

+/

/+

+/

+/

REMEMBER:
AQuestionisalwaysaskedonLEUKEMIASandmustbesolvedcorrectlywiththehelpofthefollowing
ALLTEXTSCOMPILEDkeytoleukemiaidentification:
CHARECTERISTICFEATURES

LEUKEMIA

CD:5,19,23,20+

CLL

LowlevelexpressionofSurfaceIgheavychainandeitherkappa/lambda
chain
CD:19,20,5,43+ve

Mantlecelllymphoma

Cd:23ve
Middleageandelderly
ModeratetohighlevelofexpressionofsurfaceIgandeitherkappa/

http://dbmci.examonair.com/Result/ShowAllQuestionInHtml.aspx?testid=3572

70/99

22/03/2016

DrBhatiaMedicalCoachingInstitute:OnlineTestPlatform
lambdachains
PositiveforcyclinD1.
CD:19,20,25(IL2receptoralphachain),103andotherPanBcell
markers+ve

Hairycellleukemia

PositiveforsurfaceIgheavyghain(IgG)andeitherkappaorlambda
chains
CD:5and23ve
FMC7+ve
TARTRATERESISTANTACIDPHOSPHATASE(TRAP)
CD:19,20,10+ve

Follicularlymphoma

SurfaceIg+ve
Translocation(14;18)
BCL2proteinandBCL6+ve
Ki67fractionislowerthaninrectivefollicles
CD43+ve
Myeloperoxidaseandlysozyme+ve

Granulocyticsarcoma/
Chloroma

CD:19,20,10+ve

Burkittlymphoma

BCL6+ve
BCL2ve
MatureBcellsexpressingsurfaceIgM
Monotypickappaorlambdachain
Translocation(8;14),(2;8),(8,22)
CD:20,45+ve

DiffuselatgeBcelllymphoma

CD:5,3ve
CD:3,30,+ve

Anaplasticlargecelllymphoma

CD:15,20ve
EMAandALK+ve
TdTandCD19,10+ve

PrecursorBcellALL

Aboveplus

EarlypreBcellALL

AbsentcytoplasmicIgMheavychain
Aboveplus

LatepreBcellALL

PresentcytoplasmicIgMheavychain
TdTandCD:1,2,5,7,+ve

PrecursorTcellALL

Aboveplus

EarlypreTcellALL

CD:3,4and8allve
Aboveplus

LatepreTcellALL

CD:3,4,8all+ve
CD:15,30+ve

ClassicalHodgkinsdisease

CD:15,30ve

LymphocytepredominantHD

CD:20,45+ve
AndGerminalcentrespecifictranscriptionfactorBCL6+ve
CD13,33,34+ve

AMLM0

CD:13,33,34,15+ve

AMLM2

GlycophorinA+ve

AMLM6Erythroleukemia

CD41,CD61

AMLM7Megakaryocytic

CD11b,14+ve

AMLwithmonocytic
differentiation(M4,M5)

CD;20,79bandsmig+ve

ProlymphocyticBcellleukemia

(Q.143) A63yearoldmanpresentedwithmassivesplenomegaly,lymphadenopathyandaTLCof17000permm3.Theflowcytometry

showedCD19positive,CD5positive,CD23negative,monoclonalBcellswithbrightkappapositivelycomprising80%ofthe
peripheralbloodlymphoidcells.Themostlikelydiagnosisis:
(a)

Mantlecelllymphoma

(b)

Spleniclymphomawithvillouslymphocytes

(c)

Follicularlymphoma

(d)

Hairycellleukemia

YourResponse:

CorrectAnswer:

Exp:

Mantlecelllymphoma
Mantlecelllymphoma
Presentsinthemiddleagedandtheelderly(MeanAge=63yrs)
ClinicalprofileconsistofPainlesslymphadenopathy,Splenomegaly,andOccasionalGIinvolvement.
ImmunocytochemicalmarkersofMantleCellLymphoma

http://dbmci.examonair.com/Result/ShowAllQuestionInHtml.aspx?testid=3572

71/99

22/03/2016

DrBhatiaMedicalCoachingInstitute:OnlineTestPlatform
CDmarkers

SURFACEimmunoglobulins

CellsarepositiveforPANB
markersD19,CD2,CD22,
CD24

Moderatelyhighlevelsof
t(11;14)translocation14t(11;14)
surfaceimmunoglobulinsheavy translocationleadstoincreasedexpression
chainsIgMandIgD
ofcyclinD1

CellsarepositiveforCD5

BcellspresentbrightK
positivity(eitherKorlight
chainmaybepresent)

CellsarenegativeforCD23
CellsarenegativeforCD10

Translocation

(Q.144) Achildunderwentatonsillectomyat6yearsofagewithnocomplications.Heunderwentapreoperativescreeningforbleedingat

theageof12yearsbeforeanelectivelaparotomy,andwasfoundtohaveaprolongedpartialthromboplastintimebutnormalPT
andthrombintime.Therewasnofamilyhistoryofbleeding.Thepatientislikelytohave?
(a)

AcquiredvitaminKdeficiency

(b)

Acquiredliverdisease

(c)

FactorXIIdeficiency

(d)

MildhemophiliaA

YourResponse:

CorrectAnswer:

Exp:

FactorXIIdeficiency:
AcquiredvitaminKdeficiencywillcausereducedfactor2,7,9,10andcausebothraisedPTandaPTT
SimilarlyliverdiseasealsoaffectbothPTandaPTT
Weareleftwithfactor8deficiencyorfactorXIIdeficiency
IsolatedprolongationofPTTsuggestadefectintheintrinsicpathwayofcoagulationcascadeonly,and
leavesuswiththepossibilityofeitherfactorXIIdeficiencyorHemophiliaA(FactorVIIIdeficiency).
Absenceofclinicalbleedingevenaftermajorsurgeryliketonsillectomysuggestsaclassicaldeficiencyof
contactfactorXII.
IsolatedelongationofaPTTwithbleeding
manifestations

IsolatedprolongationofaPTTwithoutbleeding
manifestations

FactorVIIIdefect

FactorXIIdefect

FactorIXdefect

Prekallikreindefect

FactorXIdefect

HMWkininogendefect

Nonbleedingdisorder/Deficiencyofthecontactfactors:
Contactfactorsfunctionatthestrepofinitiationofintrinsicpathwayoftheclottingcascadeandinclude
thefollowing:
(Q.145) ADAMTS13deficiencyorinhibitionbytheblockingantibodiesisfoundin:
(a)

IdiopathicThrombocytopenicPurpura

(b)

ThromboticThrombocytopenicPurpura

(c)

Henochschoenleiniipurpura

(d)

Glanzmannsthrombasthenia

YourResponse:

CorrectAnswer:

Exp:

ThromboticThrombocytopenicPurpura
Theetiologyofthromboticthrombocytopenicpurpura(TTP)hasbeenlinkedtoabnormalitiesofthevon
WillebrandfactorcleavingmetalloproteaseknownasADAMTS13.
SinglegeneencodesamemberoftheADAMTS(adisintegrinandmetalloproteinasewiththrombospondin
motif)proteinfamily.TheenzymeencodedbythisgeneisthevonWillebrandFactor(vWF)cleaving
protease,whichisresponsibleforcleavingatthesiteofTyr842Met843ofthevWFmolecule.Adeficiency
ofthisenzymeisassociatedwiththromboticthrombocytopenicpurpura.
Earlydiagnosisandpromptinitiationofplasmaexchangeiscriticaltotheclinicaloutcomeinpatients
diagnosedwithTTP.

(Q.146) Whichofthefollowingstatementspertainingtoleukemiaiscorrect?
(a) BlastsofacutemyeloidleukemiaaretypicallySudanblacknegative.
(b) Blastsofacutelymphoblasticleukemiaaretypicallymyeloperoxidasepositive
(c) Lowleukocytealkalinephosphatasescoreischaracteristicallyseeninblastphaseofchronicmyeloidleukemia
(d) Tartarateresistantacidphosphatase(TRAP)positivityistypicallyseeninhairycellleukemia
YourResponse:

CorrectAnswer:

Exp:

TRAPpositivityistypicallyseeninhairycellleukemia
AML(acutemyeloidleukemia):
M0 Minimallydifferentiated 23%

MPOnegative

M1 Withoutdifferentiation

20%

>/=3%blastsareMPOpositive

M2 Withmaturation

3040%

Fullrangeofmyeloidmaturation;Auerrods+;

http://dbmci.examonair.com/Result/ShowAllQuestionInHtml.aspx?testid=3572

72/99

22/03/2016

DrBhatiaMedicalCoachingInstitute:OnlineTestPlatform
t(8,21)
M3 Acutepromyelocytic

510%

Hypergranularpromyelocyteswithmanyauerrods
percell,youngerpatient,highincidenceofDIC;

M4 Acutemyelomonocytic

1520%

Myelocyticandmonocyticdifferentiation,NSE+

M5 Acutemonocytic

10%

Subtypesaandb,MonoblastsMPOnegative,
NSE+,occursinolderpatients,organomegaly,
lymphadenopathy,chloromasandtissue
infiltration.

M6 Acuteerythroleukemia

5%

Dysplasticerythroidprecursorspredominate;
>30%myeloblasts,occurinadvancedage

1%

Blastsofmegakaryocyticlineagepredominate;
BlastsreactwithplateletspecificabagainstGO
IIb/IIIaorvMF;Myelofibrosisorincreased
reticulin.

t(15,17)

(Dguglimossyndrome)
M7 Acutemegakaryocytic

A.NotallblastsofAMLarepositiveforsudanblack
BBlastofALLarePASpositive
C.LeucocytealkalineisalwayslowinCMLitsnotcharactersticofblastcrisis
(Q.147) WhichofthefollowingisnotanAcutePhaseReactant?
(a)

CRP

(b)

Fibrinogen

(c)

Tissuefactor

(d)

Serumamyloidaprotein

YourResponse:

CorrectAnswer:

Exp:

Tissuefactor.(RefRobbinspathology8th/pg.74;HarpersBiochemistry26th/pg.583)
CRP,FibrinogenandSerumamyloidAproteinarethreemostimportantacutephasereactants.
Acutephaseprotein:
Alsocalledanacutephasereactant.
Anyproteinwhoseplasmaconcentrationincreases(ordecreases)by25%ormoreduringcertain
inflammatorydisorders.TheacutephaseproteinsincludeCreactiveprotein(CRP),serumamyloidA(SAA),
fibrinogen,andalpha1acidglycoprotein.
PerhapsthebestknownoftheseacutephaseproteinsisCRP,aplasmaproteinthatrisesinthebloodwith
inflammation.ThelevelofCRPinbloodplasmacanriseashighas1000foldwithinflammation.Conditions
thatcommonlyleadtomarkedchangesinCRPincludeinfection,trauma,surgery,burns,inflammatory
conditions,andadvancedcancer.Moderatechangesoccurafterstrenuousexercise,heatstroke,and
childbirth.SmallchangesinCRPoccurafterpsychologicalstressandinseveralpsychiatricillnesses.
Creactiveproteinisatestofvalue.MarkedrisesinCRPreflectthepresenceandintensityofinflammation.
AnelevationinCRP,however,isnotatelltalesignpointingtojustonedisease.
AcutePhaseReactants:
1Acidglycoproteina
AlphaMacroglobulinCRP
CeruloplasminHaptoglobin
HemopexinLipopolysaccharidebindingprotein(LBP)
1inhibitor3oralphaXprotein)aMurinoglobulin
Thiostatin
Miscellaneous
Lactoferrins:Humanandbovineholoandapolactoferrin
InfectiousDiseaseAntigens:HepatitisBcoreandHepatitisBeantigens
HumanCardiacMarkers:
TroponinI,troponinT,troponinC,myoglobinandfattyacidbindingprotein(FABP).

(Q.148) Bananashapedgametocytesareseenin?
(a)

Pl.falciparum

(b)

Pl.vivax

(c)

Pl.malariae

(d)

Pl.ovale

YourResponse:

CorrectAnswer:

Exp:

Pl.falciparum

CharacteristicsFalciparumVivaxOvaleMalariae

Accoleforms+

http://dbmci.examonair.com/Result/ShowAllQuestionInHtml.aspx?testid=3572

73/99

22/03/2016

DrBhatiaMedicalCoachingInstitute:OnlineTestPlatform
Durationofintrahepaticphase(days)5.58915
Merozoites/infectedhepatocytes30,00010,00015,00015,000
Durationoferythrocytecycle48485072
RedcellpreferenceYoungercellsReticsReticsBand
MorphologyRingformsRingformsEnlargedTropozoites
BananagameEnlargedRBCsRBCs
TococytesSchuffnerdotSchuffnerdot
PigmentcolorBlackYellowDarkbrownBlack
AbilitytocauserelapseNoYesYesNo
NephroticSyndromeNoNoNoYes

Drugswhich,attackexoerythrocyticstage(hypnozoites)giventogetherwithaclinicalcurativeachievetotal
eradicationofparasitefrompatientsbody,knownasradicalcure.
Aradicalcureisneededinrelapsingmalaria(plasmodiumalemalaria),whileinFalciparummalaria,
adequatetreatmentofclinicalattackleavesnoparasiteinthebody(thereisnosecondaryexoerythrocytic
tissuephase).
(Q.149) A70yrpatientwithlongstandingtype2DMpresentswithC/Opainintheleftearwithpurulentdrainage.O/E,thepatientis

afebrile.Thepinnaoftheleftearistender,andtheexternalauditorycanalisswollenandedematous.TLCisnormal.Theorganism
mostlikelytogrowfromthepurulentdrainageis?
(a)

Staphylococcusaureus

(b)

Moraxellacatarrhalis

(c)

Haemophilusinfluenzae

(d)

Pseudomonasaeruginosa

YourResponse:

CorrectAnswer:

Exp:

Pseudomonasaeruginosa
Malignantexternalotitis
Theswellingandinflammationoftheexternalauditorymeatusstronglysuggestthisdiagnosis.This
infectionusuallyoccursinolderdiabeticsandisalmostalwayscausedbyP.aeruginosa.H.influenzaeand
M.catarrhalisfrequentlycauseotitismedia,butnotexternalotitis.

(Q.150) Symptomsofanxietyandpanicareassociatedwithalowlevelofwhichneurotransmitter?
(a)

Cortisol

(b)

Dopamine

(c)

Gammaaminobutyricacid

(d)

Glutamate

YourResponse:

CorrectAnswer:

Exp:

Gammaaminobutyricacid.
DISEASE

ASSOCIATEDCHANGEINNEUOTRANSMITTER

Sucidalbehaviour

5HIAAandserotonin

Anxiety

NE,GABA,serotonin(5HT).

Depression

NEandserotonin(5HT).

Alzheimersdementia

ACh.

Huntingtonsdisease

GABA,ACh.

Schizophrenia

dopamine.

Parkinsonsdisease

dopamine,ACh.

(Q.151) Allofthefollowingarepathologicaldefensemechanismsexcept?
(a)

Conversion

(b)

Denial

(c)

Splitting

(d)

Displacement

YourResponse:

CorrectAnswer:

Exp:

Displacement
Vaillant'scategorizationofdefencemechanisms
LevelIpathologicaldefences(i.e.psychoticdenial,delusionalprojection)
LevelIIimmaturedefences(i.e.fantasy,projection,passiveaggression,actingout)
LevelIIIneuroticdefences(i.e.intellectualization,reactionformation,dissociation,displacement,
repression)

http://dbmci.examonair.com/Result/ShowAllQuestionInHtml.aspx?testid=3572

74/99

22/03/2016

DrBhatiaMedicalCoachingInstitute:OnlineTestPlatform

LevelIVmaturedefences(i.e.humour,sublimation,suppression,altruism,anticipation)
Level1:Pathological
Themechanismsonthislevel,whenpredominating,almostalwaysareseverelypathological.Thesesix
defences,inconjunction,permitonetoeffectivelyrearrangeexternalexperiencestoeliminatetheneedto
copewithreality.Thepathologicalusersofthesemechanismsfrequentlyappearirrationalorinsaneto
others.Thesearethe"psychotic"defences,commoninovertpsychosis.However,theyarefound
indreamsandthroughoutchildhoodaswell.
Theyinclude:
DelusionalProjection:Delusionsaboutexternalreality,usuallyofapersecutorynature.
Conversion:Theexpressionofanintrapsychicconflictasaphysicalsymptom;someexamplesinclude
blindness,deafness,paralysis,ornumbness.Thisphenomenaissometimescalledhysteria.
Denial:Refusaltoacceptexternalrealitybecauseitistoothreatening;arguingagainstananxiety
provokingstimulusbystatingitdoesn'texist;resolutionofemotionalconflictandreductionofanxietyby
refusingtoperceiveorconsciouslyacknowledgethemoreunpleasantaspectsofexternalreality.
Distortion:Agrossreshapingofexternalrealitytomeetinternalneeds.
Splitting:Aprimitivedefence.Negativeandpositiveimpulsesaresplitoffandunintegrated.
Extremeprojection:Theblatantdenialofamoralorpsychologicaldeficiency,whichisperceivedasa
deficiencyinanotherindividualorgroup.
SuperiorityComplex:Apsychologicaldefencemechanisminwhichaperson'sfeelingsofsuperiority
counterorconcealhisorherfeelingsofinferiority.
InferiorityComplex:Abehaviourthatisdisplayedthroughalackofselfworth,anincreaseofdoubtand
uncertainty,andfeelingofnotmeasuringuptosociety'sstandards.
Level2:Immature
Thesemechanismsareoftenpresentinadults.Thesemechanismslessendistressandanxietyprovokedby
threateningpeopleorbyuncomfortablereality.Excessiveuseofsuchdefencesisseenassocially
undesirableinthattheyareimmature,difficulttodealwithandseriouslyoutoftouchwithreality.These
arethesocalled"immature"defencesandoverusealmostalwaysleadstoseriousproblemsinaperson's
abilitytocopeeffectively.Thesedefencesareoftenseeninmajordepressionandpersonalitydisorders.
Theyinclude:
Actingout:Directexpressionofanunconsciouswishorimpulseinaction,withoutconsciousawarenessof
theemotionthatdrivesthatexpressivebehaviour.
Fantasy:Tendencytoretreatintofantasyinordertoresolveinnerandouterconflicts.
Wishfulthinking:Makingdecisionsaccordingtowhatmightbepleasingtoimagineinsteadofbyappealing
toevidence,rationality,orreality
Idealization:Unconsciouslychoosingtoperceiveanotherindividualashavingmorepositivequalitiesthan
heorshemayactuallyhave.[19]
Passiveaggression:Aggressiontowardsothersexpressedindirectlyorpassivelysuchas
usingprocrastination.
Projection:Projectionisaprimitiveformofparanoia.Projectionalsoreducesanxietybyallowingthe
expressionoftheundesirableimpulsesordesireswithoutbecomingconsciouslyawareofthem;attributing
one'sownunacknowledgedunacceptable/unwantedthoughtsandemotionstoanother;includes
severeprejudice,severejealousy,hypervigilancetoexternaldanger,and"injusticecollecting".Itisshifting
one'sunacceptablethoughts,feelingsandimpulseswithinoneselfontosomeoneelse,suchthatthose
samethoughts,feelings,beliefsandmotivationsareperceivedasbeingpossessedbytheother.
Projectiveidentification:Theobjectofprojectioninvokesinthatpersonpreciselythethoughts,feelingsor
behavioursprojected.
Somatization:Thetransformationofnegativefeelingstowardsothersintonegativefeelingstowardself,
pain,illness,andanxiety.
Level3:Neurotic
Thesemechanismsareconsideredneurotic,butfairlycommoninadults.Suchdefenceshaveshortterm
advantagesincoping,butcanoftencauselongtermproblemsinrelationships,workandinenjoyinglife
whenusedasone'sprimarystyleofcopingwiththeworld.
Theyinclude:
Displacement:Defencemechanismthatshiftssexualoraggressiveimpulsestoamoreacceptableorless
threateningtarget;redirectingemotiontoasaferoutlet;separationofemotionfromitsrealobjectand
redirectionoftheintenseemotiontowardsomeoneorsomethingthatislessoffensiveorthreateningin
ordertoavoiddealingdirectlywithwhatisfrighteningorthreatening.Forexample,amothermayyellat
herchildbecausesheisangrywithherhusband.
Dissociation:Temporarydrasticmodificationofone'spersonalidentityorcharactertoavoidemotional
distress;separationorpostponementofafeelingthatnormallywouldaccompanyasituationorthought.
Hypochondriasis:Anexcessivepreoccupationorworryabouthavingaseriousillness.
Intellectualization:Aformofisolation;concentratingontheintellectualcomponentsofasituationsoasto
distanceoneselffromtheassociatedanxietyprovokingemotions;separationofemotionfromideas;
thinkingaboutwishesinformal,affectivelyblandtermsandnotactingonthem;avoidingunacceptable
emotionsbyfocusingontheintellectualaspects
(e.g.isolation,rationalization,ritual,undoing,compensation,magicalthinking).
Isolation:Separationoffeelingsfromideasandevents,forexample,describingamurderwithgraphic
detailswithnoemotionalresponse.
Rationalization(makingexcuses):Whereapersonconvinceshimorherselfthatnowrongwasdoneand
thatallisorwasallrightthroughfaultyandfalsereasoning.Anindicatorofthisdefencemechanismcanbe
seensociallyastheformulationofconvenientexcusesmakingexcuses.
Reactionformation:Convertingunconsciouswishesorimpulsesthatareperceivedtobedangerousor
unacceptableintotheiropposites;behaviourthatiscompletelytheoppositeofwhatonereallywantsor
feels;takingtheoppositebeliefbecausethetruebeliefcausesanxiety.Thisdefencecanworkeffectively
forcopingintheshortterm,butwilleventuallybreakdown.
Regression:Temporaryreversionoftheegotoanearlierstageofdevelopmentratherthanhandling

http://dbmci.examonair.com/Result/ShowAllQuestionInHtml.aspx?testid=3572

75/99

22/03/2016

DrBhatiaMedicalCoachingInstitute:OnlineTestPlatform

unacceptableimpulsesinamoreadultway.(ex.Usingwhiningasamethodofcommunicatingdespite
alreadyhavingacquiredtheabilitytospeakwithappropriategrammar)
Repression:Theprocessofattemptingtorepeldesirestowardspleasurableinstincts,causedbyathreatof
sufferingifthedesireissatisfied;thedesireismovedtotheunconsciousintheattempttopreventitfrom
enteringconsciousnessseeminglyunexplainablenaivety,memorylapseorlackofawarenessofone'sown
situationandcondition;theemotionisconscious,buttheideabehinditisabsent.[citationneeded]
Undoing:Apersontriesto'undo'anunhealthy,destructiveorotherwisethreateningthoughtbyactingout
thereverseofunacceptable.Involvessymbolicallynullifyinganunacceptableorguiltprovokingthought,
idea,orfeelingbyconfessionoratonement.
Withdrawal:Withdrawalisamoresevereformofdefence.Itentailsremovingoneselffromevents,stimuli,
interactions,etc.underthefearofbeingremindedofpainfulthoughtsandfeelings.
Upwardanddownwardsocialcomparisons:Adefensivetendencythatpeopleuseasameansofself
evaluation.Theseindividualswilllooktoanotherindividualorcomparisongroupwhoareconsideredtobe
worseoffinordertodissociatethemselvesfromperceivedsimilaritiesandtomakethemselvesfeelbetter
abouttheirselforpersonalsituation.
Level4:Mature
Thesearecommonlyfoundamongemotionallyhealthyadultsandareconsideredmature,eventhough
manyhavetheiroriginsinanimmaturestageofdevelopment.Theyhavebeenadaptedthroughtheyears
inordertooptimisesuccessinhumansocietyandrelationships.Theuseofthesedefencesenhances
pleasureandfeelingsofcontrol.Thesedefenceshelpustointegrateconflictingemotionsandthoughts,
whilststillremainingeffective.Thosewhousethesemechanismsareusuallyconsideredvirtuous.
Theyinclude:
Humility:Aqualitybywhichapersonconsideringhisowndefects,hasahumbleopinionofhimself.
Humilityisintelligentselfrespectwhichkeepsusfromthinkingtoohighlyortoomeanlyofourselves.
Mindfulness:Adoptingaparticularorientationtowardonesexperiencesinthepresentmoment,an
orientationthatischaracterisedbycuriosity,openness,andacceptance.
Acceptance:Aperson'sassenttotherealityofasituation,recognizingaprocessorcondition(oftena
negativeoruncomfortablesituation)withoutattemptingtochangeit,protest,orexit.Religionsand
psychologicaltreatmentsoftensuggestthepathofacceptancewhenasituationisbothdislikedand
unchangeable,orwhenchangemaybepossibleonlyatgreatcostorrisk.
Gratitude:Afeelingofthankfulnessorappreciation,involvesappreciationofawiderrangeofpeopleand
events.Peoplewhofeelmoregratitudearemuchmorelikelytohavehigherlevelsofhappiness,andlower
levelsofdepressionandstress.Throughouthistory,gratitudehasbeengivenacentralpositioninreligious
andphilosophicaltheories.
Altruism:Constructiveservicetoothersthatbringspleasureandpersonalsatisfaction.
Tolerance:Thepracticeofdeliberatelyallowingorpermittingathingofwhichonedisapproves.
Mercy:Compassionatebehavioronthepartofthoseinpower.
Forgiveness:Cessationofresentment,indignationorangerasaresultofaperceivedoffence,
disagreement,ormistake,orceasingtodemandpunishmentorrestitution.
Anticipation:Realisticplanningforfuturediscomfort.
Humour:Overtexpressionofideasandfeelings(especiallythosethatareunpleasanttofocusonortoo
terribletotalkabout)thatgivespleasuretoothers.Thethoughtsretainaportionoftheirinnatedistress,
buttheyare"skirtedround"bywitticism,forexampleSelfdeprecation.
Identification:Theunconsciousmodellingofone'sselfuponanotherperson'scharacterandbehaviour.
Introjection:Identifyingwithsomeideaorobjectsodeeplythatitbecomesapartofthatperson.
Sublimation:Transformationofnegativeemotionsorinstinctsintopositiveactions,behaviour,oremotion
(e.g.,playingaheavycontactsportsuchasfootballorrugbycantransformaggressionintoagame).
Thoughtsuppression:Theconsciousprocessofpushingthoughtsintothepreconscious;theconscious
decisiontodelaypayingattentiontoanemotionorneedinordertocopewiththepresentreality;making
itpossibletolateraccessuncomfortableordistressingemotionswhilstacceptingthem.
Emotionalselfregulation:Theabilitytorespondtotheongoingdemandsofexperiencewiththerangeof
emotionsinamannerthatissociallytolerable

(Q.152) Anadolescentboyissuspectedtobesufferingfromalcoholabuse.Whichofthemetabolitedetectedinhisurinewillhelpconfirm

thesuspicion?
(a)

Acetaldehyde

(b)

Cocaine

(c)

Formaldehyde

(d)

Acetaminophen

YourResponse:

CorrectAnswer:

Exp:

Acetaldehyde
UrineScreeningforDrugsCommonlyAbusedbyAdolescents
DRUG

MAJORMETABOLITE

APPROXIMATERETENTIONTIME

Alcohol(blood)

Acetaldehyde

710hr

Alcohol(urine)

Acetaldehyde

1013hr

Amphetamines

48hr

Barbiturates

Shortacting(24hr);longacting(23wk)

Benzodiazepines
Cannabinoids

http://dbmci.examonair.com/Result/ShowAllQuestionInHtml.aspx?testid=3572

3days
Carboxyand

310days(occasionaluser);12mo(chronicuser)

76/99

22/03/2016

DrBhatiaMedicalCoachingInstitute:OnlineTestPlatform
hydroxymetabolites

Cocaine

Benzoylecgonine

24days

Methaqualone

Hydroxylatedmetabolites

2wk

Morphine

2days

Opiates
Heroin

Glucuronide
Morphine

Morphine

2days

Glucuronide
Codeine

Morphine

2days

Glucuronide
Phencyclidine

8days

ModifiedfromDrugsofabuseurinescreening[Physicianinformationsheet].LosAngeles,Pacific
Toxicology.FromMacKenzieRG,KipkeMD:Substanceuseandabuse.InFriedmanSB,FisherM,Schonberg
SK(editors):ComprehensiveAdolescentHealthCare.St.Louis,QualityMedicalPublishing,1992,p783.
(Q.153) Falseaboutbulimianervosais?
(a)

Onsetinearlyadulthood

(b)

Commoninmales

(c)

Weightisnormal

(d)

Mortalityislow

YourResponse:

CorrectAnswer:

Exp:

Commoninmales
CommonCharacteristicsofAnorexiaNervosaandBulimiaNervosa
AnorexiaNervosaa

BulimiaNervosa

Onset

Midadolescence

Lateadolescence/early
adulthood

Female:male

10:1

10:1

Lifetimeprevalencein
women

1%

13%

Weight

Markedlydecreased

Usuallynormal

Menstruation

Absent

Usuallynormal

Bingeeating

2550%

Requiredfordiagnosis

Mortality

5%perdecade

Low

ClinicalCharacteristics

PhysicalandLaboratoryFindingsa
Skin/extremities

Lanugo
Acrocyanosis
Edema

Cardiovascular

Bradycardia
Hypotension

Gastrointestinal

Salivaryglandenlargement

Salivaryglandenlargement

Slowgastricemptying

Dentalerosion

Constipation
Elevatedliverenzymes
Hematopoietic

Normochromic,normocyctic
anemia
Leukopenia

Fluid/Electrolyte

IncreasedBUN,creatinine

Hypokalemia

Hypokalemia

Hypochloremia
Alkalosis

Endocrine

Hypoglycemia
Lowestrogenor
testosterone
LowLHandFSH
Lownormalthyroxine
NormalTSH
Increasedcortisol

Bone

http://dbmci.examonair.com/Result/ShowAllQuestionInHtml.aspx?testid=3572

Osteopenia

77/99

22/03/2016

DrBhatiaMedicalCoachingInstitute:OnlineTestPlatform
(Q.154) An8yroldboypresentedwithwelldefinedannularlesionoverthebuttockwithcentralscaringthatisgraduallyprogressiveover

thelast8months.Whatisthediagnosis?
(a)

Annularpsoriasis

(b)

Lupusvulgaris

(c)

Tineacorporis

(d)

Chronicgranulomatousdisease

YourResponse:

CorrectAnswer:

Exp:

LupusVulgaris(Ref.Rookstextbook7th/28.1030)
Annularpsoriasis:looselyadherentsilveryscales.Noscarring.
Tinea:itchingisveryimportant.
Lupusvulgaris:lesionsconsistoferythematous,induratedplaquewhichmayulcerate.Healingwithscar
formationinsomeareasandprogressionoflesioninotherdirectionisquitefrequent.Longdurationisalso
afavorablepoint.

(Q.155) Ayoungboypresentedwithalesionoverhisrightbuttock,whichhadperipheralscalingwithcentralscarring.Theinvestigationof

choicewouldbe?
(a)

Tzancksmear

(b)

KOHpreparation

(c)

Biopsy

(d)

Sabouraudsagar

YourResponse:

CorrectAnswer:

Exp:

Biopsy(Ref.Rookstextbook7th/28.1030)
Centralclearing:T.CorporisKOHsmear
Centralscaring:LupusvulgarisBiopsy(scarisvulgardobiopsy
Centralcrusting:LeishmaniasisLTbodydemonstration(crispycrustofleishmanianeedsLTbody
demonstration)

(Q.156) TrueaboutScrofuloderma:
(a)

ChronicinfectionbyStaphylococcusepidermidis

(b)

Tubercularsinuswithdischarge

(c)

Ruptureofalymphnodewithinvolvementofskin

(d)

Lesioninlupusvulgaris

YourResponse:

CorrectAnswer:

Exp:

Ruptureofalymphnodewithinvolvementofskin
Lymphnodetuberculosispresentsaspainlessswellingofthelymphnodes,mostcommonlyatcervicaland
supraclavicularsites,aconditionoftenreferredtoasscrofula.
SCROFULODERMA
Itresultsfromenlargement,coldabscessformation,andbreakdownofalymphnode,mostfrequentlyina
cervicalchain,withextensiontotheoverlyingskin.
Linearorserpiginousulcersanddissectingfistulasandsubcutaneoustractsstuddedwithsoftnodulesmay
develop.
Spontaneoushealingmaytakeyears,eventuatingincordlikekeloidscars;lupusvulgarismayalsodevelop.
Scrofulodermaofacervicallymphnodeoftenoriginatesinthelarynxandwaslinkedinthepastto
ingestionofmilkcontainingM.bovis.
Lesionsmayalsooriginatefromanunderlyinginfectedjoint,tendon,bone,orepididymis.
Thedifferentialdiagnosisincludessyphiliticgumma,deepfungalinfections,actinomycosis,and
hidradenitissuppurativa.
Thecourseisindolent,andconstitutionalsymptomsaretypicallyabsent.
Antituberculoustherapyisusuallyeffective.

(Q.157) A62yearoldfarmerpresentedwithawartyplaqueonthesoleoftherightfootsinceayear.Examinationrevealedalarge,warty,

hyperpigmentedplaqueonthesoleoftheinvolvedfoot.Whatistheprobablediagnosis?
(a)

Tuberculosisverrucosacutis

(b)

Verrucavulgaris

(c)

Mycetomafoot

(d)

Lichensimplexhypertrophicus

YourResponse:

CorrectAnswer:

Exp:

Tuberculosisverrucosacutis

http://dbmci.examonair.com/Result/ShowAllQuestionInHtml.aspx?testid=3572

78/99

22/03/2016

DrBhatiaMedicalCoachingInstitute:OnlineTestPlatform

(Ref.IADVLTextbookofDermatology3rd/pg.244;Robbins8th/pg.1200)
Tuberculosisverrucosacutis
iscausedbyexogenousinoculationoftuberclebacilliintotheskinofindividualswithapreexisting
moderatelyhighdegreeofimmunitytotheorganism.
Intropicalclimates,tuberculosisverrucosacutisisgenerallyadiseaseaffectingchildrenoryoungadults
whocontractthebacteriabywalkingbarefootorawartyplaqueonthesoleofthefootsittingonground
contaminatedwithtuberculoussputum.
Insuchcases,lesionsdeveloponthesolesofthefeet,asinourcase,oronthebuttocks.
Achronicallypresentlarge,warty,hyperpigmentedplaqueispresentonthesoleofthefootissuggestive
ofTuberculosisverrucosacutis.
Skinhistopathologyshowsthedermalinflammation,whichincludesanepithelioidcellgranulomawitha
centralmultinucleatedLangerhansgiantcell.
Caseationnecrosisisnotpresentinthisgranuloma.

(Q.158) Falseabouttuberculoidformofleprosy?
(a) ResistancetoM.lepraeispoor,largenumbersoforganismsarepresentandskinlesionsareextensive.
(b) Pregnancyisarecognizedprecipitatingfactorduetoarelativelyreducedimmunity.
(c) SatellitepapulesareoftenfoundadjacenttothemainplaquesinBT.
(d) Lossofsensationinskinlesionsistheruleandregionalnerveinvolvementwithenlargementandpalsiesiscommon.
YourResponse:

CorrectAnswer:

Exp:

ResistancetoM.lepraeispoor,largenumbersoforganismsarepresentandskinlesionsareextensive
Tuberculoidformsofleprosy
Intuberculoidformsofleprosy,resistancetoM.lepraeishigh,smallnumbersoforganismsarepresent
andskinlesionsarefew,Histopathologyofaskinbiopsyrevealsgranulomatousinflammation,often
showingneurotropism.
Pregnancyisarecognizedprecipitatingfactorforleprosyduetoarelativelyreducedimmunity.Inthepost
partumperiodtypeIreactions(characterizedbysuddenenhancedinflammationofskinlesionsand
neuritis)canoccurwhennormalimmunefunctionisrestored.
Inborderlinetuberculoidleprosysatellitepapulesareoftenfoundadjacenttothemainplaques.
JustgothroughthefollowingtablefromHARRISON17thEdchapter152
Clinical,Bacteriologic,Pathologic,andImmunologicSpectrumofLeprosy
Feature

Tuberculoid(TT,BT)
Leprosy

Borderline(BB,BL)Leprosy Lepromatous(LL)Leprosy

Skinlesions

Oneorafewsharply
IntermediatebetweenBT
definedannular
andLLtypelesions;ill
asymmetricmaculesor definedplaqueswithan
plaqueswithatendency occasionalsharpmargin;
towardcentralclearing, fewormanyinnumber
elevatedborders

Nervelesions

Skinlesionsanesthetic Hypestheticoranesthetic Hypesthesiaalatesign;nerve


early;nervenearlesions skinlesions;nervetrunk
palsiesvariable;acral,distal,
sometimesenlarged;
palsies,attimessymmetric symmetricanesthesiacommon
nerveabscessesmost
commoninBT

Acidfastbacilli
(BIa)

01+

35+

46+

Lymphocytes

2+

1+

01+

Macrophage
differentiation

Epithelioid

EpithelioidinBB;usually
undifferentiated,butmay
havefoamychangesinBL

Foamychangetherule;maybe
undifferentiatedinearlylesions

Langhans'giant
cells

13+

Leprominskin
test

+++

Lymphocyte
transformation
test

Generallypositive

110%

12%

CD4+/CD8+Tcell 1.2
ratioinlesions

BB(NT);BL:0.48

0.50

M.lepraePGL1 60%
antibodies

85%

95%

Symmetric,poorlymarginated,
multipleinfiltratednodulesand
plaquesordiffuseinfiltration;
xanthomalikeor
dermatofibromapapules;leonine
faciesandeyebrowalopecia

(Q.159) Altemeiersprocedureisfor:
(a)

Solitaryrectalulcer

(b)

Rectalprolapse

(c)

Rectalstricture

http://dbmci.examonair.com/Result/ShowAllQuestionInHtml.aspx?testid=3572

79/99

22/03/2016

(d)

DrBhatiaMedicalCoachingInstitute:OnlineTestPlatform

Rectalcarcinoma

YourResponse:

CorrectAnswer:

Exp:

Rectalprolapse.(Ref.LB25thpg.1225)
RECTALPROPLASE
Itmaybemucosalorfullthickness(wholewallincluded).
Commencesasrectalintussusception.
Inchildren,theprolapseisusuallymucosalanshouldbeRxedconservatively.
Intheadults,theprolapseisoftenfullthicknessandisfrequentlyasociatedwithincontinence.
Surgeryisnecessaryforfullthicknessreactlaprolpase.
Theoperationisperformedeitherviaperineum(Thierschoperation,Delormesoperation,&/orAltemeier
opeartion)orviatheabdomen(Wellsoperation,Ripsteinsoperation,orlaparosocpicapproach).
TreatmentofRECTALPROPLASE:
Surgeryisrequired,andtheoperationcanbeperformedviatheperinealortheabdominalapproaches.An
abdominalrectopexyhasalowerrateofrecurrencebut,whenthepatientiselderlyandveryfrail,a
perinealoperationisindicated.Asanabdominalprocedurerisksdamagetothepelvicautonomicnerves,
resultinginpossiblesexualdysfunction,aperinealapproachisalsousuallypreferredinyoungmen.
LISTOFIMPORTANTNAMEDOPERATIONSINGENERALSURGERY:
NAME

USEDFOR

Swensonsoperation,Duhamelsoperation,Soave
operation

Hirschsprungdisease

Nissenstotalfundoplication,Toupetspartial
fundoplication,Hillsprocedure,BelseymarkIV
operation,Angelchickprosthesis

GERD

Dohlmansoperation

Zenkersdiverticulum

Delormesprocedure

Perinealrepairofadultrectalprolapse

AndersonHynesdismemberedpyeloplasty

Cong.PUJobstruction

Nesbittsoperation

Peyroniesdiseaseofpenis

Jaboulaysoperation

Smallandmediumsizedhydrocele

Lordsoperation

Largehydrocele

Hadfieldsoperation

Periductalmastitis/Ductectasia

Trendelenbergoperation

GSVvaricosity

CokketsandDoddsoperation

Subfascialligationofvaricosities

Ivorlewisoperation,Mckeown3stage
operation,Orringerstranshiatalesophagectomy

CaEsophagus

FredetRamstedoperation

CHPS

Thierschoperation,Lockheartmummeryrectopexy

Childhoodrectalprolapse

Thierschoperation,Delormesoperation,Altemiers
operation

AdultrectalprolapsePerinealapproaches

Ripsteinsoperation,Wellsoperation,Lahauts
operation

AdultrectalprolapseAbdominalapproaches

Milesoperation/APR

CaLowerrectum

Hartmannsprocedure

Leftcacolon

Burrhennetechnique

RemovalofresidualstoneinCBD

Grahamsrepair

Perforatedduodenalulcer

Kockspouch

Chroniculcerativecolitis

Puestowsprocedure

Drainageprocedureforchronicpancreatitiswithduct
dilataion

Thomsonsprocedure,LeMuserier
operation,TennissonRandelloperation,Millers
rotationadvancementtechnique

Cleftlip

Allisonsrepair

Hiatushernia

Commandosoperation

CaTongue

Kraskeprocedure

Carectum(postproctotomysphinctersparing)

MiliganMorganoperation

Hemorrhoidectomy

Mayosoperation

Umbilicalhernia

Psoashitchoperation

Uretericreconstruction

Nesovicsoperation

Sportshernia

Cheatlehenryoperation

Obturatrhernia

Sebrocksoperation

Parotidductfistula

Bentallsoperation

Aorticrootaneurysmrepair

Hellerdorsoperation

Achalasiacardia

Laddsprocedure

Midgutvolvulus

LockwoodsoperationLowrepair

Femoralhernia

McEvedyoperationHighrepaiwithvertical

http://dbmci.examonair.com/Result/ShowAllQuestionInHtml.aspx?testid=3572

80/99

22/03/2016

DrBhatiaMedicalCoachingInstitute:OnlineTestPlatform
incision

LotheissenoperationOpeninguinalcanal
Rovsingsoperation

Polcystickidneydis

Brunschwingsoperation

Pelvicexenterationincarectum

A.Perinealapproach
Theseprocedureshavebeenusedmostcommonly.
Thierschoperation
Thisprocedure,whichaimedtoplaceasteelwireor,morecommonly,aSilasticornylonsuturearoundthe
analcanal,hasbecomeobsolete.Thereasonsforitslackofpopularitywerethatthesuturewouldoften
breakorcausechronicperinealsepsis,orboth,ortheanalstenosissocreatedwouldproducesevere
functionalproblems.Delormesoperationisnowthepreferredperinealoperation.
Delormesoperation
Inthisprocedure,therectalmucosaisremovedcircumferentiallyfromtheprolapsedrectumoverits
length.Theunderlyingmuscleisthenplicatedwithaseriesofsutures,suchthat,whenthesearetied,the
rectalmuscleisconcertinaedtowardstheanalcanal.Theanalcanalmucosaisthensutured
circumferentiallytotherectalmucosaremainingatthetipoftheprolapse.Theprolapseisreduced,anda
ringofmuscleiscreatedabovetheanalcanal,whichpreventsrecurrence.
Altemeiersprocedure
Thisconsistsofexcisionoftheprolapsedrectumandassociatedsigmoidcolonfrombelow,and
constructionofacoloanalanastomosis.
(Q.160) Apatientwithleukemiaonchemotherapydevelopsacuterightlowerabdominalpainassociatedwithanemia,thrombocytopenia

andleukopeniA.Whichofthefollowingistheclinicaldiagnosis?
(a)

Appendicitis

(b)

Leukemiccolitis

(c)

Perforationperitonitis

(d)

Neutropeniccolitis

YourResponse:

CorrectAnswer:

Exp:

Neutropeniccolitis
Neutropeniccolitis(ileocecalsyndrome/typhlitis)
InGreektyphlos=blindsac=caecum
Acuteinflammationofcaecum,appendix,andoccasionallyterminalileum,mainlydescribedinchildren
withleukemiaandsevereneutropenia.
Predisposingfactors:
a.Childhoodleukemiab.Aplasticanemiac.Lymphoma
d.Immunosuppressivetherapye.Cyclicneutropeniaf.Myelodysplasticsyndromeg.
AIDS
Organismsresponsible:
a.CMVb.Pseudomonasc.Candidad.Klebsiellae.E.colif.B.fragilis
g.Enterobacter
Complications:perforationandabscessformation

(Q.161) A32yearoldadulthadabdominalinjuryfollowingavehicularinjury.Radiologicalinvestigationsrevealedsevereinjuryto

duodenum,theadjacentpancreaticheadandtheterminalbiliarytractinjury.Whichofthefollowingshallbethecorrectsurgical
approachformanagementofthispatient?
(a)

RouxenYsurgery

(b)

PrimaryDuodenoduodenostomy

(c)

SurgicalRepairofbileductanddamagedpartofduodenum

(d)

Pancreaticoduodenectomy

YourResponse:

CorrectAnswer:

Exp:

Pancreaticoduodenectomy
INTHISPATIENT,thepancreaticduodenalcomplexisseverelydisrupted(gradeVpancreaticinjury),hence
weshouldconsiderpancreaticoduodenectomyi.e.Whipplesoperationforthetreatment

ClassofPancreaticInjuryTreatment

IMinorcontusionsandlacerationsExternaldrainage
IIMajorcontusionsandlacerations(MPDintact)Externaldrainage
Distalpancreatectomy
IIIDistaltransectionormajorductinjuryDistalpancreatectomy(considerspleenpreservation)
IVProximalinjurywithmajorductinjuryExtendeddistalpancreatectomy
VMassivedisruptionofpancreaticheadResect(ordrain)pancreasexcludeduodenum;
pancreaticduodenal
complexconsiderpancreaticoduodenectomy.

http://dbmci.examonair.com/Result/ShowAllQuestionInHtml.aspx?testid=3572

81/99

22/03/2016

DrBhatiaMedicalCoachingInstitute:OnlineTestPlatform

(Q.162) Whichofthefollowingcolonicpathologiesisthoughttohavenomalignantpotential?
(a)

Ulcerativecolitis

(b)

Villousadenomas

(c)

Familialpolyposis

(d)

PeutzJegherssyndrome

YourResponse:

CorrectAnswer:

Exp:

PeutzJegherssyndrome
Cancerofthecolon
Inpatientswithchroniculcerativecolitisis10timesmorefrequentthaninthegeneralpopulation.
Thechanceofdevelopmentofcarcinomaofthecoloninpatientswithfamilialpolyposisisessentially
100%.Treatmentofthepatientwithfamilialpolyposisgenerallyconsistsofsubtotalcolectomywith
ileoproctostomyandregularproctoscopicexaminationoftherectalstump.Villousadenomashavebeen
demonstratedtocontainmalignantportionsinaboutonethirdofaffectedpersonsandinvasive
malignancyinanotheronethirdofremovedspecimens.
PeutzJegherssyndromeischaracterizedbyintestinalpolyposisandmelaninspotsoftheoralmucosa.
Unliketheadenomatouspolypsseeninfamilialpolyposis,thelesionsinthisconditionarehamartomas,
whichhavenomalignantpotential.

(Q.163) A70yearoldwomanhasnausea,vomiting,abdominaldistention,andepisodic,crampymidabdominalpain.Shehasnohistoryof

previoussurgerybuthasalonghistoryofcholelithiasisforwhichshehasrefusedsurgery.Herabdominalradiographrevealsa
sphericaldensityintherightlowerquadrant.Correcttreatmentshouldconsistof?
(a)

Ileocolectomy

(b)

Cholecystectomy

(c)

Ileotomyandextraction

(d)

Nasogastrictubedecompression

YourResponse:

CorrectAnswer:

Exp:

Ileotomyandextraction
Gallstoneileusisduetoerosionofastonefromthegallbladderintothegastrointestinaltract(most
commonlyintotheduodenum).Thestonebecomeslodgedinthesmallbowel(usuallyintheterminal
ileum)andcausessmallbowelobstruction.Plainfilmsoftheabdomenthatdemonstratesmallbowel
obstructionandairinthebiliarytractarediagnosticofthecondition.
Treatmentconsistsofileotomy,removalofthestone,andcholecystectomyifitistechnicallysafe.Ifthere
issignificantinflammationoftherightupperquadrant,ileotomyforstoneextractionfollowedbyan
intervalcholecystectomyisoftenasaferalternative.Operatingonthebiliaryfistuladoublesthemortality
ratecomparedwithsimpleremovalofthegallstonefromtheintestine.

(Q.164) Whichofthefollowingisaproinflammatorymediatorofshock:
(a)

Interleukin4

(b)

Interleukin6

(c)

Interleukin10

(d)

Interleukin13

YourResponse:

CorrectAnswer:

Exp:

Interleukin6
ProinflammatoryAntiinflammatory
IL1/IL4
IL2IL10
IL6IL13
IL8ILIra
IFNPGE2
TNFTGF
PAF
Bacterialexotoxinsandendotoxinsbothstimulateproinflammatorymediatorsbutthecontributionofeach
individualtoxininthereleaseofmediatorscausinglethalshockisincompletelyunderstood.
staphylococcalenterotoxinB(SEB)orlipopolysaccharide(LPS)leadtotheshock.
SEBaloneinducedmoderatelevelsofIL2andMCP1
LPSinduceshighlevelsofIL6andMCP1.
Bacterialexotoxinsandendotoxinsareamongthemostcommonetiologicalagentsthatcausesepticshock
.
StaphylococcalenterotoxinB(SEB)andstructurallyrelatedexotoxinsarebacterialsuperantigensthat
potentlyactivateantigenpresentingcellsbybindingdirectlytomajorhistocompatibilitycomplex(MHC)
classIImolecules.

http://dbmci.examonair.com/Result/ShowAllQuestionInHtml.aspx?testid=3572

82/99

22/03/2016

DrBhatiaMedicalCoachingInstitute:OnlineTestPlatform

TheseexotoxinsalsointeractwithspecificVregionsoftheTcellantigenreceptorsresultinginpolyclonal
Tcellactivation.
InteractionsofsuperantigenswithantigenpresentingcellsandTcellsleadtomassiveproinflammatory
cytokineandchemokinerelease,causingclinicalsymptomsthatincludefever,hypotension,andshock.
Incontrast,lipopolysaccharide(LPS)fromgramnegativebacteriabindstoadifferentreceptoron
monocytes/macrophages.
AnLPSbindingproteininserumfirstbindstoLPSandfacilitatesitsbindingtocellsurfaceproteinCD14on
monocytes/macrophagesandothercells.
ThesubsequentinteractionofLPS/CD14complexwithTolllikereceptor4onthesecellsinitiates
recruitmentofintracellularadaptorsanddownstreamsignalingpathwaysactivatingNFBandresultsin
hyperproductionofproinflammatorycytokinesandchemokines.
REF.SCHWARTZSURGERY8THEDNCHP3PG46

(Q.165) Thetreatmentofhydrofluoricacidskinburnis:
(a)

Injectionofsodiumbicarbonate

(b)

Irrigationwithsodiumbicarbonate

(c)

Applicationofcalciumcarbonategel

(d)

Localwoundcareonly

YourResponse:

CorrectAnswer:

Exp:

Applicationofcalciumcarbonategel
REF.BAILEYANDLOVE25THEDNPG372
1.Theinitialtreatmentaftercontactwiththeskiniscopiousirrigation,whichmustbecontinuedforat
least1530min.witheitherwaterornormalsaline.
2.Thesecondaspectoftreatmentaimstoinactivatethefreefluorideionbypromotingtheformationof
aninsolublefluoridesalt.
3.Topicalcalciumcarbonategelhasbeenshowntodetoxifythefluorideionandrelievepain.
4.Thetreatmentinvolvesmassageofa2.5%calciumcarbonategelintotheareaofexposureforatleast30
min.

(Q.166) AllofthefollowingareTissuespecificendpointsofResuscitationexcept?
(a)

Gastrictonometry

(b)

CO2level

(c)

Nearinfraredspectroscopy

(d)

Cardiacoutput

YourResponse:

CorrectAnswer:

Exp:

Cardiacoutput
RefSchwartz8thEdChapter4
EndpointsinResuscitation
Shockisdefinedasinadequateperfusiontomaintainnormalorganfunction.Withprolongedanaerobic
metabolism,tissueacidosisandoxygendebtaccumulate.Thusthegoalinthetreatmentofshockis
restorationofadequateorganperfusionandtissueoxygenation.Resuscitationiscompletewhenoxygen
debtisrepaid,tissueacidosisiscorrected,andaerobicmetabolismrestored.Clinicalconfirmationofthis
endpointremainsachallenge.
Resuscitationofthepatientinshockrequiressimultaneousevaluationandtreatment;theetiologyofthe
shockoftenisnotinitiallyapparent.Hemorrhagicshock,septicshock,andtraumaticshockarethemost
commontypesofshockencounteredonsurgicalservices.Tooptimizeoutcomeinbleedingpatients,early
controlofthehemorrhageandadequatevolumeresuscitation,includingbothredbloodcellsand
crystalloidsolutions,arenecessary.Expedientoperativeresuscitationismandatorytolimitthemagnitude
ofactivationofmultiplemediatorsystemsandtoabortthemicrocirculatorychanges,whichmayevolve
insidiouslyintothecascadethatendsinirreversiblehemorrhagicshock.Attemptstostabilizeanactively
bleedingpatientanywherebutintheoperatingroomareinappropriate.Anyinterventionthatdelaysthe
patient'sarrivalintheoperatingroomforcontrolofhemorrhageincreasesmortalitythustheimportant
conceptofoperatingroomresuscitationofthecriticallyinjuredpatient.
Recognitionbycareprovidersofthepatientwhoisinthecompensatedphaseofshockisequally
important,butmoredifficultbasedonclinicalcriteria.Compensatedshockexistswheninadequatetissue
perfusionpersists,despitenormalizationofbloodpressureandheartrate.Evenwithnormalizationof
bloodpressure,heartrate,andurineoutput,80to85%oftraumapatientshaveinadequatetissue
perfusionasevidencedbyincreasedlactateordecreasedmixedvenousoxygensaturation.100,101
Persistent,occulthypoperfusionisfrequentintheintensivecareunit,withresultantsignificantincreasein
infectionrateandmortalityinmajortraumapatients.Patientsfailingtoreversetheirlacticacidosiswithin
12hoursofadmission(acidosisthatwaspersistentdespitenormalheartrate,bloodpressure,andurine
output),developedaninfectionthreetimesasoftenasthosewhonormalizedtheirlactatelevelswithin12
hoursofadmission.Inaddition,mortalitywasfourfoldhigherinpatientswhodevelopedinfections.Both
injuryseverityscoreandocculthypotension(lacticacidosis)longerthan12hourswereindependent
predictorsofinfection.102,103Thusrecognitionofsubclinicalhypoperfusionrequiresinformationbeyond
vitalsignsandurinaryoutput.
Endpointsinresuscitationcanbedividedintosystemicorglobalparameters,tissuespecificparameters,and
cellularparameters.Globalendpointsincludevitalsigns,cardiacoutput,pulmonaryarterywedgepressure,
oxygendeliveryandconsumption,lactate,andbasedeficit(Table44).

http://dbmci.examonair.com/Result/ShowAllQuestionInHtml.aspx?testid=3572

83/99

22/03/2016

DrBhatiaMedicalCoachingInstitute:OnlineTestPlatform
EndpointsinResuscitation
Systemic/global
Lactate
Basedeficit
Cardiacoutput
Oxygendeliveryandconsumption
Tissuespecific
Gastrictonometry
TissuepH,oxygen,carbondioxidelevels
Nearinfraredspectroscopy
Cellular
Membranepotential
Adenosinetriphosphate(ATP)

(Q.167) Thehalflifecobalt60is:
(a)

1.2years

(b)

2.3years

(c)

3.4years

(d)

5.3years

YourResponse:

CorrectAnswer:

Exp:

REF.SCHWARTZSURGERY8THEDNPG259263
Ahalflifeistheintervaloftimerequiredforonehalfoftheatomicnucleiofaradioactivesampletodecay
(changespontaneouslyintoothernuclearspeciesbyemittingparticlesandenergy),or,equivalently,the
timeintervalrequiredforthenumberofdisintegrationspersecondofaradioactivematerialtodecrease
byonehalf.
Theradioactiveisotopecobalt60,whichisusedforradiotherapy,has,forexample,ahalflifeof5.26years.
Thusafterthatinterval,asampleoriginallycontaining8gofcobalt60wouldcontainonly4gofcobalt60
andwouldemitonlyhalfasmuchradiation.Afteranotherintervalof5.26years,thesamplewouldcontain
only2gofcobalt60.Neitherthevolumenorthemassoftheoriginalsamplevisiblydecreases,however,
becausetheunstablecobalt60nucleidecayintostablenickel60nuclei,whichremainwiththestill
undecayedcobalt.
Halflivesarecharacteristicpropertiesofthevariousunstableatomicnucleiandtheparticularwayinwhich
theydecay.
Alphaandbetadecayaregenerallyslowerprocessesthangammadecay.
Halflivesforbetadecayrangeupwardfromonehundredthofasecondand,foralphadecay,upwardfrom
aboutonemillionthofasecond.Halflivesforgammadecaymaybetooshorttomeasure(around1014
second),thoughawiderangeofhalflivesforgammaemissionhasbeenreported.

(Q.168) Regardinguretericcalculi,untrueisare:
(a) Aremostoftencomposedofcalciumoxalateorphosphate
(b) Extracorporeallithotripsyisusefulforstonesintheupperthirdoftheureter
(c) About30%ofpatientsrequireopensurgerytoremovethestone
(d) Anobstructedureterinthepresenceofinfectionisasurgicalemergency
YourResponse:

CorrectAnswer:

Exp:

About30%ofpatientsrequireopensurgerytoremovethestone
Uretericcalculi
Factorsfavouringformation
Increasedurinaryconcentrationofconstituents
Presenceofpromotersubstances
Reductioninconcentrationofinhibitors
Lifetimeriskofdevelopingauretericcalculusisabout5%
Occurmostcommonlyinmenagedbetween3060years
90%areidiopathic
10%aredueto:
Hyperparathyroidism
VitaminDexcess
Primaryhyperoxaluria
Chemicalcomposition
Calciumoxalate(40%)
Calciumphosphate(15%)

http://dbmci.examonair.com/Result/ShowAllQuestionInHtml.aspx?testid=3572

84/99

22/03/2016

DrBhatiaMedicalCoachingInstitute:OnlineTestPlatform
Mixedoxalatephosphate(20%)
Struvite(15%)
Uricacid(10%)
Clinicalfeatures
Stonesusuallypresentwithpainduetoobstructionofurinaryflow
Maycausefewsymptomsormaypresentwithtypicaluretericcolic
Uretericcolictypicallyisseverecolickylointogroinpain
Painmayradiateintoscrotuminmenandlabiainwomen
Mayalsocausefrequency,urgencyanddysuria
Painmaysettlewithpassageofthestoneorifstonefailstomigrate
Abdominalexaminationisusuallyunremarkable
Microscopichematuriaisoftenpresent
Investigation
Midstreamurinespecimen
KUBplusultrasound
Intravenousurography(IVU)
Complications
Obstruction
Uretericstrictures
Infection
Acuteinfectioninanobstructedkidneyisaurologicalemergency
Patientisusuallyunwellwithloinpain,swingingpyrexiaanddysuria
Withoutdrainage,rapidrenaldestructionmayoccur
Requiresemergencypercutaneousnephrostomy
Chronicinfectionwithureaseproducingorganisms(e.g.Proteus)precipitates
stoneformation
Magnesiumammoniumphosphateorstaghorncalculiresult
Largestaghorncalculimaybeasymptomatic
Staghorncalculicanleadtodeteriorationinrenalfunction
Management
Initialconservativetreatmentwithoralfluidsandadequateanalgesia
Checkserumelectrolytesandcalcium
Urinalysiswillnormallyshowmicroscopichematuria
IVUtoconfirmdiagnosisanduretericobstruction
Moststones<5mmindiameterpassspontaneously
Ifmorethan510mmindiameterandfailtopassspontaneouslyconsider:
Upperthirdofureterextracorporealshockwavelithotripsy(ESWL)
Lowerthirdofureterureteroscopy(USC)+lithotripsy
MiddlethirdofuretereitherESWLorUSC
Iftotalobstructionoccursinthepresenceofinfectedurineneedurgent
decompression
Decompressionusuallyachievedwithpercutaneousnephrostomy
Iflargestonesinrenalpelvisorupperureterconsiderpercutaneous
nephrolithotomy
Particularlyifstonemorethan3cmindiameterora'staghorncalculus'

Lessthan1%patientswithstonesrequireopensurgeryureteroor
nephrolithotomy
Lithotripsy
Istheuseofshockwavestobreakupstones
Lithotripsyrequires:
Anenergysourcesparkgapelectrodeorpiezoceramicarray
Acouplingdevicebetweenpatientandelectrodewaterbathorcushion
Amethodofstonelocalisationfluoroscopyorultrasound
(Q.169) Propertreatmentforfrostbiteconsistsof
(a)

Debridementoftheaffectedpartfollowedbysilversulfadiazinedressings

(b)

Administrationofcorticosteroids&vasodilators

(c)

Immersionoftheaffectedpartinwaterat4044C(104111.2F)

(d)

Rewarmingoftheaffectedpartatroomtemperature

YourResponse:

CorrectAnswer:

Exp:

(Greenfield,2/e,pp412414.)
Manymethodsoftreatingfrostbitehavebeentriedthroughouttheyears.Theseincludemassage,warm
waterimmersion,orcoveringtheaffectedarea.Rapidwarmingbyimmersioninwaterslightlyabove
normalbodytemperature(4044C)isthemosteffectivemethod;however,becausethefrostbittenregion
isnumbandespeciallyvulnerable,itshouldbeprotectedfromtraumaorexcessiveheatduringtreatment.

http://dbmci.examonair.com/Result/ShowAllQuestionInHtml.aspx?testid=3572

85/99

22/03/2016

DrBhatiaMedicalCoachingInstitute:OnlineTestPlatform

Furthertreatmentmayincludeelevationtominimizeedema,administrationofantibioticsandtetanus
toxoid,anddebridementofnecroticskinasneeded.
REFHARRISON17thEdChapter20.
Frostbite:Treatment
Frozentissueshouldberapidlyandcompletelythawedbyimmersionincirculatingwaterat3740C.
Rapidrewarmingoftenproducesaninitialhyperemia.Theearlyformationoflargecleardistalblebsis
morefavorablethansmallerproximaldarkhemorrhagicblebs.Acommonerroristhepremature
terminationofthawing,sincethereestablishmentofperfusionisintenselypainful.Parenteralnarcoticswill
benecessarywithdeepfrostbite.Ifcyanosispersistsafterrewarming,thetissuecompartmentpressures
shouldbemonitoredcarefully.
Numerousexperimentalantithromboticandvasodilatorytreatmentregimenshavebeenevaluated.There
isnoconclusiveevidencethatdextran,heparin,steroids,calciumchannelblockers,hyperbaricoxygen,or
prostaglandininhibitorssalvagetissue.AtreatmentprotocolforfrostbiteissummarizedinTable204.
TreatmentforFrostbite
BeforeThawing

DuringThawing

AfterThawing

Removefrom
environment

Considerparenteralanalgesiaand
ketorolac

Gentlydryandprotectpart;elevate;
pledgetsbetweentoes,ifmacerated

Preventpartial
thawingand
refreezing

Administeribuprofen,400mgPO

Ifclearvesiclesareintact,aspirate
sterilely;ifbroken,debrideanddress
withantibioticorsterilealoevera
ointment

Stabilizecore
temperatureand
treathypothermia

Immersepartin3740C
(thermometermonitored)circulating
watercontaininganantisepticsoap
untildistalflush(1045min)

Leavehemorrhagicvesiclesintactto
preventdessicationandinfection

Protectfrozenpart Encouragepatienttogentlymovepart
nofrictionor
massage

Continueibuprofen400mgPO(12
mg/kgperday)q812h

Addressmedicalor
surgicalconditions

Considertetanusandstreptococcal
prophylaxis;elevatepart

Ifpainisrefractory,reducewater
temperatureto3537Cand
administerparenteralnarcotics

Hydrotherapyat37C
Considerphenoxybenzamineinsevere
cases

Unlessinfectiondevelops,anydecisionregardingdebridementoramputationshouldbedeferreduntil
thereisclearevidenceofdemarcation,mummification,andsloughing.Magneticresonanceangiography
maydemonstratethelineofdemarcationearlierthanclinicaldemarcation.Themostcommon
symptomaticsequelaereflectneuronalinjuryandthepersistentlyabnormalsympathetictone,including
paresthesias,thermalmisperception,andhyperhidrosis.Delayedfindingsincludenaildeformities,
cutaneouscarcinomas,andepiphysealdamageinchildren.
Managementofthechilblainsyndromeisusuallysupportive.Withrefractoryperniosis,alternativesinclude
nifedipine,steroids,orlimaprost,aprostaglandinE1analogue
(Q.170) A43yroldfemaleisdiagnosedhavingsuperficial,gradeIItransitionalcellcarcinomaoftheurinarybladder.Whichofthefollowing

isnottrueaboutregardingthispatient?
(a)

Smokingpredisposestothiscancer

(b)

Recurrencewilloccurforwhichshemayneedrepeatedfulguration

(c)

Therearenoeffectiveanticancerdrugsforbladdercancer

(d)

Thereis70%chancethatsheeventuallywillneedcystectomyinnext5yrs.

YourResponse:

CorrectAnswer:

Exp:

Therearenoeffectiveanticancerdrugsforbladdercancer.(Ref.Harrisons17th/pg.591)
Epidemiology
Cigarettesmokingisbelievedtocontributetoupto50%ofthediagnosedurothelialcancersinmenandup
to40%inwomen.Theriskofdevelopingaurothelialmalignancyinmalesmokersisincreasedtwoto
fourfoldrelativetononsmokersandcontinuesfor10yearsorlongeraftercessation.Otherimplicated
agentsincludetheanilinedyes,thedrugsphenacetinandchlornaphazine,andexternalbeamradiation.
Chroniccyclophosphamideexposuremayalsoincreaserisk,whereasvitaminAsupplementsappeartobe
protective.ExposuretoSchistosomahaematobium,aparasitefoundinmanydevelopingcountries,is
associatedwithanincreaseinbothsquamousandtransitionalcellcarcinomasofthebladder
AnumberofchemotherapeuticdrugsinUBcancerhaveshownactivityassingleagents;cisplatin,
paclitaxel,andgemcitabineareconsideredmostactive.Standardtherapyconsistsoftwo,three,orfour
drugcombinations.Overallresponseratesof>50%havebeenreportedusingcombinationssuchas
methotrexate,vinblastine,doxorubicin,andcisplatin(MVAC);cisplatinandpaclitaxel(PT);gemcitabine
andcisplatin(GC);orgemcitabine,paclitaxel,andcisplatin(GTC).MVACwasconsideredstandard,butthe
toxicitiesofneutropeniaandfever,mucositis,diminishedrenalandauditoryfunction,andperipheral
neuropathyledtothedevelopmentofalternativeregimens.Atpresent,GCisusedmorecommonlythan
MVAC,basedontheresultsofacomparativetrialofMVACversusGCthatshowedlessneutropeniaand
fever,andlessmucositisfortheGCregimen.AnemiaandthrombocytopeniaweremorecommonwithGC.
GTCisnotmoreeffectivethanGC.
Chemotherapyhasalsobeenevaluatedintheneoadjuvantandadjuvantsettings.Inarandomizedtrial,

http://dbmci.examonair.com/Result/ShowAllQuestionInHtml.aspx?testid=3572

86/99

22/03/2016

DrBhatiaMedicalCoachingInstitute:OnlineTestPlatform

patientsreceivingthreecyclesofneoadjuvantMVACfollowedbycystectomyhadasignificantlybetter
median(6.2years)and5yearsurvival(57%)comparedtocystectomyalone(mediansurvival3.8years;5
yearsurvival42%).Similarresultswereobtainedinaninternationalstudyofthreecyclesofcisplatin,
methotrexate,andvinblastine(CMV)followedbyeitherradicalcystectomyorradiationtherapy.The
decisiontoadministeradjuvanttherapyisbasedontheriskofrecurrenceaftercystectomy.Indicationsfor
adjuvantchemotherapyincludethepresenceofnodaldisease,extravesicaltumorextension,orvascular
invasionintheresectedspecimen.AnotherstudyofadjuvanttherapyfoundthatfourcyclesofCMV
delayedrecurrence,althoughaneffectonsurvivalwaslessclear.Additionaltrialsarestudyingtaxaneand
gemcitabinebasedcombinations.

(Q.171) AGrayhackshuntis?
(a) Anastomosisofasaphenousveintooneofthecorporacavernosa
(b) Cavernosalspongiosumshunt
(c) Incisionontheglanswithcommunicationofthecorpospongiosumwiththecorpocavernosum
(d) Placementoftubeincorpospongiosum
YourResponse:

CorrectAnswer:

Exp:

Anastomosisofasaphenousveintooneofthecorporacavernosa(Ref.Swartz8th/1550)
Grayhackshuntissaphenousveincavernousshunt.
Priapismisararedisorderinwhichprolonged,painfulerectionoccurs,usuallynotassociatedwithsexual
stimulation.Thebloodinthecorporacavernosabecomeshyperviscousbutnotclotted.About25%ofcases
areassociatedwithleukemia,metastaticcarcinoma,sicklecellanemia,ortrauma.Inmostcases,thecause
isuncertain.
Rx:
Iftheerectiondoesnotsubside,needleaspirationofthesludgedbloodofthecorporafollowedbylavage
withalphaadrenergicagentssuchasphenylephrineshouldbeperformed.Delayedorunsuccessful
treatmentmayresultinimpotence.UnsuccessfultreatmentcallsfortheWinterprocedure,inwhicha
biopsyneedleispassedthroughtheglansintooneofthecorpora.Apieceoftunicaalbugineaisremoved,
creatingafistulabetweencorporacavernosaandcorpusspongiosum.Thissimpleprocedureishighly
successful,andpotencyisusuallymaintained.Otherproceduresincludecavernosalglandularshunt,
cavernosalspongiosumshunt,andsaphenousveincavernousshunt.Ifpriapismpersists,impotence
results.
Insicklecellanemia,hydrationandhypertransfusionoftengivereliefandshouldconstituteinitialtherapy.
Surgicalinterventionisusedonceitisapparentthatintracavernoustreatmenthasfailed.Surgicalshunting
asameansforblooddrainagefromthecorporacavernosainvolveseitherdistalorproximalapproaches.
Distalcavernoglanular(corporoglanular)shuntingisusuallyperformedfirstbecauseitislessinvasiveand
carriesalowerriskofcomplicationsthanproximalshunting.
Distalshuntingtechniquesincludeplacingalargebiopsyneedle(Wintershunt)orscalpel(Ebbehojshunt)
percutaneouslythroughtheglans,orexcisingthetunicaalbugineaatthetipofthecorpuscavernosum(El
Ghorabshunt).
Ifdistalshuntingfails,proximalcavernospongiosal(corporospongiosal)shuntingcanbeused.
Thisinvolvesthecreationofawindowbetweentherespectivecorporalbodies(QuackelsorSachershunt),
orananastomosisofasaphenousveintooneofthecorporacavernosa(Grayhackshunt).

(Q.172) Laparoscopicsterilizationiscontraindicatedin:
(a)

Postpartum

(b)

Gynecologictumors

(c)

FollowingMTP

(d)

Ifpatienthasmorethan3children.

YourResponse:

CorrectAnswer:

Exp:

Postpartum
Contraindicationsoflaparoscopicsterilization:
Patientwithcardiacorpulmonarydisease,headlowpositionandCo2arecontraindicated
Previousabdominalsurgeryexposesthepatienttotheriskofintestinaltraumaincaseparietaladhesions
arepresent.
Extremeobesity,diaphragmaticorumbilical.
InPID,fallopiantubesmaynotbeeasilyvisibleamongsttheadhesions.
Puerperalandvascular:Thefallopiantubesareedematousandvascularandmayeasilygettorn.The
uterusissoftandcaneasilygetperforatedwiththeuterinemanipulator

(Q.173) Forinducingtherapeuticabortion,Mifepristoneismosteffectivewhengivenwithin_______daysofpregnancy
(a)

120days

(b)

88days

(c)

72days

(d)

63days

YourResponse:

CorrectAnswer:

http://dbmci.examonair.com/Result/ShowAllQuestionInHtml.aspx?testid=3572

87/99

22/03/2016

DrBhatiaMedicalCoachingInstitute:OnlineTestPlatform

Exp:

63days

(Ref.TextbookofGynaecology5th/pg.511,Duttaobstetrics6th/pg.175&550)
BuccaluseofMifepristoneisaseffectiveasvaginalininducedmedicalabortionupto63daysgestation.
RegimensfortreatmentofMedicalabortion
Themostcommonearlyfirsttrimestermedicalabortionregimensusemifepristoneincombinationwitha
prostaglandinanalog(i.e.misoprostol)upto63daysofgestationalage,methotrexateincombinationwith
aprostaglandinanalog(i.e.misoprostol)upto49daysgestation,oraprostaglandinanalog(i.e.
misoprostol)alonewhere.
Mifepristonemisoprostolcombinationregimensworkfasterandaremoreeffectiveatlatergestational
agesthanmethotrexatemisoprostolcombinationregimens.
(Q.174) Markerofopenneuraltubedefect?
(a)

Amnioticfluidacetylcholineesterase

(b)

Amnioticfluidacetylcholinekinase

(c)

AmnioticfluidHCG

(d)

Amnioticfluidpseudoacetylcholineesterase

YourResponse:

CorrectAnswer:

Exp:

Amnioticfluidacetylcholineesterase
(Ref.DuttaObstetrics6th/107;WilliamsObstetrics
22nd/Page278)
Ifpresentafter14weeksofgestation,amnioticfluidacetylcholinesteraseisconsidereddirectevidence
ofanopenneuraltubedefect.
Normalrangebymethod:
oPolyacrylamidegelelectrophoresis:Negative
oAE2immunoassay:Negative
oInhibitionassay:5.172.63milliunits/mL(5.172.63units/L)

(Q.175) Placentalalkalinephosphataseismarkerof
(a)

Thecacelltumour

(b)

Teratoma

(c)

Choriocarcinoma

(d)

Seminoma

YourResponse:

CorrectAnswer:

Exp:

i.e.seminoma
Recently,placentalalkalinephosphatasehasbeensuggestedastumormarkerinpatientwith
seminoma.
Someothertumourmarkersare
TumorMarkers
Cancer

NonneoplasticConditions

Humanchorionic
gonadotropin

Gestationaltrophoblasticdisease,
gonadalgermcelltumor

Pregnancy

Calcitonin

Medullarycancerofthethyroid

Catecholamines

Pheochromocytoma

TumorMarkers
Hormones

OncofetalAntigens
Fetoprotein

Hepatocellularcarcinoma,gonadal
germcelltumor

Cirrhosis,hepatitis

Carcinoembryonic
antigen

Adenocarcinomasofthecolon,
pancreas,lung,breast,ovary

Pancreatitis,hepatitis,
inflammatoryboweldisease,
smoking

Prostaticacid
phosphatase

Prostatecancer

Prostatitis,prostatic
hypertrophy

Neuronspecific
enolase

Smallcellcancerofthelung,
neuroblastoma

Enzymes

Lactatedehydrogenase Lymphoma,Ewing'ssarcoma

Hepatitis,hemolyticanemia,
manyothers

TumorAssociatedProteins
Prostatespecific
antigen

Prostatecancer

Prostatitis,prostatic
hypertrophy

Monoclonal

Myeloma

Infection,MGUS

http://dbmci.examonair.com/Result/ShowAllQuestionInHtml.aspx?testid=3572

88/99

22/03/2016

DrBhatiaMedicalCoachingInstitute:OnlineTestPlatform
immunoglobulin
CA125

Ovariancancer,somelymphomas

Menstruation,peritonitis,
pregnancy

CA199

Colon,pancreatic,breastcancer

Pancreatitis,ulcerativecolitis

CD30

Hodgkin'sdisease,anaplasticlarge
celllymphoma

CD25

Hairycellleukemia,adultTcell
leukemia/lymphoma

(Q.176) Thefollowingaretruestatementsregardingsalicylatepoisoningexcept:
(a)

Hypokalemiaisafeature

(b)

Hypoventilationisanearlyfeature

(c)

Vasodilatationisafeature

(d)

Respiratoryalkalosisisseeninchildren

YourResponse:

CorrectAnswer:

Exp:

Hypoventilationisanearlyfeature
Hypoventilationisalatefeatureofsalicylatepoisoning,andtogetherwithcomaindicateseverpoisoning.
Theacidbasedisturbancesarecomplexandbothacidosisandalkalosismayoccur.Salicylateshaveadirect
stimulanteffectontherespiratorycentreresultinginhyperventilationandarespiratoryalkalosisseen
earlyon,thoughthisstagemaynotbeapparentinyounginfants.

(Q.177) Allofthefollowingaretypesofavascularnonunionoffractureexcept:
(a)

Torsionwedgenonunion

(b)

Oligotrophicnonunion

(c)

Comminutednonunion

(d)

Defectnonunion

YourResponse:

CorrectAnswer:

Exp:

Oligotrophicnonunion(Ref.TextbookofforthopedicsbyJohnEbnezar3rded.36)
Definition(FDApanel)ofnonunion:Itissaidtobeestablishedwhenaminimumof9monthshaselapsed
sinceinjuryandfractureshowsnoradiologicallyvisibleprogresssuggestiveofhealingcontinuouslyfor3
months.
Classification(MullerandWeber)ofNONUNION:
HypervasculartypesAvasculartypes
Hypertrophic(elephantfoot,exuberantcallus)Torsionwedge
HorsehoofComminuted
OligotrophicDefect

(Q.178) WhichofthefollowingisnottrueregardingMolluscacontagiosa?:
(a)

Causedbyapoxvirus

(b)

Seenasapearlypapulewithacentralumbilicus

(c)

Spontaneousresolutionisusualby6weeks

(d)

Cryotherapyessentialinalmostallcases

YourResponse:

CorrectAnswer:

Exp:

Cryotherapyessentialinalmostallcases
Molluscacontagiosaisacommoninfectionamongschoolchildrencausedbyapoxvirus.Itpresentsas
pearlypapuleswithacentralumbilicus.Spontaneousresolutionusuallyoccurswithin69months,though
thelesionscanlastforyears.Disseminatedinfectionmaybeseeninchildrenwithatopiceczemaandinthe
immunosuppressed.Notreatmentisusuallygivenasthiscanresultinscarring,however,cryotherapyisa
treatmentoptionifnecessary.

(Q.179) Falseaboutsynovialfluidis?
(a)

TypeBcellsproducesynovialfluiD.

(b)

Synovialfluidhasthixotropiccharacteristics

(c)

SynovialfluidexhibitsNewtonianflowcharacteristics

(d)

Containslubricin

YourResponse:

CorrectAnswer:

Exp:

SynovialfluidexhibitsNewtonianflowcharacteristics
Synovialtissueissterileandcomposedofvascularizedconnectivetissuethatlacksabasementmembrane.

http://dbmci.examonair.com/Result/ShowAllQuestionInHtml.aspx?testid=3572

89/99

22/03/2016

DrBhatiaMedicalCoachingInstitute:OnlineTestPlatform

Twocellstype(typeAandtypeB)arepresent:TypeBproducesynovialfluid.
Synovialfluidismadeofhyaluronicacidandlubricin,proteinases,andcollagenases.
SynovialfluidexhibitsnonNewtonianflowcharacteristics;theviscositycoefficientisnotaconstantandthe
fluidisnotlinearlyviscous.
Synovialfluidhasthixotropiccharacteristics;viscositydecreasesandthefluidthinsoveraperiodof
continuedstress.
Normalsynovialfluidcontains34mg/mlhyaluronan(hyaluronicacid),apolymerofdisaccharidescomposed
ofDglucuronicacidandDNacetylglucosaminejoinedbyalternatingbeta1,4andbeta1,3glycosidic
bonds
Hyaluronanissynthesizedbythesynovialmembraneandsecretedintothejointcavitytoincreasethe
viscosityandelasticityofarticularcartilagesandlubricatethesurfacesbetweensynoviumandcartilage.
Synovialfluidcontainslubricinsecretedbysynovialcells.Itischieflyresponsibleforsocalledboundary
layerlubrication,whichreducesfrictionbetweenopposingsurfacesofcartilage.
Thereisalsosomeevidencethatithelpsregulatesynovialcellgrowth.
Itsfunctionsare:reducingfrictionbylubricatingthejoint,absorbingshocks,andsupplyingoxygenand
nutrientstoandremovingcarbondioxideandmetabolicwastesfromthechondrocyteswithinarticular
cartilage.
Italsocontainsphagocyticcellsthatremovemicrobesandthedebristhatresultsfromnormalwearand
tearinthejoint.
Synovialfluidcanbecollectedbysyringeinaproceduretermedarthrocentesis,alsoknownasjoint
aspiration.
Classification
Synovialfluidcanbeclassifiedintonormal,noninflammatory,inflammatory,septic,andhemorrhagic:
Classificationofsynovialfluidinanadultkneejoint
Normal

Noninflammatory

Inflammatory

Septic

Hemorrhagic

Volume(ml)

<3.5

>3.5

>3.5

>3.5

>3.5

Viscosity

High

High

Low

Mixed

Low

Clarity

Clear

Clear

Cloudy

Opaque

Mixed

Color

Colorless/straw

Straw/yellow

Yellow

Mixed

Red

WBC/mm3

<200

2002,000

2,00075,000

>100,000

Sameasblood

Polys(%)

<25

<25

>50

>75

Sameasblood

Gramstain

Negative

Negative

Negative

Oftenpositive Negative

(Q.180) Ketaminebelongstowhichofthefollowinggroupofdrug?
(a)

Phencyclidine

(b)

Phenols

(c)

Barbiturate

(d)

Benzodiazepine

YourResponse:

CorrectAnswer:

Exp:

Phencyclidine(Ref.AnaesthesiabyAjayYadav,3rded.,p78)
KETAMINE(dissociativeanaesthesia)
Phencyclidine(PCP)derivative.
Primarysiteofactionisthalamoneocorticalprojection.
Fastonset.
Doserelatedunconsciousnessanddissociativestateanalgesia.
Stronganalgesic.
Notasamnesticasbenzodiazepines.
Nosuppressionofthecardiovascularandrespiratorysystems.
Maycausehallucinationsanddisturbingdreams,especiallyinadultsandhallucinationisthemostcommon
sideeffectketamine.
Intracranialtensionishighlyincreased.
Potentbronchodilator.
Dilatespupilsandtriggersnystagmus.
Increasedsalivation.
Increasedmuscletoneandpurposelessmovementsoftheextremities
Increasedcerebralbloodflowandintracranialpressure.

(Q.181) Ahighpotassiumlevelisseeninwhichofthefollowingconditionwhenscolineisbeingusedforintubation
(a)

Renalfailure

(b)

Chronicparaplegia

(c)

Fracturefemur

(d)

Myocardialinfarction

YourResponse:

http://dbmci.examonair.com/Result/ShowAllQuestionInHtml.aspx?testid=3572

90/99

22/03/2016

DrBhatiaMedicalCoachingInstitute:OnlineTestPlatform

CorrectAnswer:

Exp:

Chronicparaplegia(Ref.Leeanaesthesia12thed.220)
Scolin/succinylcholinehasbeenassociatedwithrapidmusclebreakdownandresultsinincreasein
potassiumlevelscausinglifethreateningheartrythumsandcardiacarrests.
Itisusedforintubationasamusclerelaxantbutoneshouldtakecarefulhistoryofpatient.
ChronicparaplagicshaveweakmusclesanditcanreleaseextremelevelsofK+withscolin
Fracturefemure/renalfailure/MIallhashighpotassiumbutthecauseisdifferentitsnotscolin
Scolincausesriseofchronicparaplegicsbycausingmusclebreakdown.
Certaingroupsofpatientsexperienceproliferationof"extrajunctional"acetylcholinereceptorswhich
occuroutsidethemarginsofthemotorendplate.Suxamethoniumcausesalargerthanusualriseinserum
potassium.Thisriseisoftheorderof23mmol/L,buthasbeenreportedashigher.Itvarieswiththe
natureoftheconditioninvolved,itsage,andseverity.
Theconditionsinwhichithasbeendescribedinclude:
BURNS.A1060dayrulehasbeendescribedsubsequenttoreportsofthephenomenonbetweendays14
66afterburns.Asinotherconditions,alatentphaseoccurs;"extrajunctional"receptorsdonotappear
immediatelyafterinjury.
ACUTEUPPERMOTORNEURONLESIONS&STRUCTURALBRAINDAMAGE.Theseincludespinalcordinjury,
anoxicbraindamage,encephalitis,subarachnoidhaemorrhage,andcerebrovascularaccidents.
PROGRESSIVENEUROLOGICALCONDITIONS.Demyelinatingdiseases,andbraintumoursmaybeassociated
withtheresponse.Againthiswillbegreaterifthediseaseisrapidlyprogressive,andingeneral,the
potentialforevokedreleaseofpotassiumisofalowermagnitudeandoccursoveralongerperiodoftime.
TherelationshipinParkinson'sdiseaseisnotclear.
CHRONICSTABLENEUROLOGICALCONDITIONS.Thesearegenerallyassociatedwitha"normal"response
only,includingcerebralpalsy.
LOWERMOTORNEURONDISEASE.Thephenomenomhasbeendescribedbetweendays42192following
acuteperipheralnervedamage,andcardiacarresthasbeenreportedfollwingsuxamethoniumwithdiffuse
neuropathies.
MUSCULARDYSTROPHIES.HyperkalemiahasbeenreportedinpatientswithDuchenne'smuscular
dystrophy,however,thiswasascribedtoMalignantHyperpyrexia.
TRAUMA.Aswithburns,thisoccursinmannerinfluencedbythetimefrominjury,andtheextentofnerve
andmuscledamage.
INFECTION.Again,thisresponsehasbeenseeninsevereinfectionincludingosteomyelitisandgangrene.

(Q.182) Siteofactionofvecuronium?
(a)

Cerebrum

(b)

Reticularformation

(c)

Motorneuron

(d)

Myoneuraljunction

YourResponse: d
CorrectAnswer: D
Exp:

Myoneuraljunction(Ref.KDT5thed.309)
Vecuronium
Excretedinbile
Notusedinbiliaryobstruction
CARDIOVASCULARstability,henceagentofchoiceincardiacpatients
Longtermuseresultsinpolyneuropathy
Longacting,Nondepolarisingmusclerelaxant.
pHofsolution4.Stablefor24hoursat25oc.(Muchmorestable
thanAtracurium)
Dose0.Img/kg.
Onsetofaction12min.
Durationofactionabout20min.
Siteofactionismyoneuraljunction.

(Q.183) Whichofthefolwoingindicatesradiographcontrastinducednephropathy?
(a)

Increasedcreatininelevels

(b)

Decreasedurineoutput

(c)

Increasedbilirubin

(d)

Decreasedbilirubin

YourResponse:

CorrectAnswer:

Exp:

Increasedcreatininelevels
REF:
1.http://emedicine.medscape.com/article/246751overview:ContrastInducedNephropathyAuthor:
RenuBansal,MD;ChiefEditor:VecihiBatuman,

http://dbmci.examonair.com/Result/ShowAllQuestionInHtml.aspx?testid=3572

91/99

22/03/2016

DrBhatiaMedicalCoachingInstitute:OnlineTestPlatform

2.AmericanJournalofRoentgenology.2004;183:16731689.10.2214/ajr.183.6.01831673
Contrastinducednephropathy(CIN)isdefinedastheimpairmentofrenalfunctionandismeasuredas
eithera25%increaseinserumcreatinine(SCr)frombaselineor0.5mg/dL(44mol/L)increasein
absolutevalue,within4872hoursofintravenouscontrastadministration.
Forrenalinsufficiency(RI)tobeattributabletocontrastadministration,itshouldbeacute,usuallywithin
23days,althoughithasbeensuggestedthatRIupto7dayspostcontrastadministrationbeconsidered
CIN;itshouldalsonotbeattributabletoanyotheridentifiablecauseofrenalfailure.Atemporallinkisthus
implied.Followingcontrastexposure,SCrlevelspeakbetween2and5daysandusuallyreturntonormalin
14days.
Complications
CINisoneoftheleadingcausesofhospitalacquiredacuterenalfailure.Itisassociatedwithasignificantly
higherriskofinhospitaland1yearmortality,eveninpatientswhodonotneeddialysis.Nonrenal
complicationsincludeproceduralcardiaccomplications(eg,QwaveMI,coronaryarterybypassgraft
[CABG],hypotension,shock),vascularcomplications(eg,femoralbleeding,hematoma,pseudoaneurysm,
stroke),andsystemiccomplications(eg,acuterespiratorydistresssyndrome[ARDS],pulmonary
embolism).ThereisacomplicatedrelationshipbetweenCIN,comorbidity,andmortality.Mostpatients
whodevelopCINdonotdiefromrenalfailure.Death,ifitdoesoccur,ismorecommonlyfromeithera
preexistingnonrenalcomplicationoraproceduralcomplication.
Otherrenalfunctionmarkers
TheuseofSCrasamarkerofrenalfunctionhasitslimitations.Indicatorssuchastheestimatedglomerular
filtrationrate(eGFR)andcystatinCareincreasinglyconsideredtobemorereliableandaccuratereflectors
ofexistingrenalfunction
TheeGFRcanbecalculatedusingtheModificationofDietinRenalDisease(MDRD)formulaorthe
CockroftGaultformula.TheCockroftGaultformulacalculateseGFRusingage,sex,andbodyweight,which
arefactorsthat,independentofGFR,influenceSCr.TheMDRDequationalsoincludesbloodureanitrogen
(BUN)andserumalbumin.
TheeGFRworksbestatlowcreatininevalues.SCrandGFRshareacurvilinearrelationship.AtlowerSCr
values,doublingSCrisassociatedwithacorresponding50%decreaseinGFR.However,inelderlypatients
withchronickidneydisease(CKD)whohavehighSCrvaluesatbaseline,a25%riseinSCrisactually
indicativeofarelativelymodestreductioninGFR.Nonetheless,evena25%increaseinSCrinthissituation
hasbeenshowntohavegreatimpact,especiallyintermsofinhospitaland1yearmortality
SerumcystatinCisaserumproteinthatissecretedbynucleatedcells.Itisfreelyfilteredbytheglomerulus
andhasbeenfoundtobeanaccuratemarkerofGFR.ComparedwithSCr,cystatinCchangesmuchearlier
aftercontrastadministrationandisnotsubjecttoconfoundingfactors,suchage,sex,andmusclemass,
thatinfluenceSCrvaluesindependentoftheunderlyingGFR.CystatinCisincreasinglybeingusedasa
markerofrenalfunctionincardiacsurgicalpatients.
1.http://emedicine.medscape.com/article/246751overview:ContrastInducedNephropathyAuthor:
RenuBansal,MD;ChiefEditor:VecihiBatuman,
2.AmericanJournalofRoentgenology.2004;183:16731689.10.2214/ajr.183.6.01831673
Contrastinducednephropathy(CIN)isdefinedastheimpairmentofrenalfunctionandismeasuredas
eithera25%increaseinserumcreatinine(SCr)frombaselineor0.5mg/dL(44mol/L)increasein
absolutevalue,within4872hoursofintravenouscontrastadministration.
Forrenalinsufficiency(RI)tobeattributabletocontrastadministration,itshouldbeacute,usuallywithin
23days,althoughithasbeensuggestedthatRIupto7dayspostcontrastadministrationbeconsidered
CIN;itshouldalsonotbeattributabletoanyotheridentifiablecauseofrenalfailure.Atemporallinkisthus
implied.Followingcontrastexposure,SCrlevelspeakbetween2and5daysandusuallyreturntonormalin
14days.
Complications
CINisoneoftheleadingcausesofhospitalacquiredacuterenalfailure.Itisassociatedwithasignificantly
higherriskofinhospitaland1yearmortality,eveninpatientswhodonotneeddialysis.Nonrenal
complicationsincludeproceduralcardiaccomplications(eg,QwaveMI,coronaryarterybypassgraft
[CABG],hypotension,shock),vascularcomplications(eg,femoralbleeding,hematoma,pseudoaneurysm,
stroke),andsystemiccomplications(eg,acuterespiratorydistresssyndrome[ARDS],pulmonary
embolism).ThereisacomplicatedrelationshipbetweenCIN,comorbidity,andmortality.Mostpatients
whodevelopCINdonotdiefromrenalfailure.Death,ifitdoesoccur,ismorecommonlyfromeithera
preexistingnonrenalcomplicationoraproceduralcomplication.
Otherrenalfunctionmarkers
TheuseofSCrasamarkerofrenalfunctionhasitslimitations.Indicatorssuchastheestimatedglomerular
filtrationrate(eGFR)andcystatinCareincreasinglyconsideredtobemorereliableandaccuratereflectors
ofexistingrenalfunction
TheeGFRcanbecalculatedusingtheModificationofDietinRenalDisease(MDRD)formulaorthe
CockroftGaultformula.TheCockroftGaultformulacalculateseGFRusingage,sex,andbodyweight,which
arefactorsthat,independentofGFR,influenceSCr.TheMDRDequationalsoincludesbloodureanitrogen
(BUN)andserumalbumin.
TheeGFRworksbestatlowcreatininevalues.SCrandGFRshareacurvilinearrelationship.AtlowerSCr
values,doublingSCrisassociatedwithacorresponding50%decreaseinGFR.However,inelderlypatients
withchronickidneydisease(CKD)whohavehighSCrvaluesatbaseline,a25%riseinSCrisactually
indicativeofarelativelymodestreductioninGFR.Nonetheless,evena25%increaseinSCrinthissituation
hasbeenshowntohavegreatimpact,especiallyintermsofinhospitaland1yearmortality
SerumcystatinCisaserumproteinthatissecretedbynucleatedcells.Itisfreelyfilteredbytheglomerulus
andhasbeenfoundtobeanaccuratemarkerofGFR.ComparedwithSCr,cystatinCchangesmuchearlier
aftercontrastadministrationandisnotsubjecttoconfoundingfactors,suchage,sex,andmusclemass,
thatinfluenceSCrvaluesindependentoftheunderlyingGFR.CystatinCisincreasinglybeingusedasa
markerofrenalfunctionincardiacsurgicalpatients.

(Q.184) Lactatingwomenwithpainfulbreast,1stinvestigationtobedoneshouldbe?
(a)

USG.

http://dbmci.examonair.com/Result/ShowAllQuestionInHtml.aspx?testid=3572

92/99

22/03/2016

DrBhatiaMedicalCoachingInstitute:OnlineTestPlatform

(b)

Mammography

(c)

CT

(d)

MRI

YourResponse:

CorrectAnswer:

Exp:

USG
CanaLactatingWomanHaveaMammogram?Patricia(Pat)A.Camillo,PhD
http://www.medscape.com/viewarticle/449395
AccuracyofDiagnosticMammographyandBreastUltrasoundDuringPregnancyandLactation.Read
More:http://www.ajronline.org/doi/full/10.2214/AJR.09.3662
Amammogramisnotthebestdiagnostictoolforalactatingwomanforthesamereasonsthatitisnot
generallyrecommendedformostwomenunderage35years.Youngwomentendtohaveverydense
breasts,makingitdifficulttofindradiographiclesions.Thelactatingbreastshowsanevengreaterincrease
inparenchymaldensity,withmorenodularandropelikecharacteristics,correspondingtoductal
distentionwithmilk.
Hereisanalternativeapproachtoconsiderforlactatingwomenwithsuspectedlesions[1]:
Beginwithanultrasoundifthereisapalpableabnormality.Ultrasoundcanclearlydemonstrateasolidvs
cysticmassorgalactocele.Thiswouldinitiallyeliminatetheneedforamammogram.
Fineneedleaspirationcanbeofferedifamassisdeterminedtobeacystorgalactocele,usuallywithno
furtherinterventionnecessary.
Ifthepalpablemassappearssolidonultrasound,abiopsyisindicated.Stereotacticguidanceforfine
needleaspirationorlargecorebreastbiopsycanbeusedsuccessfullyinthelactatingwoman.Tominimize
creationofamilkfistula,theaffectedbreastshouldbepumpedjustpriortotheprocedure.
Biopsyisalsoindicatedifthemassispalpablebuttheultrasoundisnegative.
Iftheabnormalityisfoundtobemalignant,thenabilateralmammogramisdone,thepurposeofwhichis
toexplorewhetherthereareothersuspiciouslesions.
Fortunately,breastcancersarenotcommoninlactatingwomen.Lactatingadenomasandfibroadenomas
arethemostcommonsolidpalpablemassesandthesearealwaysbenign.However,whenmalignanciesare
found,theyareofteninanadvancedstagewithapoorprognosis.Thishaslesstodowiththephysiologyof
theselesionsandmoretodowiththedelayinidentifyingthem.[3]Althoughroutinemammography
screeningisnotindicated,abaselineclinicalbreastexaminationisanimportantpartofinitialprenatal
care.Changesorabnormalitiesofanykindshouldbeinvestigatedwiththesamelevelofconcern
regardlessofwhetherawomanisbreastfeeding.
TheNationalComprehensiveCancerNetwork'sguidelinesoffirstusingbreastultrasoundfollowedby
mammographywhenthesonographicfindingsareinconclusiveorsuspiciousintheevaluationof
nonpregnantsymptomaticwomenyoungerthan30yearsappearstobeanappropriateapproachtothe
evaluationofsymptomaticpregnantorlactatingwomen

(Q.185) InvestigationofchoiceinDCIS:
(a)

Mammography

(b)

CT

(c)

MRI

(d)

PET

YourResponse:

CorrectAnswer:

Exp:

MRI(RefGraingerRadiology,4thEdition,22692273)
Ductalcarcinomainsitu(DCIS)andlobularcarcinomainsitu(LCIS)areconfinedtotheductsandacini,
respectively.TherearefoursubtypesofDCIS:comedocarcinoma,micropapillarycarcinoma,cribriform
carcinomaandsolidcarcinoma.Amongthesesubtypes,comedocarcinomaisthemostaggressive.DCIS
nowaccountsfor2040%ofallcancersdetectedbyscreeningmammography.
MRImustbeabletodemonstrateandcharacterizemammographicallydetectedlesions(includingDCIS)
and,ideally,revealmammographicallyoccultlesions.ThisrequiresMRItohavehighresolution
capabilities.
CTisveryrarelyusedinbreastimaging.MRIisusedasaproblemsolvingmodalitywhenUSG/
Mammographyfailstogiveadefinitivediagnosis.Henceifthe1stinvestigationinalactatingfemaleisthe
questionasked,USGshouldbethebestoption

(Q.186) Radiologically,increasedpulmonarybloodflowisindicatedbythefollowingexcept:
(a)

Descendingpulmonaryarterydiameter>16mm

(b)

Kerleyblines

(c)

Diameterof2peripheralarteries>accompanyingbronchiole

(d)

bloodvesselinouter1/3rd

YourResponse:

CorrectAnswer:

Exp:

Noneoftheabove
AfteraextensivesearchinRadiologyliteratureincludingSutton,Grainger,Felson,emedicine,AJRand
RSNAtheclosestreferenceIfoundwasthefollowing:
ClinicalDiagnosisofCongenitalHeartDiseaseByM.Satpathy

http://dbmci.examonair.com/Result/ShowAllQuestionInHtml.aspx?testid=3572

93/99

22/03/2016

DrBhatiaMedicalCoachingInstitute:OnlineTestPlatform
FeaturesofincreasedPumonarybloodflow(PulmonaryPlethora):
Bloodvesselsbecomevisibleintheouterthirdofthelungfields.(Atleast6vesselscanbetracedtothe
outerthird).Whenhilarandintrapulmonaryvesselsareuniformlyenlargeditisverysuggestiveofshunt
lesions.
RatioofRightdescendingpulmonaryartery:Trachea>1.
Rightdescendingpulmonaryarterydiameter>14mmsuggestsincreasedflowand>17mmisvery
stronglysuggestive.
Prominentendonvesselsseenathilum
Enfacevesselsbelow10thposteriorrib.
Prominentvesselsseenbelowcrestofdiaphragm.
Ratioofvesselstoadjacentbronchus>2:1.
Sutton7thEdChap11Pg286says:
ThedifficultyintryingtodifferentiatePulmonaryarteriesfromveinsonchestxraysmeansthattheterm
PulmonaryvesselsorPulmonaryvascularityaremorepracticalthanpotentiallyhazardousattemptsto
distinguishbetweenthem.
SuttoninthesamechapteralsomentionsKerleylinesasbeingadistinctfindinginPulmonaryvenous
hypertension(Pg289).AccordingtoFelson(TheMOTHERBOOK)ofchestradiologyKerleylinesasactually
duetoprominenceoflymphaticchannels/Interlobulearseptaandcanbeseeninvariousdiseases.Most
importantamongthesearediseasesthatcausepulmonaryvenoushypertensionlikeCCF.ThusKerleyB
linescanstillbeconsideredasINDIRECTmarkersofincreasedpulmonarybloodflow.
Nowletusgotoouroptions:
OptionA:YES.Accordingtotheabovelistedcriteria.
OptionB:KERLEYLINES:Asperourabovediscussion,YEScouldbeanindirectindicator.
OptionD:YES.Againseethelistabove.
OptionC:WELL!ThespecificcriteriaaslistedaboveisRatioofVessel:Bronchus>2.1.Justsayingthatthe
vesselislargerthanthebronchusdoesnotfitthecriteria.ForegRatioof1.5:1willnotsatisfythe
criteriaabove.
HencemethinksCshouldbetheanswer.
(Q.187) ApatientofRTAwithinjuryoverchestandlimbshaslowspo2.MmodeUSofrightupperpartofchestshowsstratospheresign.

Whatisthediagnosis?
(a)

Haemothotax

(b)

Pneumothorax

(c)

Cardiactamponade,

(d)

Pulmonaryembolism

YourResponse:

CorrectAnswer:

Exp:

Pneumothorax
REF:BedsideUltrasonographyforPneumothoraxAuthor:ZinaSemenovskaya,MD
http://emedicine.medscape.com/article/1883608overview#showall
Findingssuggestiveofpneumothorax
Thepresenceofapneumothoraxischaracterizedbytheabsenceof2findings:(1)theabsenceofpleural
(lung)sliding,and(2)theabsenceofsocalledcomettailartifacts.[12]Thesocalledlungpointisarelatively
recentlydescribedsignthat,althoughdifficulttoidentify,ispathognomonicforapneumothoraxandcan
beusedtomeasurethesizeofthepneumothorax.
Absenceofpleuralsliding
Innormalpatients,thepleurallinerepresentsboththeparietalandviscerallayersofthepleura,andback
andforthslidingofthatlineiseasilyvisualizedduringtherespiratorycycle.Inthepresenceofa
pneumothorax,airaccumulatesbetweenthe2layersandblockstransmissionofsoundwaves,sothatthe
slidingisnotvisualized.Thisphenomenoncanbeseeninrealtimeinthe2Dmodebutismoreeasily
visualizedbyviewingastillimageinMmode(motionmode;seethevideobelow).
Theappearanceofnormallunghasbeendescribedastheseashoresign(seethefirstimagebelow).This
termreferstothechangeinappearancebetweensofttissueandlung,dividedbythepleuralline,achange
resemblingthatbetweensandandseawaves.Inthepresenceofapneumothorax,thisdemarcationislost,
andtheappearanceonMmodeimagingisdescribedasthestratospheresign
Absenceofcomettails
Comettailsareartifactsthatarethoughttobecreatedwhenultrasoundwavesbounceofftheinterface
betweentheapposingvisceralandparietallayersofthepleura.Theyappearashypoechoicverticalraylike
projectionsoffthepleurallineandareparalleltotheribshadowspreviouslynoted
Thepresenceofairinthepleuralspaceinhibitsthepropagationofsoundwaves,preventingthe
appearanceofcomettails.Comettailartifactsmaybehardtovisualize,requiringconsiderablepatience;
thetransducershouldbekeptinafixedlocationonthechestasthedynamiclungisobservedsliding
throughouttheexpiratorycycle.Thepresenceofcomettailsis60%specificfortheabsenceof
pneumothorax.Combinedwiththeabsenceoflungsliding,theabsenceofcomettailshasanegative
predictivevalueof100%andaspecificityof96.5%.
Lungpoint
Thelungpointisamorerecentlydescribedsignthatispathognomonicforthepresenceofa
pneumothorax.Thelungpointistheactualpointatwhichthenormallungpattern(ie,lungslidingand
comettailartifacts)isreplacedbyapatternconsistentwithapneumothorax(ie,nolungslidingandno
comettailartifacts).
Thelungpointisadynamicsignand,likethecomettails,canbevisualizedonlybykeepingthetransducer
inafixedpositionandwatchingthepleurathroughouttherespiratorycycle.Thispointshouldbesoughtby

http://dbmci.examonair.com/Result/ShowAllQuestionInHtml.aspx?testid=3572

94/99

22/03/2016

DrBhatiaMedicalCoachingInstitute:OnlineTestPlatform

longitudinallyscanningtheanterior,lateral,andposteriorpositionsofthechestwall.Althoughitisthe
mostspecificsignofpneumothorax,itisalsothehardesttovisualizeandmayrequireanexperienced
operatortolocate.Findingbothtransitionzones(fromnormallungtopneumothoraxandthenbackagain)
allowscalculationofpneumothoraxsize.

(Q.188) RegardingKawasakidisease,trueis?
(a) ItisaPolyarteritis
(b) Afeverof>37.5cshouldbepresentfor5daystomakethediagnosis
(c) Untreated,nearly2%ofchildrenwilldevelopcoronaryarteryaneurysms
(d) Intravenousimmunoglobulinshouldbegivenwithin3weeksofonsetofdisease
YourResponse:

CorrectAnswer:

Exp:

ItisaPolyarteritis
Kawasakidiseaseisaninfantilepolyarteritis.Thediagnosticcriteriaareafeverof>38.5Cfor>5days
togetherwith4ofthefollowingfeatures:
*Bilateralnonpurulentconjunctivitis.
*Oralmucosalchanges.
*Cervicallymphadenopathywithonenode>1.5cm.
*Involvementofhandsandfeetwitherythema,swellingorpeelingofthepalmsandsoles.
*Rash(polymorphus).
Thechildisusuallyextremelyirritable,withcoughorcoryzalsymptoms,andmayhavewaterydiarrhea.
Cardiaccomplicationsaresignificantcauseofmorbidity:coronaryarteryaneurysmsoccurinupto20%of
childrenwhoarenottreated.
Athrombocythaemiaisseeninthesecondandthirdweeks,andahighWCCandanaemiamaybeseen.
Managementiswithhighdoseintravenousimmunoglobulinover12hours,whichshouldbegivenwithin
10daysofdiseaseonset.Aspirinisgivenfor6weeksoruntilthecoronaryaneurysmsaregone,whichis
assessedbyechocardiogramatfollowup.

(Q.189) FollowingareusefulinRxofPPHinwomenwithRheumaticheartdiseaseexcept:
(a)

Oxytocin

(b)

Misoprostol

(c)

Methylergometrine

(d)

Carboprost

YourResponse:

CorrectAnswer:

Exp:

Methylergometrine
(RefDuttaobst.6thed447&Tableno.19,Williams,Obstetrics,21sted.,620,635)
TreatmentofPPH:
Ifuterusflabby:
a.Massageuterus
b.IVmethargin
c.Oxytocin
d.Bladdercatheterization
e.Examineexpelledplacenta
Ifstillflabby,exploretheuterus.
Injectionergometrine0.25mgorMethargin0.2mgisgivenintravenouslyfollowingthebirthofanterior
shoulder.Itshouldnotbeusedincardiaccasesorseverepreeclampsiaforfearofprecipitatingcardiac
overloadintheformerandaggravationofbloodpressureinthelatter.

(Q.190) Aprimigravidfemaleat37weeksofgestationhasmilduterinecontractions.Onexaminationshehasnoeffacementofcervixand

cervixisonly1cmdilateD.Therearearound10uterinecontractionsperhour.Sheishemodynamicallystableandnotindistress.
Whatisthemostappropriatemanagement?
(a)

Waitandwatch

(b)

TakeherforLSCS

(c)

Amniotomy

(d)

Inductionoflaborwithruptureofmembrane

YourResponse:

CorrectAnswer:

Exp:

Waitandwatch
Thisprimigravidfemaleat37weeksofgestationhasmilduterinecontractions.Shehasnoeffacementof
cervixandcervixisonly1cmdilated.Therearearound10uterinecontractions/hr.Sheisnotindistress,
hencebestmanagementwouldbewaitandwatch.

(Q.191) DiagnosisofectopicpregnancycanbemadewithpredictedbetahCGlevelsofmorethan:
(a)

1000mIU/ml

http://dbmci.examonair.com/Result/ShowAllQuestionInHtml.aspx?testid=3572

95/99

22/03/2016

DrBhatiaMedicalCoachingInstitute:OnlineTestPlatform

(b)

(c)

2000mIU/ml

(d)

2500mIU/ml

1500mIU/ml

YourResponse:

CorrectAnswer:

Exp:

1500mIU/mL
(RefTeLindesOperativeGynecology10th/pg.804)
Presently,transvaginalultrasonographyreliablydetectsintrauterinegestationsasearlyas1weekafter
missedmenses(hCG>1,500IU/L;56weeks'gestation).Barnhartandassociatesreportedthatwitha
hCGconcentrationof1,500IU/Lorhigher,anemptyuterusontransvaginalultrasonographyidentifiedan
ectopicpregnancywith100%accuracy.EvenusingadiscriminatoryserumhCGconcentrationof1,000
IU/L,Cacciatoreandassociatesidentifiedanintrauterinegestationinallintrauterinepregnanciesandin
noneoftheectopicpregnancies.Furthermore,theseinvestigatorsreportedthatthedetectionofan
adnexalmassincombinationwithanemptyuterushadasensitivityof97%,specificityof99%,positive
predictivevalueof98%,andnegativepredictivevalueof98%,providedthatserumhCGconcentrations
exceeded1,000IU/L.ThecouplingofhCGtiterswithtransvaginalultrasonographicfindingshastherefore
greatlyfacilitatedtheearlydiagnosisofectopicgestation.

(Q.192) Confinedbloodchimerismisseenin:
(a)

Monochorionicdizygotictwins

(b)

Dichorionicdizygotictwins

(c)

Singletonpregnancy

(d)

Vanishingtwin

YourResponse:

CorrectAnswer:

Exp:

Monochorionicdizygotictwins
Confinedbloodchimerismisfrequentlyseeninmonochorionicdizygotictwining.
Thewordchimerismisusedwhenanorganismcontainscellpopulationfromtwoormorezygote.Thismay
beoftruechimerismorconfinedchimerism.
Intruechimerismadmixtureof2ormorezygoteoccursveryearlyinembryoniclifeandeverytissueof
individualincludingplacentaarechimeric.
Typicalexampleistruehermaphroditewith46,XX/46,XYchromosomecomplement.Inconfinedchimerism
admixtureof2ormorezygoteislimitedtoonetypeoftissueviz.blood(confinedbloodchimerism;CBC)or
placenta(confinedplacentalchimerism;CPC).
Mechanismforchimerismcouldbeduetoplacentalvascularanastamosisoranadmixtureoftrophoblastic
cellsduringearlyblastocystdevelopment.
CBCfrequentlyseeninmonochorionicdizygotictwining.

(Q.193) Anambiguousgenitaliaina46XXfemaleisunlikelytobedueto?
(a)

Earlyantenatalexposuretoandrogensfromfetaladrenals

(b)

Placentalsteroidsulfatasedeficiency

(c)

Fetalaromatasedeficiency

(d)

FetalWnt4mutation.

YourResponse:

CorrectAnswer:

Exp:

Placentalsteroidsulfatasedeficiency.
Anambiguousgenitaliaina46XXfemaleisunlikelytobeduetoPlacentalsteroidsulfatasedeficiency.
Fetaladrenalsteroidogenesisisestablishedatleastasearlyas8weeks.Consequently,CAHgirlshave
prolongedexposuretohighfetalconcentrationsofandrogens,particularlyinassociationwithsevere
CYP21A2.
Placentalsteroidsulfatasedeficiency:Xlinkedichthyosis(XLI)isaskinconditioncausedbythehereditary
deficiencyofthesteroidsulfatase(STS)enzymethataffects1in2000to1in6000males.XLImanifests
withdry,scalyskinandisduetodeletionsormutationsintheSTSgene.XLIandcanalsooccurinthe
contextoflargerdeletionscausingcontiguousgenesyndromes.Treatmentislargelyaimedatalleviating
theskinsymptoms.ThemajorsymptomsofXLIincludescalingoftheskin,particularlyontheneck,trunk,
andlowerextremities.Theextensorsurfacesaretypicallythemostseverelyaffectedareas.

(Q.194) Mostaccuratefordiagnosisofpregnancyinfemalewith6wksamenorrhea?
(a)

Usgshowingfetalcardiacactivity

(b)

Doppler

(c)

UrineforHCG

(d)

Uterussize

YourResponse:

CorrectAnswer:

Exp:

USGshowingfetalcardiacactivity
REF:Diagnosticultrasound:Carolrumack4thEd,Chap4,Chap32:Firsttrimesterultrasound.

http://dbmci.examonair.com/Result/ShowAllQuestionInHtml.aspx?testid=3572

96/99

22/03/2016

DrBhatiaMedicalCoachingInstitute:OnlineTestPlatform
NowthedilemmaisbetweenA.USGVsB.Dopplermodeultrasound.These2methodsinvolvedirect
visualizationofthegestationalsacandfetalpoleascomparedtoUrineforHCGandUterinesize
estimation.
Thesinglemostimportantfeatureforconfirmationofembryonicandfetallifeistheidentificationof
cardiacactivity.UsingTransabdominalultrasound,cardiacactivityisalwayspresentwhentheembryois
visualizedthatiswithTASitisabnormaltovisualizeanembryowithoutdemonstratingcardicactivity.
Thecaveatisthattheexaminationmustbeofhighquality,usingmodernequipmentandtheentire
embryomustbevisualized.Itisessentialtouseahighframerateandtheframeaveragingmodemustbe
turnedoff.
UsingTVStheembryoandembryoniccardiacactivitycanbereliablyandconsistentlyidentifiedearlierthan
withTAS.AlthoughitisabnormaltoidentifyanembryowithoutcardiacactivitywithTAS,TVScanidentify
anormalembryowithoutcardiacactivity.
NowthequestiondoesnotmentionaboutTASorTVS.Nonethelesswhatinevitablyhappenswhenweturn
ontheDopplermodeisthattheFramerateofdisplaygoesdownsignificantly.Alsoat6weeksamenorrhea
thefetalpolevisualisedisonlyafewmillimetersinsize.ThusputtingaDopplerwindowonastructure
under5mminsize,andexpectingittodetectcardiacmotionIsindeedaskingtoomuch.Henceat6weeks
amenorrheaconsideringvariousfactorslike
Verysmallsizeoffetalpole(Fewmillimeters).
Needofhighframerate,withoutframeaveragingforproperimaging.
HighresolutionofroutineBmodeTAS/TVSscanning
TheMOSTACCURATEfordiagnosisofpregnancywillbeA.USGshowingfetalcardiacactivity.
(Q.195) A22yearoldnewlymarriedwomanconsultsyoubecauseofpainfulcoitus,withthepainlocatedatthevaginalintroitus.Itis

accompaniedbypainfulinvoluntarycontractionofthepelvicmuscles.Othersthanconfirmationthesefindings,thepelvic
examinationisnormal.Ofthefollowing,whatisthemostcommoncauseofthiscondition?
(a)

Endometriosis

(b)

Psychogeniccauses

(c)

Bartholinsglandabscess

(d)

Vulvaratrophy

YourResponse:

CorrectAnswer:

Exp:

Psychogeniccauses
Thispatientpresentswithvaginismus,definedasinvoluntarypainfulspasmofthepelvicmusclesand
vaginaloutlet.Itisusuallypsychogenic.Itshouldbedifferentiatedfromfrigidity,whichimplieslackof
sexualdesire,anddyspareunia,whichisdefinedaspelvicand/orbackpainorotherdiscomfortassociated
withsexualactivity.Dyspareuniaisfrequentlyassociatedwithpelvicpathologysuchasendometriosis
pelvicadhesions,orovarianneoplasms.Thepainofvaginismusmaybepsychogenicinorigin,ormay
becausebypelvicpathologysuchasadhesions,endometriosis,orleiomyomas.Treatmentofvaginismusis
primarilypsychotherapeutic,asorganicvulvarorpelviccauses(suchasatrophy,Bartholin'sglandcyst,or
abscess)areveryrare.

(Q.196) Uterineheightmorethancorrespondinggestationalagewithcomplainsofvomitingandpervaginalbleedingfavorsthediagnosis

of?
(a)

H.Mole

(b)

Threatenedabortion

(c)

Placentapreviae

(d)

Abruptioplacentae

YourResponse:

CorrectAnswer:

Exp:

H.mole.(RefDuttaObst.5thEd.206)
Hydatidiformmole/vesicularmole:
Abnormalconditionofovumwithpartlydegenerationandpartyhyperplasticchangesinyoungchronicvilli.
Itisbenignneoplasiaofchronicwithmalignantpotential.
HighestincidenceinPhilippines/[SoutheastAsia]
InIndia1in400.
Canbecompleteorincomplete.
Completemoleshave46XXKaryotype,molarchromosomesderivedentirelyfromFather(androgenous).
VesiclefluidisinterstitialfluidrichinHCG.
Thereisnotraceofembryooramnioticsac.
Vaginalbleedingcommonestpresentation(90%)
Expulsionofgrapelikevesiclespervaginumdiagnostic.
Featuresofpreeclampsiapresentinabout50%.
Sizeofuterusmorethanexpectedforperiodofammenorrheain70%,correspondswithperiodof
ammenorrheain20%andsmallerin10%.
Unilateralorbilateralenlargementofovary
(ThecaluteinCyst)in2550%ofcases.
USGSnowstormappearance.
HCGinurinebetween1in200to1in500beyond100dayshighlysuggestive.

http://dbmci.examonair.com/Result/ShowAllQuestionInHtml.aspx?testid=3572

97/99

22/03/2016

DrBhatiaMedicalCoachingInstitute:OnlineTestPlatform
FeatureCompletemolePartialmole
Karyotype46xx(46,xy)Triploid
VillousedemaAlivillisomevilli
TrophoblastproliferationDiffuseFocal
AtypiaOftenpresentAbsent
HCGintissue+++++
Behavior2%choriocarcinomaRare
(Q.197) Whichofthefollowingistheprimarytreatmentforpubertymenorrhagiaina16yrsoldgirlwith3gm%Hb?
(a)

DandCwithbloodtransfusion

(b)

Danazolwithbloodtransfusion

(c)

Progestogenwithbloodtransfusion

(d)

OestrogenandProgestogenwithbloodtransfusion

YourResponse:

CorrectAnswer:

Exp:

OestrogenandProgestogenwithbloodtransfusion
Pubertymenorrhagiaisathresholdbleedingofadolescencecausedbyexcessorunopposed
Oestrogenandabsenceofprogesteroneintheanovulatorycycle.Anaemiamaydemandbloodtransfusion
orhaematinictreatment.Oestrogentherapyyieldgoodresultsbutisnotrecommendedduetohighdose
andsideeffects.ProgestogenandOCpillsareusedduringanepisodeofbleedingaswellaslaterto
regularizeandcontrolthemenstrualbleeding.Danazoliscontraindicatedinyounggirlsbecauseitmay
causehirsutism.Ifagirlfailstorespondtohormonaltherapycurettageofendomentriumisnecessaryto
ruleoutgenitalTBwhichisseenin4%ofthesegirls.

(Q.198) A36yearsoldladywith4monthsamenorrhoeapresentswithc/oirritabilityandheadache.HerFSHandLHincreasedestradiollow

diagnosisis:
(a)

PCOD

(b)

Prematuremenopause

(c)

Ovariantumor

(d)

Pituitaryadenoma

YourResponse:

CorrectAnswer:

Exp:

Prematuremenopause
Prematuremenopauseisdefinedassecondaryamenorrhoeaforatleast3monthswithraisedFSH,raised
FSH/LHratioandlowestrogenlevelinawomenunder40yrsofage.

(Q.199) Contradiisyndromeresultsfromtheuseofwhichdruginpregnancy?
(a)

Warfarin

(b)

Lithium

(c)

Phenytoin

(d)

Penicillin

YourResponse:

CorrectAnswer:

Exp:

Warfarin
IMPORTANTDRUGSANDTHEIRTERATOGENICEFFECTS:
DRUG

EFFECT

Isotretinoin

Craniofacialdefects:Cleftlip,cleft
palate,microcephaly
CNS:NTDs,Microcephaly,hydrocephalus
CVS:ASD,Conotruncalandoutflowtracteffects
Thymicandrenalmalformations

Lithium

Ebstiensanomaly
Tricuspidatresia
Fetalgoiter

Alcohol

FetalAlcoholSyndrome:microcephaly,mental
retardation,maxillaryhypoplasia,growth
retardation,ASD,PDA,VSD,microphthalmia

Methanol

Craniofacial:microcephalyandmaxillary
hypoplasia
Mentalandgrowthretardation

Phenytoin

FetalHydantoinsyndrome:
Microcephaly,cleftlipandpalate
Hypoplasticphalanges

Warfarin

http://dbmci.examonair.com/Result/ShowAllQuestionInHtml.aspx?testid=3572

Contradiisyndrome:

98/99

22/03/2016

DrBhatiaMedicalCoachingInstitute:OnlineTestPlatform

1sttrimester:craniofacialabnormalities,
chondrodysplasiapunctate
2ndtrimester:CNSmalformations
3rdtrimester:riskofbleeding

Barbiturates

Riskofneonataldependence,congenitalheart
disease,

Tetracycline

Discolorationofteeth,bonegrowthretardation

Valproate

NTDs

Indomethacin

Prematureclosureofductusarteriosus

Stilbesterol

Clearcellvaginalcarcinoma

ACEinhibitors

Renaldamage

Thalidomide

Phocomelia

Carbimazole

Choanalatresia,patchyscalphairdefects,trachea
esophagealfistula,psychomotordelay,fetal
goiter,hypoplasticphalanges,

(Q.200) Lorcaserinisa
(a)

5HT2Cagonist

(b)

GABAinhibitor

(c)

SNRI

(d)

Noneoftheabove

YourResponse: a
CorrectAnswer: A
Exp:

5HT2Cagonist
Thisisaweightlossdrug
Serotonergicpropertiesactasanorectic
A.ItisthedrugofchoiceforobesetypeIIdiabetes.Itdoesnotcause
hypoglycemia.Q
SideEffectsmostcommonisheadache.nauseaanddiarrhoea.
ItshouldnotbeusedinCRF,liverfailure&alcoholics(duetoriskof
lacticacidosis).
ProlongusecancauseVitB12deficiency.

http://dbmci.examonair.com/Result/ShowAllQuestionInHtml.aspx?testid=3572

99/99

Вам также может понравиться